You are on page 1of 196

M.L.Krasnov G.I.

MaKarenKo
A.I. Kiseliov
;

CALCULO
VARIACIONAL
(ejemplos y problemas)

~ Editorial MIR
Los autores de este libru sen
Mijafl i<tasnov, Gri5tori Makn-
-~,ko, candidatos 8 Doctores en
Ciencias Usico-matel~átieas y do-
centes cl~l Instituto Ene~tico
de Moscú, y Alexandr Kiseliov,
colaborador- cientfficu superior
del Instituto Unlñeade de In-
vestigaciones Nucieares de la
ciudad de Duhna.
Este compendio contiene pro-
blemas y ejercicios dedieedos 11
ilustrar 108 diferentes principiffl
de la !eoria y los métodos de
resolución de las ecuaciones por
el cálculo de variaciones.
M. L. Krasnov, G. I. Makarenko,
A. l. Kiseliov

CÁLCULO
VARIACIONAL
(ejemplos y problemas)

Traducido del ruso por Carlos Vega)


candidato a doctor en ciencias físico-matemáticas

~
\~
Editorial MI R
I.S.B.N.: 84-604-1605-4
D.L: M-$299-1992
@Editorial MIR- 1992
Impresión: Gráficas Zenit - Madrid
Prefacio a la edición española . . . . . . . . . . 7
Observaciones preliminares . 9
Capítulo 1. Extremo de funciones de varias variables
§ l. E xtrerno incondicionado 11
§ 2. Extremo condictcnaoo . . . . . . . . . Hl
Capítulo 11. E¡(tremo de funcionales
§ 3. Funcional. Variación de una funcional 'i sus propie-
dades 26
§ 4. Problema elemental del Cálculo var iacronal. Ecua-
ción de Euler . . . . . . . . . . . . . . . ., 50
§ 5. Generalizaciones del problema elemental del Cálculo
variacional . . . . . . . . . . . . . . . . .. 68
§ 6. lnvarianc¡a de la ecuación de Euler . 77
§ 7. Campo de extremales . . . _ . . . . . . . . .. 80
~ 8. Condiciones suficientes de extremo de una funcional D1
§ 9. Extremo condicionado 106
§ 10. Problemas varíacionales con fronteras móviles .. , 121
§ Ii. Problemas discontinuos. Variaciones Unilaterales 134
§ 12. Teoría de H amil ton _. J acobi. Princip ¡OS var lacionales
de la Mecánica . . . . . . . . . . _ . . . . .. 143
Capítulo 1JI. Métodos directos en el Cálculo vartaclonal
§ 13. Método de diferencias finitas de Eul er . . 15B
§ 14. Método de R. Hz. Método de Kantoróvich . 160
§ 15. Métodos variaclonales para la determinación de Jos
valores y de las funciones propios . . . . . ., 167
Respuestas e indicaciones 180
BIBLIOGRAFJA
l. Courant R. y D. Hilbert, Methoden der Mathernatischen Physik
(Métodos de la Física Matemática), vol. 11, Spr inger, Berl ín, 1937.
2. L. Elsgoltz, Ecuaciones diferenciales y cálculo vartacional, Edito-
ríal MIR, Moscú, 1969.
3. 11. M. FellbfjJal.¡() u C. B. f/JOAIUH., BapH8l(HOHHOe JtC'IHCJleHHe,
<l>l:I3M3TfH3, 1961 (/. M. Guelland y S. V. Fomin, Cálculo varia-
cional).
4. E. Goursat, Cours d'analyse mathérnat lque (Curso de Análisis
Matemático), vol. !JI, 58 ed., Gauth íer-Villars. París. 1942.
5. M. A. Jlaepenmuee u Jl . A. Jliocmepnux, Kypc aap aauaonaoro
HC'IHCJH'RHSI,rOcTexH3J(3T, 1950 (M. A. Laurientieo y L. A. Lus-
ternik, Curso de cálculo v artacional).
6. J. Rey Pastor, P. Pi caun«, y A. C. Treio, Análisis matemático,
vol. UI, 211 ed., Kapelusz, Buenos Aires, 1961.
7. L. C. Young, Lectures on the calculus of varratrons and optimal
control tneory (Lecciones sobre el cálculo var lacíonal y la teoría
de control óptimo), Ph il adelph ia, London, Toronto, 1969.
PREFACIO A LA EDICiÓN ESPANOLA

Hoy día todo ingeniero tropieza frecuentemente con


problemas que requieren buenos conocimientos matemáticos
y pericia en la aplicación de distintos métodos matemáticos.
Se puede afirmar que la elevación de la cultura matemática
de los ingenieros contribuye a nuevos logros en la Técnica
El Cálculo varlaclonal es uno de los capítulos del Anál ¡sis
Matemático clásico más importante para las aplicaciones.
Actualmente, en varios Institutos politécnicos el Cálculo
variacional se incluye en el programa obligatorio del curso
de Matemáticas superiores. Existen valiosos libros sobre el
Cálculo variacional corno, por ejemplo, Jos libros de L. Els-
goltz 121, de E. Goursat (4), de 1. C. Young [7J, etc. En cuanto
a los problemas, muchos de ellos aparecen diseminados en
numerosos textos o artículos científicos especiales dedicados
a este tema. Pero, por lo que conocernos, no existe en la
literatura correspondiente ningún libro de problemas dedi-
cado especialmente al Cálculo variacional. Los autores se
han planteado la tarea de preparar un cierto minlme de
problemas referentes a los carítulos principales del Cálculo
variaclonal clásico (sin tocar as cuestiones relacionadas con
la Teoría de dirección óptima).
El libro está escrito de forma que al principio de cada
parágrafo se dan los elementos teóricos ind ispensables (defi-
niciones, teoremas y fórmulas) y se analizan detalladamente
ejemplos tí picos.
El libro contiene más de 100 ejemplos resueltos y 230
problemas para el trabajo individual. Al final se dan las
respuestas a todos problemas y, en algunos casos, las indica-
ciones correspondientes. Por eso, el libro puede servir tan to
para estudiar asignatura individualmente como para profun-
dizar en el material que se expone en las lecciones.
8 PREFACIO

Consideramos deber nuestro agradecer al traductor Carlos


Vega, candidato a doctor en ciencias físico-matemáticas, por
el gran trabajo que se ha realizado al revisar los problemas
y por las útiles observaciones que ha hecho contribuyendo
al mejoramiento de) libro.
M. L. Krasnou
O. l. Makarenko
A. l. Kiselioo
,~\O~CÚt 30 de Octubre de 1974.
OBSERVACIONES PRELIMINARES
l. Si A es un conjunto cualquiera de elementos, la proposrcron
«el elemento a pertenece al conjunto A, se representa simbólicamente así:
a E A.
Si se escribe a E A (o bien a E A). ello significa que el elemento a
no pertenece al conjunto A.
Siendo A y 8 dos conjuntos. la proposición «A es un subconjunto
del conjunto B» (A c: B, simbólicamente) significa que todo elemento x
del conjunto A también pertenece al conjunto B.
2. La unión y la intersección de dos conjuntos A ':1 8 se definen
del modo siguiente:
la unión A U B = {x I x E A o x E 8} es la totalidad de los
elementos x que pertenecen por lo menos a uno de los conjuntos A o B:
la intersección A n B = {x I x E A. x E 8} es la totalidad de los
elementos x que pertenecen tanto a A como a 8.
3. Si A es un conjunto Formado por números reales, se denomina
cota superior (cota superior exacta) de A el menor número real M tal
que a ~ M para todo a E A. En otras palabras, M es la cota superior
de A si para todo a E A es a ~ M Y si para cualquier e > 0, por peque-
ño que sea. existe como minimo un elemento b E A tal que M - e < b.
Si no existe tal número. convendremos en decir Que la cola superior
de A es +00.
En ambos casos designaremos la cota superior del conjunto A
por sup A.
Análogamente se define la cota inferior del conjunto A que se re-
presen la por inf A.
4. Se denomina espacio lineat todo conjunto R de elementos
x, y, z•... de naturaleza arbitrarla para los cuales están definidas dos
operaciones. de adición y de multiplicación por números. que cumplen
los axiomas siguientes:

2 (x
++ + +
l~ x y = !I
y)
x;
+
z = x + (y z) i
8 existe un elemento O (elemento nulo) tal que x + O = x para
todo x E R:
4) para todo x E R existe un elemento -x (elemento opuesto)
tal que x + (-x) = O;
5) l·Jt = x;
6) a (Jlx) = (a~) x;
7) (ex.+ ~)x = ax + ~x;
+ ti) = + ay.
8) ex. (x (X.X
10 OI3SEI~VACJONES PRELlMINARP.S

5. Vil espacio lineal R se llama normado si a todo elemento x E R


le corresponde un número real no negativo 11 x tI. llamado norma de
este elemento, con la particularidad de Que
1) 11 x 11 == O sólo si i(".." O;
2) Ctx 11 = I Ct , ti .e 11;
3) x -1- y 11 ~ 1I x 11 +
11 y 11 (axioma triangular para las nor-
ni a s);
6. Un conjunto M de elementos x, y, z••.. de naturaleza arbitra-
ria se denomina espacio métrico si a todo par de elementos x, y de M
le corresponde un número real no negativo p (x, y) de modo que
1) p (x, y) = o si, y sólo si, x = y (axioma de identidad);
2) p (x, y) = P (1/. x) (axioma de simetría);
3) (l (x, y) + p (y, z) ~ p (x, z) (axioma triangular).
El número p (x, y) lleva el nombre de distancia entre los elementos
x e y.
TOCio espado lineal norrnado es métrico: basta tomar p (x, y) =
= 11 x-ull·
7. El espacio e la, b] es <.>1espacio formado por todas las funciones
.r/ (x) continuas en {a. b] donde
11 y 110 = máx I y (x l.
Il~x::::b
El espacio el
[a, b] es el espacio formado por todas las funciones
y (x) Que, a parte de ser continuas, tienen derivada primera continua
en {a. bl donde
11 y IICl = rnáx
a~;¡;<b
I y (x) I + a~x<b
máx I y' (x) l·

El espacio en
la, b] es el espacio formado por todas las íunclones
y (x) Que, a parte de ser continuas, tienen en la, bl derivadas continuas
hasta de orden n-ésimo inclusive (fl es un número natural fijo) donde
7\
11 y /len =~ máx I 11(Ir.) (x) l.
k=O a~x~b

A veces la norma del elemento y (x) en en la. b) se define así:


IIIJ lIen = máx {I y (x) J. ,y' (x) 1, ...• 1yen) (x) I }.
a:!'i%~b
Capitulo 1

EXTREMO DE FUNCIONES DE VAR,IAS VARIABLES

§ l. Extremo incendicionado
Sea f {Xl' X2, •••• Xn). o brevemente f (x), una Iunción definida
en un recinto D del espacio euclídeo En de n dimensiones.
Diremos que la función f (x) alcanza su valor máximo (mínimo)
en el punto Xo E D si
f (x) ~ f (xo) (1 (x) ~ f (xo)
cualquiera que SE.'3 el punto x E D.
TEOREMA DE BOLZANO- WEIERSTRASS. Toda función continua en
un recinto acotado cerrado alcanza en él sus valores máximo y mínimo.
DEF1NICJON J. Sea f(x) una función definlda en un recinto De ECnj.
Diremos que el punto xCO) = (x y, ...• 4) E D es un punto de máximo
estricto (un punto de mínimo estricto, respectivamente) de la función
f (x) si existe una vecindad Q (xtO» del punto Xl O) lal que la desigualdad
f (x) < f (x(O» (1a desigualdad f (x) > f (xIO», respectivamente) se
cumple para todos los puntos x E g (x(o» n D. x =t= x(o). Es decir, lo
que caracteriza el punto de máximo estricto (el punto de mínimo es-
tricto, respectivamente) es que
óf = f (x) - f (x<o» < O (óf > O. respectivamente)
para todo x E Q (xlO» n D. x =F x(o).
En cambio, si para el punto xc!»~ existe una vecindad Q (x(O»)
tal que para todos los puntos x E Q (xIO) n
D se cumple la desigualdad
f (x) ~ f (XCO» (la desigualdad f (x) ~ f (x<O». respectivamente). se
dice simplemente que el punto xlO) es un punto de máximo (un punto
de minimo, respectivamente).
DEFINICION 2. Los puntos de máximo y de mínimo da la función
f (x) se denominan puntos de extremo de la misma.
J. Basándose en la definición, hallar Jos puntos de extre-
mo de las funciones
a) {(XII X2)=X~+X~;
x¡ +xi si Xi+X~* 0,
b) ¡(XI> xz)= { 1 si
xr+ x~=O;
e) f (XI' .\'"2) = xi - x~
en el recinto D {x: + x~ ~ I}.
12 CAP. 1, EXT~f;MO DE FUNCIONES

TEOREMAI (condición necesaria de extremo). Sea (x), x = j


= , Xn). Una función definida en una vecinda
(Xl> XZ, ... del punto
x,o> = (xr, xg, ... , xg). Si este punto es un punto de extremo de la
función f (xl y .si en él existen las derivadas ;~f (i = 1, '2•... , n), todas
vXj
ellas son iguales a cero:

al (xIO» =o (J'='
. I 2, .•.• n.)
i)Xj

Si la función f (x) es diferenclable en el punto de extremo xlOl• su


diferencial en este punto es igual a cero: dI (X\o,) = O.
EJEMPLO l. Hallar los puntos de extremo de la lunción z = x2 y2. +
SOLUCION. Los puntos de extremo están entre los puntos para los
cuales dz = O. En nuestro caso. dz = 2x dx 2y dy. La condición +
dz = O se cumple en el punto x = 0, y = O solamente. En efecto, sí
x = y = 0, tenemos dz = O. Recíprocamente, sea dz = O; basándonos
en que dx y dy son arbitrarios, tomemos dy = O de modo que 0= dz =
= 2x dx de donde. puesto que ax es arbitrario, resulta que x = O;
análogamente encontramos que también y = O. En el punto (O, Q)
tenemos z = O; en todos los demás puntos tenemos z = :(,1. yZ > O. +
Por eso, el punto (O. O) es un punto de minimo estricto de la función
z = XZ + y2.
Si se am p1ía 1a elase de f unciones en 1a que se busca el ex tremo
incluyendo en ella las funciones no dllerenciables en algunos puntos,
se llega a la siguiente condición necesaria de extremo.
Si x(o) es un punto de extremo de la función f (Xl. XZ, ••• , xn).
cada una de las derivadas parciales :'
Vx¡
(i = 1, 2, ... , n) es igual a cero
o no existe en dicbo punto,
EJEMPLO 2. Consideremos la parte superior z ;?- O del cono z~ =
= +
.yz. Es obvio que la función z tiene. mínimo en el punto (O, O).
Pero
X2

las derivadas ::
DEFINICIÓN 3.
y:; no existen en este punto.
Los puntos en los que se cumple la condición necesa-
ria de extremo de la [unción f (x) se denominan puntos críticos de la
misma.
Los puntos x(o) en los que dI (X,Ol) = O se denominan puntos esta-
cionarios de la función f (x).
La condición dI (x(O» = O es equivalente a la condición
al (.~IO) O (i=I.2, ... ,n).
aX¡

La existencia de punto cr itlco no garantiza aún la existencia del


extremo de una función. Por ejemplo, el punto (0, O) es un punto esta-
cionario de la función z = x - y'J. y, sin embargo, la función z no
tiene extremo en él: en cualquier vecindad del punto (O. O), por pequeña
que sea. la función toma valores tanto positivos como negativos.
§ 1 EXTREMO INCONDICIONADO 13

10• Condiciones suficientes de extremo estricto


DE!"¡ NICION 4. Se dice que la forma cuadrá Iica

n
A (x) = A (Xl> X2, ... , xn) = ¿_; o¡Jx¡Xj;
i, ;-t
0íj""" ají; i, i = 1, 2, ... , TI;

es definida positiva (definída negativa, respectivamente) si A (x) > O


(A (x) < O respectivamente) para todo punto x E En, x:::fo O. y se
anula sólo para x = O, o sea, para Xl = X9 = ... = xn_ = O.
La forma cuadrática S~ denomina no negativa si jamás toma valores
negativos. Por ejemplo, las formas

son ambas no negativas. La primera es definida positiva ya que se


anula sólo para Xl = Xz = ... = Xn = O; en carnhlo la segunda no lo
es ya que se anula, por ejemplo, para Xl = l. x2 = -1, x. = x. =
= " . = xn = O.
Una forma cuadrática definida positiva o definida negativa se
denomina forma cuadrática definida.
Una forma cuadrática que toma valores tanto positivos como
negativos se denomina indefinida.
TEOREMA 2. (condiciones. suficientes de; extremo estricto). Sea
f (x) una función definida en una vecindad del punto X,Ol =
= (xY, x], ... , x~) en la que son continuos sus segundas derivadas y sea
x(O) un punto estacionario de la función f (x). Si la forma cuadrática

( 1)

o sea, la segunda diferencial de la función f en el punto X<Ol. es definida


positiva (definida negativa, respectioamentes, el punto x<O) es punto de
mínimo estricto (punto de mdximo estricto, respectivamente): si la forma
cuadrática (1) es indefinida, no hay extremo en el punto xto).
CRITERIO DE snVESTER DE FORMAS CUADRATICAS DEFINIDAS
POSITIVAS. Condición necesaria y suiiaente
para que la forma cuadrática

(2)
14 CAr 1. ux IIH::MO DE FUNCIONL':>

eOIl aH = aJj; i, ; =- 1, 2, ... , Il; sea definida positiva es que se cumpla


aH 012 a13
a121>
«u > O.
I aH
021 a22
O. a21 022 a23
a31 a32 U:\3
>0, ...
all 012 ... a' n

a21 (122 ••• °211


.. ~, >0 .

Condicté« necesaria y suficiente para que la forma cuadrática (2)


sea definida negativa es que se cumpla

tll\<'O. ¡a 11 (llzl>o.
(12.\ D22
all al2 al3
a21 U22 a23 < 0, ...
a31 a32 a33

au al:! ..• Dln


a21 azz ... a2n
(-w·>o.

CASOn=2. Sea f (x, y) una función definida en una vecindad del


punto (xo. Yo) en la que son continuas sus derivadas parciales de segundo
orden y sea (xo, Yo) un punto estacionario. es decir, sea
f~ (xo, Uo) = fú (xo. Yo) =. O.
Entonces, si en el punto (:eo. Yo)
I'!c:r.f~y - (f;'v)~ > O.
hay extremo en este punto: a saber. máximo si en él
<O
¡';ex (f~1J< O)
y mínimo si en él
n, > O (!yy > O).
Si en el punto (%0, Yo)
t;xfúll - (f~y)" O, <
no hay extremo en E.'Ipunto (xo. Yo). Por último, si en el punto (:eo. Yo)
f';.:.:lvy - (f';,1I)2 = O,
§I EXTREMO INCONDICIONADO 15

en dicho punto puede haber extremo y puede no haberlo; este caso


requiere un estudio complementaría.
. EJEMPLO3. Consi derernos las funciones z = xf y4, z = -x' - y' +
y z = xf - y'. El punto (O, O) es un punto estacionario de las tres y
en él se tiene z:~:xZÜy- (Z;y)2. = O para cada una de las funciones.
Es fácil ver que el punto (0, O) es un punto de mínimo de la pri-
mera función, un punto de. máxlrrio de la segunda y no es punto de
extremo de la tercera. Efectivamente, en los tres casos tenemos
2 (O, O) = O; sin embargo, en cualquier vecindad del punto (0, O),
a excepción del propio punto, los valores de la función son positivos
en el primer caso y negativos en el segundo mientras que en el tercer
caso la función z = x' - y' toma, en cualquier vecindad del origen
d~'coor denadas, valores tanto positivos (por ejemplo, si x ::fo Oe y = O)
como negativos (por ejemplo, si x = O e y ::fo O).
tiJEMPLO 4. Hallar el extremo de la función de tres variables
f = x20 + +
y2 zia _ xy x - 2z. +
SOLUCIÓN. Determinamos los puntos estacionarios de la función f.
Consideremos con este fin el sistema de ecuaciones

:~ =2x-y+ I=0, )~
i)f
oy =2y-x= 0,
af
oz =2z-2=O; JI
2 I
resolviéndolo, encontramos Xo= -3' Yo = - 3' y' Zo =: l.

Consideremos 11I Iorrna cuadrática (1) en el punto Po ( _. ~ ,

--t, 1). Tenemos


)';;;0:= 2, ';,,=-1, f;z=O,
fÍlx=-I, tv" = 2, ¡~rl.= O,
t;x= O, ¡;"= o. f;z =2.
En el punto Po encontramos
an = 2, au = -1, a13 =
0,
an = -1, a" = 2, a23 = O,
a81 = 0, aa, = 0, a~3 = 2,
de modo que
> 0,
012 1_1 2 - I 1-
3 -.., O
au
1 al1
G21 a22 - l 2 - ~. ,
2 -1 O
-1 2 O =6::>0.
O O 2
16 G,\P. 1. EXTREMO DE FUNCIONES

Basándose en el crríerto de Sylvester. llegamos a la conclusión de que


la íorma cuadrática es definida positiva; por Jo tanto, en virtud del
teorema 2, el punto Po es un punto de mínimo estricto siendo f (Pu) :::::
4
=-"3'
EJEMPLO 5. Hallar el extremo de la función de dos variables
z= X'Jy2 (6-x-y).1
SOl.UCION. Determinamos los puntos estacionarios;
z~:-.' J8X2y2 _ 4xay2 _ 3X2y3 = O, }
Zy'= 12x'ly - 2~y - 3x3y2 = O,
de donde XI -=. O, Y1 O y X2 = 3, yz = 2. Hemos obtenido dos puntos
-r :

estacionarlos Pi (O, O) Y P,¿ (3, 2).


Calculemos las segundas dertvadas de la función:
z~x = 36xyz - 12x'Jy?' - 6xyS,
zÚ!J = 12.,,~- 2xt - 6x'Jy•
Z;y = 3Gx2y - 8x3y _ 9x:!y2.

En el punto PI tenernos z;x = z" y .= z;., = O de modo que


z~:x:zZY - (~¡¡)2 = O y queda pendiente eY probleIna sobre la existencia
de extremo en este punto; para resolverlo habrá que recurrir a las
derivadas superiores.
En el punto Pa tenemosz;'x = -144, Zyy = -162 Y z;;1I = -108.
Queda claro que ~xZÚY. - (zXI/)2 >
O Y como z;'x < O, en el punto
P" l3, 2) hay maxrmo siendo zmáx = 108.
Hallar tos máximos y los mínimos de las funciones:
2. f = (x - 1)2 - 2y2.
3. f = X4 + y4 - 2Xll + 4xy - 2y'l.
4. f = (x2 + !f2) e-(x2+y~}.
f I+x-y
5. = VI +x2+y2. •
y2 Z2 2
6. f=x+--¡x+y-+-z(x>O, y>O, z>O}.
7. f = x1. - xy + y2 - 2x + y.
8. f sen x sen y sen (x
= + y) (O ~ x ..::;;;n, O ~ y ~ ;1)
9. f """"XIX~ ••• x~ (l - Xl - 2X2 - . . . - nxn)
(Xl> 0, X2> O, ... , XII > O).
§ 1. EXTREMO INCONDICIONADO J7

10. Demostrar que la función z = (J + e/l) cos x - yeY


tiene una cantidad infínita de máximos y no tiene mínimos.
11. ¿Será condición suficiente para que la función f (x, y)
tenga mínimo en el punto Mo (xo, Yo) el que esta función
tenga mínimo a Jo largo de cualquier recta que pase por el
punto MI)? Considerar la función f (x, y) = (x - y2) X
X (2x _ y2).
12. Demostrar que (a diferencia de las funciones de una
variable) incluso para las funciones de dos variables la exis-
tencia de un extremo único - máximo o mínimo - en un
recinto D no significa aun que este extremo represente el
valor máximo o mínimo de la íunción respecto a todo el
recinto. Considerar los ejemplos:
a) z = X2 - yZ + 2e-.xa,
-00 <x<+ 00, -00 <y <+ 00;
b) z = XS - 4Xll + 2xy _ y2,
D {-5 ~ x ~ 5; -1 ~ y ~ 1}.
13. Sea f (x) una función periódica con período 2:rt. Entre
todos los polinomios trigonométricos de grado n
n

a; +~ (ah COS kx + ~hsen kx)


h=1
determinar, escogiendo convenientemente los coeficientes a"
y ~k, aquel que ofrece el valor mínimo para el error cuadráti-
co definido por la igualdad
re n
ó!= 2~ 1 [f(x)- i -~(akcoskx+~ltsenkx)ydx.
-n k=1
2°. Método del gradiente. Supongamos que es preciso hallar el
mfnimo de la función f (x). donde x = (Xl' X2 ••••• xm). Tomemos
un punto ¡f) = (xf, xU•... , xg.) y calculemos el gradiente de la luncíón
f (x) en este punto
m
grad f (XO) = ~ a~~~} el.
l
i-l .

donde eh ez•... , en es una base ortonorrnal del espacio R",


Sí se tiene gr ad f (xO) :;é= O. ponemos
xl = xi - h1 (grad t (¡f). eh) (k = 1, 2, ...• m),

2-01387
18 GAP. 1. EXTREMO DE FUNCIONES

donde h1 > O es suficientemente pequeño. Si se tiene grad t (~..1)*O,


ponemos

en general, si se tiene grad f (xn-l) *0, ponemos


x~ = X~-l - h,. (grad f (xn-l), eJt)
(k = 1, 2, .•. , m; hn > O).
Así obtenemos, si se cumplen determinadas condiciones, una su-
cesión monótona decreciente {I (xn}). Si xn - X y es un punto de
mínimo de la función f (x), se tiene grad f (xn) -+ O cuando n -+ oo.
x
éJEMPLO 6. Hallar el punto de mlntrno de. la función f (x) = x'.
SOLUCIÓN. Tomemos, por ejemplo. el punto xO = 1. Tenemos

Por eso, ponemos


gr ad f (xO} = 2xOI = 21 O. *
xl- = xO - h 2 = 1 - 211. donde h > O.
Tenemos ahora
grad f (Xl) = 2 (1 - 211) l.
Si h * !' es grad f (Xl) *° y ponemos
x? = Xl - 2h (1 - 2h) = (1 - 2h)2.
Continuando este proceso. encontramos
xn = (1 _ 2h)n.
Es claro que, siendo 0< h < 1, se tiene xn - O para Il_ oo. El
punto x = O es el punto de mínimo de la función f (x) = XS.Si" = -}.
°
es Xl = O y gead f (xt) = y obtenemos la sucesión estacionaria {O}
cuyo límite es el cero.
EJEMPLO 7. Hallar el punto de mlnírno de la función f (x, y) =
= x:A + y2.
SOLUCIÓN. Tomemos. por ejemplo, el punto (l. 1), o sea, tomemos
XO = 1 e /jo = 1. Tenemos
grad f (1, 1) = 21 + 2j.
Puesto que grad T (1. 1) *0, ponemos
Xl = xO _ 2xo" =
1 - 2h,
(h > O)
yl = !l - 2!lh =
J - 2h.
Tenemos
.
grad f (xl, yl)=2(1-2h)l+2(1-2h)j=t- O
y. por eso, tomamos
2x1h = (1-- 2h)2,
x2 = xl_
y2=y1_2ylh=(I_2h)2.
2. EXTREMO COND1CIONADO 19

Continuando este proceso, encontramos


xn = (1 - 2h)U,
Un = (1 - 2h}n,
de modo que para O < h < 1 obtenemos una sucesión de puntos
Mn (x"", ",11) convergente al punto M (O, O) de rnlnímo de la [unción
considerada. Es obvio que
+
grad f (xn, un) = 2 (1 -_ 2h)n ¡ 2 (1 - 2h)'n j -+ O cuando n -.. oo ,
Es decir, el punto (O, O) es el punto de mínimo de la lunción { (x, y) =
;:: x~ + U2•
Empleando el método de gradiente, hallar el punto de mínimo
de la Iunclón
z = x2 + y'J. _ 2x + 4y + 5.
§ 2. Extremo condicionado
Sea z = 1(Xl' X2 ••••• Xn) una [unción de n variables definida de
un recinto D del espacio En.
Supongamos. además, que las variables Xl. X2> ••• , '"'71 están
ligadas por (m < n) condiciones complementarias

:1_<X~' .x~':.:' "~~)~~'} (1)


C¡>m (x •• "2, ... , xn) = 0,
que se denominan ecuaciones de enlace.
Sea %(0) = (xi, x~•... , x~) un punto interior del recinto D.
Se dice que f (Xl. Xs, ••• , xn) tiene máximo condicionado (mínimo
condicionado, respectivamente) en el pun to (x~, ~ •... , x~) si la de-
sigualdad
f (Xl. X2. • ••• Xn) ~ f (x~. x~, ...• xi\.)
{la desigualdad f (Xl. Xa, ... , Xn) ~ f .(xY. X~, ' •.• X~), respectívarnen-
te) se cumple en una vecindad del punto (.x~, x~, ' . _, x~) siempre que
los puntos (Xl' X2 •••• , Xn) y (X~. X~ •••• , xV.) verifiquen las ecuaciones
de enlace (l).
EJEMPLO 1. La función z = X2 +
yo¡, tiene mínimo incondicionado,
igual a cero. en el punto (O, O). Agreguemos la ecuación de enlace
x + !I - 1 = O; se trata entonces de determinar el rnínirno de las
Z-coordenadas de los puntos de la superficie z = x~ + y'J. considerando
s6lo aquellos valores de x y de y que satisfacen la ecuación x +
!I - l =
= O. Este mínimo condicionado no se puede alcanzar en el punto
(O. O) pues este último no satisface la ecuación de enlace. Resolviendo
la ecuación de enlace x +
U - I = O respecto a y e introduciendo la
expresión obtenida y = \ - x en la ecuación de la superficie, encon-
tramos z = x2. +
(I - x)2.. o sea, obtenemos una función de una
variable. Extremándola, encontramos xer = ~ y Zmlll = ~. A partir
~*
20 CAP. 1. EXTREMO DF. FUNCIONES

de la ecuación de enlace, determinamos Ycr = ;. El punto ( ;, ;, ~ )


es el vértice de la parábola que corresponde a la intersección del para-
boloide z = X2 +
y'l.. con el plano x y - 1 = O.+
Análogamente se puede proceder en situaciones más generales.
Supongamos que se busca el extremo condicionado de la función
z =f (x, y) síendo (jl (x, U) = O la ecuación de enlace. Supongamos que
para los valores considerados de x y de IJ la ecuación q> (x, y) = O deter-
mina !I corno una función unívoca diferenciable y = 'P (x). Sustituyen-
do y por 'i> [x) en la función I (x, y), obtenemos una función de una
variable x: z = f (x, 'l> (x» = F (x). El extremo (incondicionado) de
la función F (x) sera el extremo condicionado buscado de la función
f (x, y) con la condición de enlace (jl (x, y) = O. En la práctica este
método resulta poco cómodo ya que para ap licarlo es preciso resolver
la ecuación (jl (xo y) = O respecto a una de las variables.
Para hallar los extremos de la función z = f (Xl. ;\:2, ... , Xn) con
las condiciones de enlace (1) se emplea el método de los multiplicadores
de Lagrange.
MeTODO DE LOS MULTIPLICADORES DE LAGRANOE. Supongamos que
1) las derivadas pardales de primer orden de las Iunciones
f (Xi> x2, ••. , xn) y epi (Xi, X2, .. "Xn) (1 = 1,2, ... , m) son continuas
en el recin lo D;
2) m < n, srendo el rango de la matriz (~:;) (i = J, 2, ... , m;
1= 1,2, ... , n) Igual a ni en todo punto del recinto D.
Consideramos una función nueva (lunción de Lagrange)
111
. -"':1
<D •.=-J+ k.J "'[<rio
1.-1

donde "" son factores constan les indeterminados.


Después analizamos el ex tremo incondicionado de la íunción
<f) (XJ, xz, .... xni, o sea. formamos el sistema de ecuaciones
éJ<I> o;,tJ o<l>
-,,- = O, -a = 0, •.. , -:;¡-- = o
UXi ~
(2)
vXn

'1, a partir de este sistema y de l as m ecuaciones de enlace


(PI = 0, tpll = O, .. IPm. = O, '0

determinamos los valores de los parámetros "lo 1..2•••• , Am y las coor-


denadas (Xl. X20 ••. , xn) de los posibles puntos de extremo.
Las condiciones (2) son condiciones necesarias de extremo tanto
para la función de Lagrange como para la [unción inicial z =
= f (Xl' X2, .•• , xn)·
Si el punto (4 X~, ..• x~) es un punto de extremo condicíonado
0

de la [unción l {Xl' XZo' ., xn l. será a la vez un pun lo estacionario de


1a función de Lllgr auge, o sea, en este pun to será diJ· ~

·'<i
= O (i = 1, 2, ...
... , tl). Para analrz ar el punto estacionario (xY. x~, ...• XYI) en tanto
§ 2. EXTR.EMO CONDICIO"lA.DO 21

que ex tremo condicionado de la Iuncíén de Legrange (1)(Xi' X2, ..• , xl1)


habrá que considerar la Iorma cuadrática
n,.. tn
B(dxt. dX2•... , lLl:n-m}= ~ /JIJ,1x¡ dx], (3)
i. ;-1
o sea, la segunda diferencial de la función de Lagrange, teniendo en
cuenta las condiciones
01'1 oq>'
-¡¡- dXt
v,\t
+TX2
v
dx,¿~ ". (i= 1, 2, ... , m).

Si la lorma cuadrátlca (3) es definida, en el punto (x~. xg, ... , x~)


tendremos extremo condicionado estricto; a saber, máxime condicio-
nado estricto si la forma cuadrática (3) es definida negativa, y mlnimo
condicionado estricto si la forma cuadrática l3) es definida positiva.
En cambio si la forma cuadrática (3) es indefinida, en el punto
(x~, x~, ... , x~) no habrá ex tremo condicionado.
Por consiguiente, la existencia en el punto (x.,. x~, ... , x~) de
máximo (mínimo) incondicionado de la función de l,agrange (tomada
con los valores encontrados para)...1>Aa, .. " ~'nl)implica la existencia
de este punto máximo (mínimo) condicionado de la función z =
= f (Xi' X2, ••• , Xn) con las condiciones de enlace
<Pi (Xl. X2 •••.• XII) = O (l = l. 2 •.. " m).
La ausencia de extremo incondicionado de la función de Lagrange
el> (Xl. Xa •.•. , xn) no significa aún la ausencia de ex tremo condicionado
de la (unción f (XI. XZ, •••• Xn):
EJEMPLO .2. Hallar el extremo de la función z = xy con la con-
dición y - x = Q.
SOLUCION. Formamos la función de Lagrange
<D (x, y) = xy + '"(y - x)
y el sistema correspondiente para deterrnlnar 'k 'j las coordenadas de
los posibles puntos de extremo

~~ =Y-'k=O,}
O<D
ay =x+1..=O,
y-x=O.
La primera ecuación da A = y. Teniéndolo en cuenta, encontramos la
segunda ecuación x + y = O. Es decir.
x+y= O, }
y-x=O,
de donde x = y = O y. además, A = O. Por lo tanto. la (unción corres-
pondiente de Lagrange es <D (x, y) = xy. La función <D (x, y) no tíene
extremo Incondicionado en el punto (O. O).
22 CAP. l. EXT"REMO DE PUNCIONES

Sín embargo, existe el extremo condicionado de la runción z = xy


con la condición y = x: en efecto, tenemos en este caso z = x', de
donde resulta que hay mfnlmo condicionado en el punto (O, O).
EJEMPLO 3. Hallar el extremo condicionado de la {unción
f (x, y, z) = xyz
con las condiciones
<PI (x, !I, z)=x+y-z-3=O, }
<r2 (x, y, z)=x-y-z-8=O. (4)
SOLUCION. Formamos la función de Lagrange
+
ct> (x, y, z) = xyz "'1 (x +
y - z - 3) + /..9 (x - y - z -
y el sistema de ecuaciones para determinar los parámetros At
8)
y Aa
y Ias coordenadas de los posibles puntos de extremo

: =YZ+/..I+~=O, )

: ~-"+'I-"~O,
T=XY-Ai-~=O'I
I (5)

x+y-?-3=0,
x-y-z-8=O ..
Resolviendo el sistema de ecuaciones (5), obtenemos
11 231 11 5 11
Al ='"32 , ~=-32' x=-;¡-, y=-y y z=-4'
la segunda dllerencial de la íunclén (!> (x, y, z) es ig\lal a
(}2(Jl i}zlD é)za.
=--
d2<.D
i}.x'Z
dxz+ --dg2
ayZ
+-- iJz2 oz2 +
iJ2(D ~$ ~$
+2-ax vy
;, dxd!l+2~dxdz+2-a
<i!l oz
'" dydz.
y oz
En nuestro caso,
tl2cD = 2z dx dy 2y dx dz +
2x dy dz. + (6)
Oc las ecuaciones de enlace (4) encontramos
dx+dy-dz=O, }
dx-d!l-dz=O,
de donde dx = dz, dy = O. Introduciendo estas expresiones en (6),
obtenemos
8 (dx) = 2y dx2.
En el punto estacionario se tiene B = - 5 dx2 <O, o sea, en el punto
11 5 11 ) . 605
( T' -'2' -T hay máximo, Siendo / mAx= 32.
§ 2. EXTREMO CONDICIONADO

EJEMPLO 4. Hallar el extremo de 13 función z = ('(>:;2 X -1- cos2 !I


con la condición
n
y-x=T'

SOLt:C1ÓN. Formamos la función de Lagr ange

tD(x. y)=cos2..-.;+cos2y+]" (y-x-:)


y el sistema de ecuaciones para determinar el parametro ,< 'i la~ coor-
deuadas de los posibles puntos de ex tremo

~~ ~~-2cosxsenx-;\-O, )
al11
i)y -v-: -2 cos y sen v+ ;'=0, ~
n .
y-x--=O,
4 J
es decir,
sen 2x= -A, (7)
sen 2y=A. (8)
n
y-x=T' (9)

De las ecuacloncs (7) y (8) tenemos sen 2x sen '2y =. 0, o sea, +


2sen (x +
y) cos (y - x) = {l. (10)

Debido a (9). tenemos cos (y - x) = }~~=fo ° y, ¡JOf eso, de (10)


resulta sen (x + y) = 0, es decir,
x + y ~ kn, k = O, ± 1, ±2, (11)
Resolviendo las ecuaciones (9) y (11), tendremos

Determinamos las se¡,:ulldas derivadas de la Iunclón IlJ (x, y):


1]2(!l t)2q)
ox = - 2 cos 2x,
--=-:--"2
ox cJy =O,
-;¡--;;-

i.J2tD
ay2 =- 2 ces 2y.

kn n krr. n )
En los puntos PIi. ( 2-'8' '2+8 se tiene
24 CAP. J. EXTPEMO DE FUNCIONES

(!);:t(!)~V _(tl>;1/)2 = 4 cos ( k1f. - : ) ces ( kn + ~)=


= 2 cos 2kn = 2> O.
Por consiguiente, hay extremo condicionado en los puntos Pilo Además.
para k:= 2n es

aiJx2Cl>2 1
P2n
= -V2<O
y. pOC I!SO, en los puntos P-an se tiene máximo condicionado siendo

..'"max-_1+V2.
-2- t

para x= 2n+ 1 es

a2Cl>
7fT
X
I J)zn+l
= V-2>0,
o sea. en los puntos Ptn+ 1 tenemos mínimo condlcíonado siendo

zmln= 1--2-,
V2

En los problemas que siguen hallar el extremo condicio-


nado.
t 4.f = xy siendo x'J + .112 = l.
15. f=x2+!l' siendo ~ + ~= l.
16. f = xyz siendo x + y + z = 5 Y xy + yz + zx = 8.
17. f = eXY siendo x + y = a.
18. f = 6 - 4x - 3.11 siendo x'J. + .112 = 1.
19. f = x - 2.11 + 2z siendo X2 + + y2 ZZ = 9.
20. f
sen x sen y sen z siendo x
= + y + z = ~, x> O,
y> O y z> O.
21. Demostrar la desigualdad
n~ 1, x~O e y~O.
§ 2. E XTR,EMO CONDICIONADO 25

22. Hallar el valor máximo del producto xyzt de cuatro


números no negativos x, y, z y t si la suma de los mismos
permanece constante: x + y + z + t = 4c.
23. Hallar la distancia mínima del punto M (1, O) a la
elipse 4x2 + 9y2 = 36.
24. Hallar la distancia de la parábola y = XZ a la recta
x-y = 5.
25. Hallar los lados del rectángulo de área máxima
inscrito en la circunferencia X2 + y2 = R'J,
26. Inscribir en la esfera de radio R el cilindro de máxima
superficie total.
Capítulo JI

EXTREMO DE FUNCIONALES

§ 3. Funcional. Variación de una funcional y sus


propiedades
1°. Definición de funcional. Proximidad de curvas. Sea M una
clase de funciones y (x). SI a toda {unción y (x) E M le corresponde,
según una regla, un número determinado J se dice que en la clase M
está definida la funcional J y se escribe J = J ly (x)].
La clase M de funciones y (x) en la que está definida la funcional
J ry
(x)] se denomina campo de definiciÓn de la funcional.
EJEMPLO 1. Sea M = e [0, Ir el conjunto de todas las [unciones
continuas y (x) definidas en el segmento [0, IJ Y sea
I
J ly(x}1= j y(x}dx >.
l (1)
o
Entonces J Iy (x)} es una funcional de y (x): a toda [unción y (x) E
e
E [0, 1]le corresponde un valor determinado JIu (x)]. Tomando en
(l) funciones concretas en lugar de y (x), obtendremos los valores corres-
pondientes de J r"l. Por ejemplo, si y (x) = 1, tenemos
1
J[l)=) l·dx=!;
O
si y (x) = e», tenemos
I
J (eX) =) eXd.x=e-l¡
O
si y (x) =cos nx, tenernos
1
J(cosnx)= Jo cosnxdx=O.

1) En adelante, al considerar funcionales in tegrales, escribiremos


en el integrando y en lugar de y (x), y' en lugar de y' (x), y etc.
§S. FUNCIONAL. VA~JACJON DE UNA FUNCIONAL 27

EJEMPLO 2. Sea M = el [a. b) la clase. de funciones y (x) que


tienen derivada continua en el segmento [a, b) y sea
J [y (x)l = /j' (xo) , donde Xo E la, b¡.
Queda claro que J [y (x)J es una funcional deíinl da en la clase de
funciones señalada: a toda función de esta clase le corresponde un
número determinado, el valor de la derivada de esta función en el
punto fijo xo.
Siendo. por ejemplo, a = 1, b = 3 y xo = 2. tenemos para
y (x) = X2

para IJ (x) = +1 encontramos J {xZ + 11 = 4 Y para Y (.~) =


I x-Z='3'
X2

= In (1 + x)
tendremos J {In (1 + x») = I
l+.x l
EJEMPLO 3. Sea M = e [-l. l] la clase de funciones y (x) con-
tinuas en el segmento [-l. 1)Ysea cp (x, y) una función definida y
continua para todos los -1 x::;;;; 1 Y para todos los valores reales
::;;;;
de y. Entonces.
1
J [y (x)) = j cp (.~, y) dx
-1

será una funcional definida de la clase de funciones indicada. Por


ejemplo, si cp (x, y) = 1 ~yz' para la función !I (x) = x tendremos
t

J!x]= Ji x dx
l+x2 =0 y para y(x)=I+x
-1

tendremos
I

1[I+x)=
r
J
x dx
1+{I+x)2 In V5- arctg 2.
-1

EJEMPLO". Sea M = el [a, b) la clase de funciones y (x) que tienen


derivadas continua yl (x) en el segmento {a. bl. Entonces
b
Jfy(x)]= I
a
VI+y'2dx (2)

será una funcional definIda en esta clase de funciones. Desde el punto


de vista geométrico, la funcional (2) representa la longitud del arco
de la curva !J = y (x) cuyos extremos son los puntos A (a. y (a) y
B (b, y (b».
kl Se denomina variación o incremento óy (x) del argumento y (x)
de la funcional J (y (x)lla diferencia entre dos funciones y (x) e Yo (x)
28 C!\P. 11. EXTREl>lO DE FUNCIONALES

pertenecientes a la clase considerada M de funciones:


ay (x) = y (x) - Yo (x) 1).
Para la clase de funciones k veces dítercncrables tenemos
(<'3y)(II) = 6y(lI) (x).
Diremos que las curvas {/ = y (x) e !I = 9, (x) definidas en el
segmento la, bl san cercanas en. el sentida de proximidad de orden nulo
si es pequeña en la. bl la magnitud I !I (x) - Yl (x) l. Desde el punto
de \' ista geo mét rico. esto significa que son próximas las ordenadas
de dichas curvas en la. bJ,
Diremos qu~' las curvas y = !I (x) e y = {/l (x) definidas en el
segmento la. bl son, cercanas en el sentido de proximidad de primer orden
si son pequeñas en [a. bl las magnitudes I !I (x) - Yl (x) I y
I y' (x) - yí (x) I Desde el punto de vista geométrico, esto significa
que en la, b 1 son próx imas tanto las ordenadas de dichas curvas C0l110
las direcciones de sus tangentes en Jos puntos correspondientes.
Las curvas 11=. !I (x) e y = Yl (x) son cercanas en el sentido de
proximidad de k-ésimo orden si son pequeñas en [a. bl1as magnitudes
I y (x) - Yt (x) J, I y; (x) - Yi (x) 1. ... , I yl"-) (x) - y¡lI:) (x) l.
Si las curvas son cercanas en el sentido de proximidad de k-ésimo
orden, con mayor razón lo serán en el sentido de proximidad de cual-
quier inferior.
11
EJEMPLO S La curva 1/ (x) = sen n x con n suñctenternen te grande
- rt
Y la cUf\'a!JI (x) ~. O son cercanas en [O, n] en el sentido de proximidad
de orden nulo ya que
senn2 x I
ly(x)-yJ(x)l=
! rt ~n'
o sea, el valor absoluto de esta diferencia es pequeño en todo el segmen-
to [0, ni si n es suücíenternente grande.
No hay proximidad de primer orden ya que
I y' (x) - uí (x) I = n I cos n'J.x I
y, por ejemplo, ('11 los puntos x = !~ tendremos I y' (x) - Yí (x) I "",
=. 11,o sea, I y' (x) - y; (x) I puede resultar tan grande como se quiera
si n es suf iclentemento grande.
EJEMPLO 6. La curva y (x) = se~2nx con n sullctentemente grande
y la curva Yl (x) ..",. O SOI1 cercanas en (0, n] en el sentido de proximidad
de primer orden ya que. tanto

¡
I y (x)-Yt (x) 1= sen/12nx <fj2 I I

1) Para abreviar, en lo que sigue escribiremos simplemente 6¡¡


en Jugar de By (x).
, 3. FUNCIONAL. VARIACiÓN DE UNA FUNCIONAL 29

como
I y , (t) - Yl• (x) 1=lcosl1xII
-11- <: n
son pequeños.
Determinar el orden de proximadad de las curvas en los
problemas que siguen.
cos nx
21. U(x)=
n2+1
e Uf (x) 55! O en [O, 2nl.
sen x
28. y(x)= 11
e Y. (x) = O en [O, nI.
x e en [O, 1J.
29. U(x)=sen- 11
Ut (x) =0
Se denomina distancia entre las curvas y::::: y (x) e y = Yl (x)
(a =o;;; x =o;;; b), donde y (x) e 111(x) son funciones continuas en [a, bl.
el número no negativo p igual
al máximo del módulo I Yl (x)- y
M (l. 1)
- 11 (x) I en el segmento
a~ x~ b:
P= P [Ul (x), U (x)] =
= máx Iys(x) -y (x) l.
o::>'~b
7. Hallar la dis-
EJEMPLO
tancia p entre las curvas
y (x) = x e !/l (x) = Xi en el
segmento [O. I J (Hg. 1). )(
SOLUCIÓN. Según la defini-
ción p = máx t r-x
1, osea,
O::;x~ I Fíg. I
P= máx (x - x'). La Iun-
O::>':I<~l
ción y = x - x' se anula en los extremos del segmento 10. 1].
Determinemos el máximo de la íuncíón y = x - X2 en el segmento
te, l}.
Tenemos
1
y' = 1- 2x e v' = O para x = "'2
de modo que

p= máx Y = <X-X2)1
O~x~t :1;-
t
'2
=+.
En los problemas que siguen hallar fa distancia entre
las curvas en los segmentos indicados.
30 CAP. 11. EXTRF.MO DE FUNCIONAI,F.!;

30. !J (x) = xe:" e Yt (x) == O en [O, 2].


3.. Y (x) = sen 2x e !1i (x) = sen x en [o, ~ l
32. y (x) = x e !J1 (x) = In x en rrl, ej.
Supongamos que las curvas y = y (x) e y = 111 (x) tienen en el
segmento la. bl derivadas continuas de orden n.
Se llama distancia de n-ésimo orden entre las curvas y = 11 (x)
e y = 111 (x) el mayor de los máxl mas de 1as exprestones
, Yl (x) - y (x) ,•. 1 y; (x) - y' (x) 1, ... , 1 y\n) (x) - ycn) (x) 1
en el segmento [a, bl. Representemos esta distancia así
Pn = Pn IYI (x). y (x») = rnáx máx I yiR) (x) - y(ll) (x) l.
O~/¡:'.f.n a~x:'.f.b
Desde este punto de vista se puede interpretar la distancia definida en
la pág. 29 como distancia de orden nulo.
EJEMPLO 8. Hallar la distancia de primer orden entre las curvas
y (x) = x' e 111 (X) = x3 en el segmento O ~ x ~ l.
SOI.UCIÓN. Calculemos las deriv adas de las funciones dadas:
y' (x) = 2x e Jlí. (x) = 3X2 y consideremos las funciones y, (x) =
= Xi - x3 e IJI (x) = 2x - 3Xil. Determinamos sus máximos valores
en el segmento lO. 11. Tenemos y; = 2x - 3x2.

fig. 2

Igualando esta derivada a cero, encontramos los puntos estacio-


narios de t a función Jl2 (x) : .tl = O y Xz = ~. i\hora bien, Y2 Ix=-o = O,

yll a =..i.. y el valor de Jli (x) en el extremo de la derecha es


:t'~- 27
3
§ a. FUNCIONAL. VARIACiÓN DE UNA fUNCIONAL 31

Ya (1) = O. Por eso,

Determinamos ahora la distancia Po de orden nulo entre las derivadas


/j' (x) = 2x e Yi (x) = 3xs:
Po = máx I Y3 (x) 1= máx I 2.x-3x2 1-
O~%~t O~x~i
Consideremos el gráfico de la función y, = I 2x - 3xs I (ligo 2). Puede
verse de él que Po = l. Por consiguiente, la distancia Pl de primer
orden entre las curvas y (x) = Xi e Yl (x) = r será iguál a
PL = máx (Po, po) = l.

33. Hallar la distancia de primer orden entre las curvas


y (x) = In x e Yl (x) = x en el segmento l, ej. re-
34. Hallar la distancia de segundo orden entre las curvas
y (x) = x e Yl (x) = -cos x en el segmento [0, ~ ]
35. Hallar la distancia de lOOl-ésimo orden entre las
curvas y (x) = eX e Yl (x) = x en el segmento [0, I J.
Se llama e-vecindad de n-ésimo orden d.e la curva !I = IJ (x)
(a ~ x:S:;;; b) el conjunto de las curvas y = 1J1 (x) cuyas distancias de
n-éstrno orden a la curva y = !I (x) son menores que e:
Pn = Pn llJ (x), Yl (x)) < e,
La s-vecindad de orden nulo se denomina e-vecindad fuerte de la
función y = y (x).
La e-vecindad fuerte de la curva IJ = Y (x) está formada por todas
las curvas comprendidas en la franja de 28 de anchura construida a
partir de la curva y = y (x).
La s-vecindad de primer orden se denomina e-vecindad débil de
la función. 11= Y (x).
2°. Continuidad de una funcional. Una funcional J I.IJ (x}) definida
en la clase M de funciones y (x) se llama contlnua en y = Ye (x) en
el sentido de proximidad de n·ésimo orden si cualquiera que sea el
número 8 > O existe un número "l > O t-al que la desigualdad
I J [y (x)] - J l.!Io (x)] I < 6 se cumple para todas las funciones admi-
sibles y = y (x), o sea, para todas las funciones que satisfacen las
condiciones
I !I (x) - !lo (x) I < "l. I y' (x) - Yó (x) I < 11. . ..
. . " I y(n) {x} - y~n) (x) I < 1'].
En otras parabras, si se tiene I J ly (x)l - J Iyo (xH I < e siempre que
{In [y (x), !lo (x)1 < 1).
32 CAP. TI. EXTREMO De FUNCIONALES

Toda funcional que no sea continua en el sentido de proximidad


de Il·ésimo orden se denominará discontinua en este sentido de proxl-
mldad. Poniendo
/j("') (x) = y~ll) (x) + <xw(k) (x) (k = O, 1, 2, ... , n),
donde <X es un parámetro y w (x) es una función cualquiera de la clase
M, podemos persuadirnos de que
Iím ylh} (x) = y~k} (x) (k = O. 1,2, .•. , n)
a;... 0.l

y, por eso, podemos deñnir la continuidad de la funcional J fy (x)]


en y = Yo (x) de la forma siguiente
lim J [Yo (x) <xw (x)) = J {Yo (x»).
a...O
+
EJEMPLO 9. Demostrar que la funcional
t
l'
Jly(x)l= ~ (1I+2y')dx
o
considerada en el espacio CI 10, 1( es continua en la funcian Yo (x) = x
en el sentido de proximidad de primer orden.
SOLUCIÓN. Tomemos un número cualquiera e > O y demostremos
que existe un número r¡> O tal que I J Iy (x)) - J [x) < e siempre
que I IJ (x) - x I < '1 Y 1 y' (x) - I 1 < 'l. Tenemos
1
IJ[I/(X))-J(X]I=I ~ (Y+2yl-X-2)dx
o
l<
t 1

~ Jo Jy-xJdx+2 J Iy'-lldx.
o
e
Tomemos r¡=3' Entonces, para todas las funciones y (x) ECdO, II
tales que
e e
Iy (x)-x 1< 3 e I.!I' (x)- 1 1< 3
tendremos
I J [y (x») - J [x] < 8.
Es decir, para todo 8> O existe un número 1') > O (por ejemplo,

r¡= ~) tal que I J (y (x)) - J [x] I que Pt Iy (x), x] <


< 8 siempre
< 'l. Pero, según la definición, esto significa precisamente que nuestra
funcional es continua en la funci6n Yo (x) = x en el sentido de proxí-
I 8. FUNCIONAL. VARIACIÓN DE UNA FUNCIONAL

mldad de primer orden. Es fácil ver que esta funcional es continua e


el sentido de proximidad de primer orden en cual quier curva y (x)
E C, lO. 1).
EJEMPLO 10. Consideremos la funcional
I [g (x)} = y' (xo).
donde las funciones y (x) E C, [a, b] y Xo E la, b).
Esta funcional es discontinua en el sentido de proximidad de
orden nula en cualquier función 11 (x). Efectivamente, escogemos cp (x)
de modo que <p' (xo) = l Y que I <P (x)' < 11 en el segmento la, b}.
Consideremos la función y (x) = Yo (x) + <p (x), donde !lo (x) E
el
E la, bl. Entonces tendremos y' (xo) = lió (xo) l. Es obvio que +
P (y (x), !Jo (x») < ", O sea. que las curvas y (x) e Yo (x) son cercanas en
el sentido de proximidad de orden nulo. Al mismo tiempo J Iy (x)) -
- J [Yo (x») = 1, es decir, los valores de la funcional no son próximos
por cercanos que sean, en el sentido de proximidad de orden nula. los
argumentos y (x) e Yo (x).
Hablando con más precisión, existe un 8 >
O (por ejemplo, cual-
quier 8 < 1) tal que para cualquier" > O existirán funciones y (x)
para las cuales
Po [y (x), Yo (x)} < 1] y, sin embargo, , J fu (x)! - J (Yo (x)] I ~ 8.
Esto significa precisamente que la funcional J (y (x)) es discontinua
en el sentido de proximidad de orden nulo.
Demostremos que esta funcional es continua en el sentido de
proximidad de primer orde t.
Tomemos un & > O cualquiera. Tendremos
I J (y (x)] - J (Yo (x») = I y' (xo) -!l~ (xo) ¡.
Queda claro que tomando 11 = 8 tendremos
I J [O (x)J - J ¡Yo (x») I <e
siempre que p, [y (x), Yo (x) J < r¡ que es lo que se queria demostrar.
Este ejemplo permite ver que de la continuidad de la funcional en el
sentido de proximidad de n-ésimo orden no implica, hablando en tér-
minos generales, su continuidad en el sentido de proximidad de orden
Inferior,
EJEMPLO 1). Consideremos Ia funcional
R

J 1.1>' (x)] = Jo y''J. dx

definida en el espacio el [ü, n]. Demostremos que es discontinua en la


función Yo (x) == O en el sentido de proximidad de orden nulo.
_ sennx
En electo, sea Yo (x) ;: O en lO, nJ y sea {lit (X) Enton- = ---.n
1
ces, Po (Yo (x), Yn (x)) ::= -
n de modo que Po ~ O cuando II ~ co,

3-01381
34 CAP. 11. EXTREMO DE FUNCIONALES

Por otro lado, la díferencia


11
n
J [Yn (x)\-J (Yo (x)] = J\ cos2nx d
n x=2"
O

no depende de fi. Es decir, J [Yn (x)) no tiende hacia J [Yo (x» = O


cuando n -- 00 Y. por consiguiente, nuestra funcional es discontinua
en la {unción Yo (x) == O en el sentido de proximidad de orden nulo.
Proponemos al lector demostrar que esta funcional es continua
en la función Yo (x) == O en el sentido efe proximidad de primer orden ..

Analizar la continuidad de las funcionales siguientes


36. J fy (x)J = y (xo), donde y (x) E C [a, bl y Xo E la, b),
en el sentido de proximidad de orden nulo
37. J fy (x)J = rnáx I y (x) 1, donde y (x) son funciones
continuas en el segmento la, bJ, en el sentido de proximidad
de orden nulo.
38.
V si y (x) toma al menos un valor negativo,
J [y (x») = -4- si y (x) == 0,
{
1 si U(x)~O e y(x)9É0,
en el sentido de proximidad de orden nulo
1
I I
39. J (U (x)] = ) y' dx, donde las funciones y (x)
o
tienen primera derivada continua en el segmento [O, 11:
a) en el sentido de proximidad de orden nulo;
b) en el sentido de proximidad de primer orden.
n
40. J {y (x)J = ) VI + y'Z dx en la función Yo (x) == O,
o
donde y (x) E CI [0, tt): O
a) en el sentido de proximidad de orden nulo;
b) en el sentido de proximidad de primer orden
J(

4 t. J fy (x)l = j (1 +2y'2) dx en la función Yo (x) c;:;,s 0,


o
§ s. FUNCIONAL. VARIACiÓN DE UNA FUNCIONAL 35

donde y (x) E C; Iü, rr], en el sentido de proximidad de


primer orden.
EJEMPLO 12. Demostrar que la funcional
1
J(y{x)]= Jo Vl+y dx
x3 2

definida en el conjunto de las funciones y (x) E e [0, 1] es continua


en la función yo(x) = x!- en el sentido de proximidad de orden nulo.
SOLUCION. Pongamos y (x) = r +
enl (x), donde T) (x) E C (0, 1)
Y a. es tan pequeño como se quiera, tenemos
1

J [11 (x)] = J [x2 + a:f) (x}J = J xli VI +(x2+ a:1l)2dx =


O
1

= Jo
x3Vl+x4+2a:x2r¡+a.2rt2dx•

Pasando al límite cuando el -+ O, obtenemos de esta igualdad


t
lím lJy(x)J=
a:.. O JO
r x3V 1+~dx=J[x2}

lo que equivale a la continuIdad de la funcional en la función


Yo (x) = xa•
DEPINICION. Sea M un espacio lineal normado formado por las
funciones !I (x).
La funcional L [y (x)) definida en el espacio M se denomina
lineal si satisface las condiciones
1) L {ey (x)) = el: (y (x)J,
donde e es una constante cualquiera y
2) L 1111 (x)J + v« (x)) = L [Yt (x)J + L (Y:a (x)l,
donde 111 (x) E M e 112 (x) E M.
Por ejemplo, la funcional
b
L [y (x)) = J
a
W+y)dx

definida en el espacio Cl la, b] es, obviamente, lineal.


éAP. TI. EXTREMO DE FUNCIONALES

Existe otra definición de funcional lineal:


La funcional L Iy (x) 1 se denomina lineal si 1) es continua y 2) satis-
face la condición
L [Yl ():) + Y'l; (x») = L (Yl (x)] + L [g, (x)]
cualquiera que sean 9, (x) E M e Y2 (x) E M.
42. Demostrar la equivalencia de las dos definiciones de
funcional lineal.
43. Demostrar que la funcional L [y (x)1 = y (xo) es lineal.
44. Sea L [(y (x)l una funcional lineal. Demostrar que si
la razon
' L11 y(x)
(y(x)J .
JI -+- O cuando 11y (x) 11-+ 0, es L ry (x) ] =
- O.
3°. Variación de una funcional. Sea J Ly (x)1 una funcional defini-
da en el conjunto M de funciones y (x). La magnitud
t::.J = s: (11 (x)) = J [y (x) +
ay] - J [y (x)]
(6y = y'(x) - y (x), donde Ij (x) E M e (x) E M) y
se denomina incremento de la funcional J {y (x)] correspondiente al
incremento 8y del argumento.
EJEMPLO 13. Hallar el incremento de la funcional
t
J [y (x}) "'" J y!!' dx
o
definida en el espacio el [a, bJ si 11 (x) =x e !h (x) = x'.
SOLUCIÚN. Tenemos
t t 1

M=J(x2}-J[x]= J x22XdX-) x.l'dX=) (2xS-x)dx=O.


o O O
45. Hallar el incremento de la funcional del ejemplo 13
siendo y (x) ex e Yl (x) = 1.
DEFINICIÚN. Si el incremento de la funcional J [y (x)]
llJ = J [y (x) + 6yl - J [y (x)]

se puede representar en la forma

Al = L [y (x), ay) + ~(y (x), ay) 11 8y 11,


donde L {y (x), ayJ es una funcional lineal res pedo a ay y ~ (y (x), 6y)-
- o cuando !l 6y 11 - O, entonces la parte del incremento lineal res-
pecto a ay, o sea, L [y (x), óyJ. se llama variación de la [uncional y se
FUNCIONAL. VARIACION DE UNA FUNCIONAL 37

representa por (jJ. Se dice en este caso que la funcional J [y (x)) es


dí [erenolabte en el punto y (x).
46. Demuéstrese que la variación &1 de la funcional
1fy (x)l se determina unívocamente (si es que existe).
EJEMPLO 14. Demuéstrese que la {uncional
b

J [y (x)] = J !I d:z
a

definida en el espacio e la, b] es diíerenclable en todo punto y (x)


de este espacio.
SOLUCION. Tenemos

6.J=J (y (,r)+6yJ - J Iy (xl) =


b b b
= J
a
(y+~y) d:r - r
a
ydx= J aV
o
ax•

b
Es decir, l1J = J ay áx,
Pero ésta es una funcional lineal respecto
a
a (¡y. Todo el incremento se ha reducido en nuestro caso a una funcional
lineal respecto a 6y. La funcional considerada es diferenciable en todo
b
punto y (x) y su variación es ó! = J 6y dx.
o
47. Demostrar que toda funcional 1 ty (x)] lineal continua
es siempre diíerenciable.
EJEMPI.O 15. Demostrar que la funcional
b
J tu (x)l = ) y2dx
a

definida en el espacio e [a, b] es dilerenciable en todo punto !I (x).


SOLUCiÓN. Tenemos
b b b b
6J= J
o
(y+ay)2dx-)
o
y2dx.= J
a
2yOydx+ ) {Oy)2dx.
a
(3)

En el últlmo miembro de (3) la primera integral representa la Iunclo-


nal lineal respecto a 611 cualquiera que sea-la funci6n fija y (x). Estl·
38 CAP. 11. EXTREMO DE FUNCIONALES

memos la segunda integral de este miembro. Tenemos


b b
) (óy)2 dx= J
I óy 12 dx e
a a
b
~(máx
Q~x~b
Jagl)2 r dX=(b-a)lIóIIIIZ=((b-a)1I6YIIJlffSYII.
Ja
Si 11 ay 11 - O, la magnitud
(b - a) 11 ay n -1- O.
Es decir, hemos logrado representar el incremento llJ de la funcional
como la suma de L 111(x), óy! y de una magnitud infinitésima de
segundo orden con respecto a 110,,11. Segun nuestra definición, la
funcional considerada es dilerenciable en el punto 11 (x) Ysu variación es
b

6J = 2 ) y oydx.
a
t
48. En la funcional J (y (x)J = J y'l.dx tomar y = 2x
o
y fJy = CtXZ; comparar /jJ y IlJ para Ct = 1; -0,1 Y 0,01.
1 .

49. En la funcional J fy (x)l = J x!ldx tomar y = ¿s: y


o
ay = comparar tlJ y 6J para Ct = 1; 0,1 Y 0,01.
CtX;
50. Analizar si son o no diferenciables las funcionales
siguientes:
1) J fu (x)] = y (a) en el espacio e la, bl.
2) J IU (x)J = y (a) en el espacio el fa, el,
3) J [y (x)] = VI + y'z (a) en el espacio el fa, bl.
4) J fy (x)l = I y (a) I en el espacio e la, bJ.
51. Demostrar que la funcional J'A [y (x)] es diferenciable
si lo es J fy (x)1. Hallar la variación de J2 [(y (x)L
52. Sea F (x, y) una función continua de sus argumentos
con derivadas parciales continuas hasta de segundo orden
x ~ b, -00 <y < + oo. Demos-
inclusive en el recinto a ::::;;;
trar que la funcional
b

J !y (x)] = ) F (x, y) dx
o
§ 3. FUNCIONAL VARIACION DE UNA FUNCIONAL 39

definida en el espacio e [a, ir] es diferenciable y que su varia-


ción es
b
<'J = f
~
iJF~, y) oydx.
y
a
EJEMPLO 16. Consideramos la funciona!
b

J fu (x)J = J F (x, y, y') dx


a

definida en el espacio el [a, b) de funciones y (x) que son continuas


en el segmento fa, bJ y (IUe tienen en él derivada continua de primer
orden. La función F (x, !I, y') es continua respecto a todos sus argu-
mentos y tiene derivadas parciales continuas hasta de segundo orden
inclusive en el recinto
a ~ x ~ b, -00 < y < +00, -00 < y' < +00.
Determinamos el incremento IlJ de la funcional correspondiente al
Incremento óy de) argumento siendo 6y E el la, bl, Tenemos
b
IlJ (y (x» = 5 (F(x, y+ay, y'+6y')-F(x, y, y')Jdx. (4)

Según la fórmula de Taylor


F(x. g+óy, [1' +6y')- F (x, y, y') =:

= ~: oy+ ~:' ay' -1- R (x, U. y', 6g. tly'), (5)

donde R [x, Y. y'. f5y, 6y') es el término complementario de la lórrnula


de Taylor. Introduciendo (5) en (4), obtenemos
b b

Al (y (x}1= j (~~
hy+ ::' oy') dx+ ) R (x, y, y', ~!I, hy') dx, (6)
a a
El primer sumando en el segundo miembro de (6) es lineal respecto a
6YIY 6y·. SuponR'amos que las segundas derivadas parciales de la
fundón FI.(x, y, y ) respecto a y e y' no pasan, en valor absoluto, de una
constante M > O en un recinto acotado respecto a y e !I'. Tendremos
entonces
b b

J
o
J R (x, y, JI', ~U, oy') I dx ~ 2M J IJ 6yll2
a
dx=2M (b-a) 11 óy 112,
40 CAP. 11, EXTREMO DE FUNCJONALES

donde 11 8y 11 = 1_!lá!, (1 6y l. \ 6y' \). Por conslguíente, el segundo


a..~x.~b
sumando del segundo miembro de (6) es una infinitéslma de segundo
orden respecto a n 6y 11. Es decir, en virlud de la definici6n, la funcio-
nal J [y (x)1 es dlíerericlable en el espacio el (a, b) y su variación es
b

~J = J (~~ ~y+~:' Oy') dx (7)

EJEMPLO 17. Hallar la variación de la funcional


1
J [y (x)] =J (y'ell+xyZ) dx,
-1
SOLUCION. La función F (x, y, y') = !leY xy' es, evidentemente, +
conlinua respecto a todas las derivables x, 11 e y' en conjunto y sus
derivadas pardales de cualquier orden respecto a y e y' son acotados
en cualquier recinto acotado de variación de y. e y'. Por esto, la funcio-
nal considerada es düerencíable en el 1-1, 1) y, según la f6rmula (7),
su variación es

8J = ) [(y' ell + 2xy) Oy + ell5y' Idx.
-1
53. En la funcional
,
J [y (x)J = j (y' y + xy''l.) dx
t
k(x-l)
tomar y = In x y 6y = e:=T ; comparar I1J y 6J para
k = 1; 0,1 y 0,01.
54. En la funcional
1
J [y (x)] = ) (XZy'2_!I) dx
o
tomar y = x'J. y 6y = kX3; comparar I1J y tU para k == 1;
0,1 y 0,01.
n
55. En la funcional J ry (x}1 = 5 s" sen x dx tomar y =
o
= sen x y fJy = k cos x; comparar /.}} y 6J para k = -1;
0,3' Y 0,03.
p. FUNCIONAL. VARIACION DE UNA FUNCIONAL 41

56. Las derivadas parciales de segundo orden de la fun-


ción F (x, Zl. Z2, ••. , zm+t) respecto a todos los argumentos
son continuas en el recinto a =::;;;; x =::;;;; b Y - 00 <lit < + 00
(k = 1, 2, ... , m + 1). Demostrar que la funcional
b
J {U (x)] = J
Q
F (x, y, lJ', •.. , y<"'J) dx

es diferenciable en el espacio Cm La, bl y que su variación es


b

6J = J~ [ :: 6y + :~ 6y' + ... + a:(~) o!lm)J dx.


4". Segunda definición de la varIación de una funcional. Se llama
variación de la [unctonai J [y (x)] en el punto y = y (x) el valor que
toma en a = O la derivada de la funcional J (U (x) + a~!ll (constdera-
da en tanto que función de a) respecto al parámetro a:
a
6J = OIZ J [y (x)+a6yll~_o.

Si existe la variación de la funcional en tanto que parte principal


lineal de su incremento (o sea, si existe la variación en el sentido de
la primera definición), también existe la variación en tanto que valor
en a = O de. la derivada respecto al parámetro a y ambas variaciones
coinciden.
EJEMPLO 18. Empleando la segunda definición. hallar la variación
de la funcional
b
J [y (.~)] = J JI"
a
dx,

SOLUCiÓN. La variación de esta luncional en el sentido de la prí-


mera definición es
b
6J = 2 J 116y dx
4

(véase el ejemplo 15). DetermInemos la variación de la funcional


J [y (x)] basándonos en la segunda delínición. Tenemos
b
Jlv(x)+a6y}= j (y+aBy)2dx.
o
CAP. JI. EXTREMO DE FUNCIONALES

Por eso,
b

iJ~ JIY(X)-!-MY)=2) (y+<xóy)ógdx


(1

de modo que
e
óJ = iJ~ J fu (x) + <x~u] Ia.... o = 2 J
(l
yóy dx,

Las variaciones de la funcional en el sentido de la primera y de


la segunda definiciones coinciden.
Para las funcionales que siguen hallar, en los espacios
correspondientes, sus variaciones en el sentido de la segunda
definición.
e
57. J lY (x)} = J (x+ y) dx.
a
b
58. J{y(x)J= J (yZ_y'2)dx.
a
1
59. J (y (x)] = y2 (O) + J (xy+ y'Z) dx.
O
11:

60. J [y (x)l = ) y' sen ydx.


o
b

6t. J(YII Uz, ... , Ynl= r


v
a
F(x, YI' UZ, ... , Y'l! y;, y~,
..• , U~)dx,
donde F es una función continua de sus argumentos y sus
derivadas parciales respecto a lodos los argumentos son
a
continuas en un recinto acotado de variación de los mismos.
OBSERVACION. La segunda deflnlctón de la variaci6n de una fun-
cional es en cierto sentido más amplia que la primera pues existen
funcionales que tienen variación en el sentido de la segunda definición
aun cuando no se pueda despejar la parte principal lineal en el incre-
mento de las mismas. Para explicarlo recurrlremos a las funciones;
§ 3. FUNCIONAL. VARIACIÓN DE UNA FUNCIONAL 43
en este caso nuestra afirmación equivale a que la exlstencla de las
derivadas en cualquier dirección no basta para la existencia de la dife-
rencial de la [unción.
Sea
xy
f (x, y)

donde p y q> son las coordenadas polares del punto (x, y). Las derivadas
pardales :~ y !~ existen en todo punto y son iguales a cero en el origen
de coordenadas; sin embargo. no existe la diferencial en el origen el,
de coordenadas. Efectivamente, supongamos que df existe. En este
caso el gradiente de la función f seria igual a cero en el origen de coor-
denadas y, por eso, la derivada df ~iO) en cualquier dirección también
seria igual a cero. Pero es fácil persuadirse de que
dI (O, O) l 2
di ="2sen q>
lo que, en general, es diferente de cero. Agul cp es el ángulo entre el
vector 1 y el ele Ox.
5°. Segunda variación de una funcional. Una funcional J [.r, y]
dependiente de los elementos x e!/ (que pertenecen ambos a un espacio
lineal) se denomina bilineal si es una funcional lineal en para x
fijo y una funcional lineal en x para y fijo. O sea. la funciona J [x, 111
r
es bilineal si
J (alX] + a2xS' U] = al) [Xl, y] + aaJ [X2' gl,
J (x. ~1l(1 + ~2!1!al = ¡ljJ [x. YtJ + ~2J [x, Y21.
Poniendo en la funcional bilineal y = x obtenemos la expresión
J {x, x] llamada funcional cuadrática.
Toda funcional bilineal definida en un espacio de dimensión
finita se denomina forma bllineal.
Una Iuncíonal cuadrática J Ix, x] se denomina definida positiva
si J [x. x) > O cualquiera que sea el elemento no nulo x,
Por ejemplo.
1) la expresión
b
¡Ix, !/]= Ja
A(t) x (/) g(t)dt,

donde A (/) es una funci6n continua fija. representa una funcional


b
bilineal en el espacio e la. b] mientras que la expresión 1
(l
A (l) Xi (t) dt

representa. en este mismo espacio, una funcional cuadrática que resulta


definida posítiva si A (t) >
O para todo t E (a, ti);
CAP. u. EXTREMO DE FUNCIONALES

2) Ji' expresión
b
J (x, xl = J (A (/) 1'2 (t) +B (1) x (1) x' (/) +C (t) x'2 (/)] dt
a
es un ejemplo de una funcional cuadrática definida para todas las
funciones pertenecientes al espacio CI [a, bJ;
3) la expresión
h b
J(X,Yl=l ) K(s,t)x(s)y(t)dsdt,
a a
donde K (s.t) es una función fija de dos variables, representa una run-
clona] bülneaí en CIa. b).
DEFINrerON. Sea J fy (x) ) una funcional definida en un espacio
lineal normado. Diremos que la funcional J [y (x)} tiene segunda
variación si su incremento 6J = J Iy (x) + 8y1 - J [y (x)) puede
ser representado en la forma
1
s: =L, [~g)+2 ~I~y)+~ 11 ~y 1(2,
donde LI (6y) es una funcional lineal, L2 [6y] es una funcional cuadrá-
tica y ~ -+ O cuando 1I 6y \1 -- O.
La funcional cuadrática L2 [ay) se denomina segunda variacU5n
(o segunda diferencial) de la luncíonaí J [y (x)] y se desIgna por 6"J.
La segunda variación de una funcional se determina unívocamente
(Sí es que existe).
EJEMPLO 19. Hallar la segunda variación de la funcional
1
J [y (x») =) (xy2+y'9) dx
O
definida en el espacIo Cl [O, 1) de las funciones y (x).
SOLUCIÓN. Tenemos

6J = J (y (x) +6$11- J [y (x)] -=-


1
= J [x (y+6y)'2+(y' +Oy')3_xy'2_y'3] dx=
O
1
= ) ¡2.xyOy+x (6y)2 + 3y'20y' +3$1' (6g')'2 + (6y')3Jdx=
o
t 1 t
= J
O
(2xy6y+3y'26y') dx + J
O
(x (6y)2+3y' (611')'21 dx+ ) (011')3dx,
O
(8)
FUNCIONAL. VARIACIÓN DE UNA FUNCIONAL 45
Fijemos y (x). El primer sumando del último miembro de (8) será
entonces una funcional lineal respecto a 6y; el segundo sumando de
este miembro sera una funcional cuadrática: finalmente, el último,
tercer, sumando de este miembro puede ser estimado as!
1 1 I

I ) (6y')Sdx I~ (rnáx 16y' 1)2 5 J 6y' I dx ~ ( Jo 16y' I dX) 116y 112


o O
e
(la norma se toma en el sentido del espacio t (0, 1)), o sea, podrá ser
representado en la íorma ~ 11 6y II~. donde ~ -+ O cuando 11 6y 11 -+ O.
Por definición, para nuestra funcional existe la segunda variación
fJ'J¡ igual a
I
62J =2 5 (x{liy)2+3y' (6y')2.) dx,
O

62. Hallar la segunda variación de una funcional cuadrá-


tica.
63. Hallar la segunda
variación de la funcional eF(IIJ
siendo F (y) una función dos veces díferenciable.
64. Demostrar que las funcionales de tipo
b
J [y(x)] = J F (x, y. !/) dx
(1

definidas en el espacio eJ [a, bJ son dos veces diferenciables


si la función integrando F tiene derivadas continuas hasta
de tercer orden inclusive. Hallar la fórmula para la segunda
variación.
Consideremos la función a> (a) = ¡ [y (x) +
aOy]. La segunda
variación 6't.J de la funcional J [y (x)] se define también mediante el
valor de la segunda derivada de la función ID (a) en el punto a = o:
a21D (al
da'J
I~-o'
Para las funcionales de tipo integral (que predominarán en nuestras
consideraciones) ambas deriniciones coinciden.

Hallar la segunda variación.

65. J [y (x)J = r
b

el
F (x, y, ti, ... , y(ml)dx.
CAP. 11. I!XTRP.MO DE PUNCIONALES

66. J (z (x. !J) 1 = Jf


G
P (x. !J, z, zx, z/I) dx dy.

11

67. J [!JI' !J2, .•. , !Jn] = ) F (x, !Jt, !J~, ... ,


a
... , y~) dx,
6°. Extremo de una funcional. Condición necesarla de extremo.
DIremos que la funcional J (y (x)] alcanza su mázlmo en la curva
y = Yo (x) si los valores que toma la funcional J [y (x)1 en cualesquiera
curvas próximas a y = Yo(x) no son mayores que J (Yo(x)1, o sea, si
ó.J = J [y (x)) - J (Yo(x)} ~ O.
Si ó.J ~ O y tJ.J = O sólo para y (x) = Yo (z), diremos que se
alcanza máximo estricto en la curva y = Yo (x).
Análogamente se define la curva y = 110 (x) en la que se alcanza
un minimo, En este caso se tiene ó.J >
O para todas las curvas próxl-
mas a la curva IJ = Yo (x),
EJEMPLO 20. Demostrar que la funcional
I
J (y (x)l =) (x2+ (2) dx
O
alcanza rnlnímo estricto en la curva y (x) s O.
SOLUCION. Cualquiera que sea la función IJ (x}. continua en [O, 1],
tenemos
I l'
M = J (y (x)] - J [O] = ) (x2 + (2) dx - ) X2 dx = J y2 dx #- O;
o O O

además, el signo de Igualdad se da 5610 para y (x) == O.


EXTREMOS PUE1ITE y DeBIL. Oiremos que la funcional J (y (x)J
alcanza su mdxlmo relativo fuerte en la curva y = Yo (x) si
J Iy (x)] :s;;;; J (!le (x))
para todas las curvas admisibles y = y (x) pertenecientes a una s-vecln-
dad de orden nulo de la curva y = Yo (x). Análogamente se define el
mínimo relativo fuerte de una funcional.
Diremos que la funcional J [y (x)l alcanza su máximo relativo débil
en la curva Y = Yo (x) si
J [y (x») ~ J [Yo (x)1
para todas las curvas admisibles y = y (x) pertenecientes a una s-ve·
cindad de primer orden de Ia curva y = Yo (x). Análogamente se define
el mínimO relativo débil de una /lmcional.
FUNCIONAL. VARIACIÓN DE llNA FUNC.IONAl 47
Los máx imos y mínimos (fuertes 'i déblles) de la funcional
J (y (x)l se denominan extremos relativos.
Todo extremo fuerte es al mismo tiempo extremo débil pero no
viceversa.
El extremo de la funcional J (y (x)l referente a la totalidad de las
funciones en las que está definida 1a funcional se denomina extremo
absoluto.
Todo extremo absoluto es al mismo tiempo extremo relativo fuerte
y débil pero no todo extremo relativo será extremo absoluto.
EJEMPLO 21. Consideremos la funcional
1'1

J (JI (x) 1= ) y2 (1 - y'2) dx


O
en el espacio de funciones JI (x) E el [O. n) que satisfacen la condición
JI (O) = JI (n) = O. En el segmento (O. n) del eje Ox hay mínimo débil
de J. En efecto, tenemos J = O si y = O; por otra parte, para las curvas
pertenecientes a una e-vecindad de primer orden .M este segmento,
donde e es cualquier número positivo menor que 1, se tiene I y' I < l
de modo que el integrando es positivo para JI ::;6: O y, por consiguiente.
la funcional se anula sólo si y = O. Es decir, la íunclonal alcanza
mínimo débil en la curva y = O.
Mínimo fuerte no hay. Basta tomar
I
y (.1:) = Vii sen nI
En este caso
n
J Iy (x») = ~ J sen2nx (1 -n cos2 nx) dx=
o
=1.n J r
sen2nxdx-..!....
4 J
r
sen22nxdx,.,,~-~
2n 8'
O O
O sea, J < O en estas curvas si n es suficientemente grande. Por otra
parte, siendo tl suficientemente grande, todas estas curvas se encuen-
tran en una vecindad tan pequeña como se quiera de orden nulo de la
curva JI = O. Por consiguiente, no se alcanza mínimo fuerte en y = O.
EJEMPLO 22. (Weierstrass). Consideremos la funcional
1
J[y{x»)= J x'2y'2dx; y(-I)=-I. y!I)=1.
-1

En el segmento [-1, 1) tenemos J Iy (x)] ~ O y, además, J [11 (x») = O


sólo si y' (x) = 0, o sea, si y (x) = e = const. La función 11 (x) = e
pertenece a la clase el [-1,1] de las íunclcnes que tienen primera deri-
vada continua en ('1segmento (-1, lJ, pero 110 satisface las condiciones
de frontera dadas. Por consiguiente, J [y (x)] O para todas las>
(unciones y (x) E el
[-l. 1] que satisfacen las condiciones y (-1) ::i::; -1
48 CAP. JI. EXTREMO DI! PUNCIONALES

e 11 (1) = 1. En otras palabras, la funcional tiene cota inferior pero


ésta no se alcanza en las curvas y (x) E el 1-1, 11. Efectivamente,
consideremos la familia monoparamétrica de curvas
x
arcfg-
~
Ya. (x)= J' ~> O.
arctg-
n
Todas ellas satisfacen las condiciones de frontera: Ya. (-1) = -1 e
Ya. (1) = 1. Pasando al límite para a -- O. obtenemos la lunclén
-1, si -1 ~;c:<O,
y(x) = O, si x=O,
{
-+ 1 ,si O <x 1. <
es decir, y
(x) = sg x (Hg. 3). Esta función pertenece a la clase de
funciones díferenclables a trozos en el segmento [-l. 1).
y

y ""Yo(x)

FIg. 3
Tenemos
t
J (Ya. (x)J = J
-t (a2+x2) arctg2-
(=
a
I
2a r 2a 1 (I-a arctg ~ ) .
i
arctg2 -a o
J arctg2-
~
§3. PUNCIONAL. VARIACIÓN DE UNA FUNCIONAl. 49

Queda claro que J {y~ (x)J -+ O cuando ce. -+ O. En la función límite


y(x), que satisface las condiciones de frontera y(-l) = -1 ey (1) ::.:"
= 1, el valor de la funcional J [y (x)l es igual á cero: J (x» = O. íY
Por consiguiente, la funcional J fy (x)) alcanza su valor mínimo
en la curva y (x) = sg x que pertenece. a la clase de funciones diíeren-
dables a trozos en el segmento [-1, 11 pero no pertenece a la clase
el [-1, 1].
Teo~EMA (condición necesaria de ex tremo de la lunclonal}. Si la
funcional diferenciable J [y (x)l alcanza su valor extremo en la curva
U = Yo (x), donde Yo (x) es un punto interior del campo de deft/~ición
de la funcional, entonces en y = Yo (x) se tiene
f>J [Yo (x)] = O. (9)
Las funciones para las cuales 5J = O se denominarán funciones
estacionarias.
Hallar las ecuaciones funcionales para las funciones esta-
cionarias de las funcionales que siguen, empleando la condi-
ción necesaria de extremo (9) y los lemas fundamentales del
Cálculo variacional.
b b
68. J [q> (s») = J J K (8,
(l a
t) q> (s) rp (t) ds dt +
b b

+ J rp2(s)ds-2 J q¡(s)f(s)ds,
Q a
donde K (s, t) es una función continua simétrica de s y t en
a -c s <.
el recinto D { a ~ t _~ b
b} .! (s) es una función continua
en fa, b] y cp (5) es el argumento funcional continuo incógnito.
+""
69. J [q¡ (x)] = J
-00
[p (x) rp'2 (x) + 2rp (x + 1) X
x <p (x-l) - rp2 (x) - 2cp(x) f (x)] dx,
donde el argumento funcional rp (x); es continuo y tiene deri-
vadas continuas a trozos en todo el intervalo - 00 < x <
< + 00, p (x) tiene derivada continua y f (x) es continua.
70. J [q¡ (x)J = r [p (x) q>'2 (x) +q(x) qJ2(x)--2(jl (x) f(x)}dx,

4-01387
50 CAP. 11. EXT~fMO DEiPUNCIONALES

donde p (x) tiene derivada continua, q (x) y f (x) son conti-


nuas y el argumento funcional <:p (x) tiene dos derivadas
continuas.

§ 4 Problema elemental del Cálculo variacional.


Ecuac.ión de EuJer
Supongamos que la función F (x, y. 11') tiene derivadas parciales
continuas hasta de segundo orden inclusive respecto a todos sus argu-
mentes.
El problema elemental del Cálculo variacional es el siguiente:
entre todas las funciones y (x) que tienen derivada continua y que
satisfacen las condiciones de frontera
y (a) = A e y (b) = B (1)
hallar la [unción que ofrece extremo débil a la íuncíonal
b
J 111(x)1 = j F (x, Y. y/) dx, (2)

En otras palabras. el problema elemental del Cálculo var iacíonal


consiste en hallar el extremo débil de la funcional de tipo (2) en el
conjunto de todas las curvas suaves que unen dos puntos fiJOS PI (a, A)
y p.. (b. B).
TEOREMA l. Condición necesaria 1) para que la funcional (2), defi·
nida en el coniunto de todas las funciones 11= {I (x) que tienen derivada
continua IJ que satisfacen las condiciones de frontera (1), alcance su valor
extremo en la functón y (x) es que esta función verifique la ecuación
de Euler
d
F,,--F
Y dx 11
1=0. (3)

Las curvas integrales de la ecuación de Euler se denominan exire-


males (o curvas de Lagrnnge).
En forma desarrollada la ecuación de Euler da
V~ (x) FIj'II' -1- y' (x) Fyy' +
Fxy'- Fu = O (FY'll' ,*0)
y representa una ecuación diferencial de segundo orden de modo que
su solución "general comprenderá dos constantes arbitrarias cuyos
valores se determinan. hablando en términos generales, de las con di-
ciones de frontera (1)_
La funcional (2) puede alcanzar extremo sólo en las cxtrcrnales que
satisfacen las condiciones (1).
1) Esta condición es necesaria para el extremo débil. Como quiera
que todo extremo fuerte PoS al mismo tiempo un extremo débtl, cual-
quier condición necesaria para el extremo débil también será necesaria
para el extremo fuerte.
§ 4. PROBLEMA ELEMENTAL 51

El problema de contorno

no siempre tiene solución y si la solución existe, puede no ser única.


EJEMPLO I./.En qué curvas puede alcanzar Su extremo la funcional
2
J[y(x))= J (y'2-2xy)dx; y(I)=O, y{2) -'.-11
I
SOLUCIONo Aquí tenemos F (x, y, y') = y''}. - 2xy de modo que
la ecuación de Euler da y' + x = O. Su solución general es
xl
y= -6 +C1x+C:¿.
Utilizando las condiciones de frontera, obt enernos para CI y C2 el
seguiente sistema de ecuaciones lineales:

Ct+C2= ~,}
2C1+Cz ="6'
De aquí resulta C,=! y C2=O. Por consiguíente , el extremo puede
alcanzarse sólo en la curva
x
Y='6{I-.x:2).

EJEMPLO 2. Hallar las extremales de la funcional


3

J [U (x)) =j (3x - y) .1/ ctx


1

que satisfagan las condiciones de frontera y (1) = 1 e U (3) = 4- ~.

SOLUCIÓN. La ecuación de Euler es 3x - 2y=O, de donde y = ; X.

La extremal IJ = ~x no satisface la condición y (1) = I y, por eso,


nuestro problemavariacional no tiene solución.
EJEMPLO 3. Hallar las extrernales de la funcional
21"(
Jly(x)]= ~ (U/~_y2)dx
O
que satisfagan las condiciones de frontera y (O) = 1 e !J (2n) = l.
4*
52 CAP. 11. EXTREMO DE FUNCIONALES

SOLUCION. La ecuación de Euler tiene 13 forma 1/' + g = O; su


solución general es
y = el cos x e9 sen x, +
Utilizando las condiciones de frontera, encon tramos
y = ces x + e sen x,
donde e es una constante arbitraria. Es decir, el problema variacional
considerado tiene un conjunto tnlinlto de soluciones.
Hallar las extrernales de las funcionales siguientes.

o
71. J(y(x)1=) (12xy-y'2)dx; y(-1)=I, y(O)=O.
-1
2

72. Jly(x)1=) (y'2 + 2yy' +y2) áx; y(I)= 1, y(2)=0.


1
t
73. J [y (x)1 = J y y (1 + y'Z) dx;
o
y (O) = y (1) = ;2.
1
74. J[y(x)l= ~ yy'2dx; y(O)= 1, y(I)=:;4.
o
n
75. Jly(x)l= J (4ycoSX+y"2_y2)dx¡
o
y (O) = 0, y (n) = O.
1
76. J{y(x)= J (y'2_y2_y)ezrdxj y(O)=O, y(I)=e-t•
O
1
77. J[y(x)l= J (y'2-2xy)dx; y(-I)= -1, y(I)=1.
-1
O
78. J Iy (x)] = j (y'2_ 2xy) dx; y { -1) = O, Y (O) = 2.
-1
e
r
79. J [y (x)} = J (xy'2+ yy') dx; y(l) = O, y (e) = l.
I
pnOHLEMA ELEMENThL 53

La ecuación de Euler (3) para la funcional (2) es una ecuación


dllerenclal de segundo orden y, por eso. la solución y = y (x) de 13
ecuación de Euler debe tener segunda derivada y" (x). Sin embargo.
se dan casos en que 1a lunctón que ofrece el extremo a la funcional
b
J {y (x)} =
a
I F (x. y, y') dx no tiene segunda derivada.

EJEMPLO.j. La funcional
I
J Iy Ix)) = J y2 (1- y')2 dx
-1
con las condiciones de frontera
e y (1) = 1
y (-1) = O
alcanza su valor mínimo. igual a cero, en la función
( ) _ {O
si x ~
v x - x si x>O.
O,
Aun cuando la función v (x) no tiene segunda derivada. sattsíace la
ecuación de Eulcr correspondiente.
Efectivamente, tenemos F (x, y. y') = y? (1 _ y')2 y, poniendo
y = v (x), obtenemos la ecuación de Euler

2V(I-v')2+! (2v2(I-v')]=0. (4)

Pero, según la definición de la función v (x), en (-1, 1) tenemos


F , = _2vll (1 - VI) == O y, por consiguiente, también dd F, = O;
11 X "
o sea, a pesar de que la ecuación de Euler (4) es formalmente de segundo
orden y a pesar de que u· (x) no existe, la ecuación de Euler se convierte
en identidad al sustituir en ella v (x).
TEOREMA 2. Sea y = y (x) soluci6n de la ecuación (Ú Euler
d
F y - "'(fX F 11' = O.
Si la función F (x, y, y') tiene derivadas parciales continuas hasta de
segundo orden inclusive, entonces la función y = y (x) ttene segunda
derivada continua en todos los puntos (x, y) para los cuales
FY'JlI [x, y (x), y' (x)] *0.
COROLARIO. La extrema! y = y (x) puede tener puntos angulares
s6/0 en aquellos puntos en los que Fy,y, = O.
Asi, en el ejemplo 4 tenemos que F",y' = 2y2 se anula en los
puntos del eje Ox; la extremal tiene punto. angular en )t = O.
TEOREMA 3. (Bernstein). Supongamos que en la ecuación
y. =f (x, y, y')
54 C,\P. 11. ex'rREMO DE FUNCIONALES

las funciones f. 11/ y f ' son continuas en lodo punto finito (x, y) para
cualquier valor finito de !J' !J supongamos que existe una constante k > O
y unas funciones
~ = a (x, y) ~ O y ~ = ~ (x. y) ~ O.
acotadas en cualquier porción [inita del plano, tales que
tv (x, y, g') > k Y I f (x, y, y') 1 ~ a.y'2 (6) + ~.
Entonces, por dos cualesquiera puntos del plano (a, A) y (b, B) de abscisas
distintas (a =1= b) pasa una curva integral y = 'l' (x) de la ecuación (6),
y sólo una.
EJEMPl.O 5. Demostrar que por dos cualesquiera puntos del plano
de abscisas distintas pasa una extremal única de la funcional

J{y(x\]"""
.
\ c-211%(y'2_1)<I,1:

SOLUCiÓN. La ecuación de Euler para la funcional


.

considerada es
yn = 2y (1 -1- y',
de modo que se puede aplicar el teorema 3. En efecto, tenemos en
este caso
f(x, y, y') = 2y 11 + y'Z) y fy = 2 (1 + y(2) ~ 2 = k.
Además,
I f (x, I = I 2y (1
y, !J') + y''}.) I~ 2Iy I y'2 + 2 I y 1,
o sea, a. = ~ = 2 I y I > O.
EJEMPLO6. Demostrar que no hay extrema! de la funcional
J Iy (x)j = J (y2+ VI + y'2) dx
que pase por dos cualesquiera puntos del plano de abscisas distintas.
SOLUCION. La ecuación de Euler tiene la forma
3
yn=2b'{1+.!I'2)2 (7)
y el teorema 3 no se puede aplicar ya que no se cumple la segunda de
las condiciones (6) (debido a que f (x, y, y') crece, respecto a y', más
rápido que la segunda potencia de y'). Las condiciones del teorema 3
son de carácter suficiente. O sea, si estas condiciones no se cumplen. de
ello no se puede deducir que no hay extrema) que pase por dos puntos
cualesquiera de abscisas diferentes. Demostremos que por los puntos
A (O, O) Y B (~ , 2) no pasa ninguna extrema! de la funcional con-
siderada.
Tomando en la ecuación (7)
n dp
y'=p e y =p d!l '
§ 4. PROBLEMA ELEtv\ENTAL 55

obtenemos
dp -
.~
P -=2y (1 +p)Z 2. o sea, pdp 3 ~2fJdy.
dy
(1 +p2.)2
Integrando, encontramos
I
-~-==!¡2-C, o sea,
lfi"+P"2
(C-y2) VI +y'2= I
de modo que

donde e es una constante real. Separando las variables en la última


ecuación e integrando del punto A al punto B, obtenemos
2
I
-=
ri C-y2
dl/. (8)
2 .
o
VI _(C_yZ)2
Cualquiera que sea el número real C. el denominador del integrando de
(8) será complejo en cierto intervalo (a, ~) c: (0, 2) de variación de
la variable y. Por consiguiente, la igualdad (8) es imposible. Esto quiere
decir que no se puede trazar extrernal alguna por los puntos A (O. O)
YB(~, 2).
80. Demostrar que por dos cualesquiera puntos del plano
pasa una y sólo una extremal de la funcional
J [[1 (x)l = J 1/1+!f + y'2 dx,
EJEMPLO 7. Demostrar que toda ecuación
y{t (x} = f (x, y, y') (9)
es ecuación de Euler para cierta funcional
r
J fu (x)}= J
F (x, y, y') dx,

1) ¿Cómo se determina la función F (x, Y. y') a partir de la función


f (x, y. y')?
2) Hallar todas las funcionales cuyas extrernales son las rectas
y = C1x C2•+
SOLUCION. Busquemos la funcional cuya ecuación de Euler
fy-E"'x-f y,,,y' -f"'I/'Y" =0
56 CAP. 11. EXTREMO DE FUNCIONALES

coincida con la ecuación (9). Es decir, debe cumplirse la identidad en


x, y e y'
Fy-F"'x-FY'IIY' -F¡¡,v'! (x, y, y');; O.
Derivando respecto a {J', obtenemos

+
F1I'II'x+ Fy'Y'YY' FlI'lI'lI,l+FII'lI,fll,=O.
Tomando u = FY'II" obtenemos para la función u una ecuación en
derivadas parciales:
au +'
r)x y
(Ju
iJy
ou
+f iJy' +f 1/' u=
O (10)

Por consi~uiente, la búsqueda de la funcional, o sea, de la función


F (x, y. y ), se reduce a la integración de la ecuación lineal en deriva-
das parciales (10) y a la cuadratura sucesiva.
Consideremos la segunda cuestión. En este caso la ecuación de
Euler debe ser y. = O y para la función u se obtiene, de acuerdo con
(10), la ecuación
j::_+
ax
,2!:..-o .
y ay - (11)
Integremos esta ecuación.
La ecuación de las caraderisticas tiene la forma
dx dg dy'
-1-=7=--0

Integrando este sistema, encontramos


y' = Cl e Y = clx + C2•
de donde Cs = y - xy'. Por eso, la solución general de la ecuación
(1t) es
u (x, y. y') = (l) (g'. y - xy').
donde ID es una función arbitraria diferenciable de sus argumentos.
De aquí
z
F(x, y, z)=a(x, Y)+z~(x, y)+ 1
o
(z-/)I1>(t, y-tx)dt,

donde a (x, y) y ~ (x, y) son funciones arbitrarias de sus argumentos


que cumplen la relación
oa o~
ay = a;: .
Se puede ver de la sol ución que existe una cantidad infinita de proble-
mas variacionales que tienen la ecuación (9) como ecuación de Euler.
§4. PROBLEMA ELEMENTAL

CASOS ELE.'>\ENT AI.ES DE I NTEGRACION DE J.A ECUACiÓN DE EULER.

1°, F no depende de y', o sea, F = F (x, y).


En este caso la ecuación de Euler tiene la forma
Fy (x, y) = O. (12)
La solución de esta ecuación finita (no dllerencial) no contiene ele-
mentos arbitrarios y, por eso, no satisface, hablando en términos gene-
rales. las condiciones de frontera y (a) = A e y (b) = B,
Sólo en casos excepcionales, cuando la curva (12) pase por los
puntos de frontera (a, A) y (h, B), existirá una curva en la que podrá
alcanzarse el ex tremo.
EJEMPLO 8. Hallar las extrernales de la funcional

2"'
J (U (x)! = S y (2x - U) rlx; y (O) = 0, y ( ; ) = ~.
o
SOLUCION. La ecuación de Euler tiene la forma 2x-2y = 0, o sea.
y
,SI
= X. Puesto que las condiciones de frontera se satisfacen, la integral
'2
j Y (2x - y) dx puede alcanzar su extremo en la recta y = x. Para
o
otras condiciones de frontera, por ejemplo. y (O) = Oe y (~)= 1,

la extrema! y = x no pasará por los puntos frontera (0, O) Y ( ~ , 1)


de modo que el problema variacional con estas condiciones de frontera
no tendrá solución,
20, F depende- de y' en forma lineal. o sea.
F (x, y, y') = M (x, y) N (x, y)!J'. +
En este caso la ecuación de Euler tiene la forma
oM eN
Ty--ax=o.
Igual que en el caso 1°, la ecuación obtenida es fillib:l y no diferencial.
En general, la curva determinada por la ecuación O..M - élaN = O
uy x
no satisface las condiciones de frontera y, por consiguiente, el
problema variacional no tiene, como regla, solución en la clase de
funciones con tinuas. Por otra parte, si en un recinto D del
p 1ano x Oy se ulene Ty
oM - a; iJN = O . I a cxpreston
"F (x, y. y ') =
"'" M (x. y) dl( + N (x, y) dy es una diferencial total exacta y la
58 CAP. 11. EXTREMO DE FUNCIONALES

funcional
b (b. B)
J [y (x)] = ) F (x, y, g') dx= J (M dx+N dy)
a (a, A)

no depende del camino seguido en la integración: el valor de la Iuncio-


nal J ry (x)] es el mismo en todas las curvas admisibles. El problema
varlacíonal carece de sen ti do.
EJEMPLO9. Anallzar el extremo de la funcional
b
J(y(x»)= ) (y2+2xyy')dx; y(a)=A, y (b)=B.
a
SOLUCiÓN. Aquí F depende de U' en forma lineal. Tenemos
aA1 aN oM _ aN = O
éJy =2y, iJx. =2y y ay ax - ,

o sea, el integrando (yz + 2xyy') dx es una diferencial exacta. Por


consiguiente. la integral no depende del camino seguido en la íntegra-
ción:
b (ti, B¡

J [U (x) 1 = J (y? dx + 2xy dy) = Jr d (xy2) = Xy21X~b


x=-a
= bB2 - aA2
a (a, Al

para cualquier curva de integración y (x) que pase por los puntos (a, A)
y (b, B). El problema variacional carece de sentido.
30• F depende sólo de y', o sea, F = F (y').
La ecuación de Euler tiene la forma
Fy,y·Y· = Q.
En este caso las extrernales son todas las rectas posibles
y = C.x +
C~,
donde Cl y e, son constantes arbilrarias.
EJEMPLO 10 Hallar las extremales de la funcional
b
J Iy (x)} = J VI +y':l
ds, y (a) = A, y (b) =8.
a
Esta funcional determina la longitud de la curva que une los plintos
(a, A) y (o, 8). Desde el punto de vista geométrico. el problema con-
siste en hallar la curva de longitud mínima que une dos puntos fijos.
SOLUCIÓN. La ecuación de Euler tiene la forma y" = O.Su solución
general es
PROBLEMA eLEM~NTAL 59

La extrernal que satisface las condiciones de f rentera y (a) = A e


IJ (b) = Bes, obviamente, la recta que pasa por los puntos (a, A)
y (b, B):

!I=
B-A
b (x-a)+A.
-a
4°. F no depende de y. o SClI, F"= F (x. y').
En este caso la ecuación de F.uler os ;¿ Fy' (x. y') :.... O. de donde
resulta
(I3)

siendo Cl una constante arbltrar!a.


La ecuación (13) es una ecuación dílerencial de pri mer orden.
Integrándola, encontramos las extrernales del problema.
EJEMPLO 11. Entre las curvas que unen los puntos A (1, 3) Y
B (2,5) hallar la curva 1.'11 la que puede alcanz ar su extremo la Iunciona l

2
J Iy (x)l='~ ) y' (1 + x~y') dx,
1

SOLUCl<)N. Puesto que F no depende (.1\: y, la ecuación de Euler


tiene Ia forma :'(, Fy. (x, y') .~.O, o sea. ,~x (1.\ 2xZy') = O de donde

1 +2x2y' =C,.

En t onces, y ,C
=1-1 2x2 de 010
do que q
Y=T" 'C 2,
donce
d *-
C1-
l-e
2
l
. Por consiguiente, las ex tremales representan una familia
de hipérbolas. Determinemos la extremal que pasa por los puntos
fijos, Para hallar los valores de las constantes Cf y C2 lormamos el
sistema

de donde resu Ita Cr = - 4 Y e2. = 7. La ex trema 1 buscada es y =


=7-~.
x
5°. F no depende explícitamente de x, o sea, F = F (y, y').
En este caso Ia ecuación de. Euler tiene Ia forma
"» -Fyy'Y' _FII'y,yn =0.
60 CAP. 11. EXTREMO DE FUNCIONAl.ES

Multiplicando ambos miembros de esta ecuación por y', obtendremos


en el primer miembro la derivada exacta :,. (P-y'FII,), o sea, la

ecuación será :1' (F - y' Fy') = 0, de donde


(14)

siendo el una constante arbi traria. Esta ecuación puede ser integrada
resolviéndola respecto a g' y separando las variables o introduciendo un
parámetro.
EJEMPLO 12 (cuerpo de resistencia mínima en un fluido). Deter-
minar la forma del cuerpo sólido que, al moverse en un fluido de gas,

del fluido

x
do gas

Fig. 4

encuentra resistencia mínima. Para simplificar consideraremos el


cuerpo de revoluci6n (Hg. 4).
SOLUCION. Suponiendo que la densidad del gas es suficientemente
pequeña y que las moléculas. al chocar con la su perficie del cuerpo, se
reflejan de forma especular, obtenemos para la componente normal de
la presión
p = 2ptill sen2 a,

donde p es la densidad del gas, t! es la velocidad del gas respecto al


cuerpo y f) es el ángulo entre la velocidad y su componente tangencial.
La presión es perpendicular a la superficie de modo que la componente
según el eje O,. de la fuerza que actúa sobre un anillo de anchura
(1 + ytll) IJ2 dx y de radio y (x) se puede representar en la forma
1

dF=2pt/}'sen2 e [2ny (1 + y'2}2¡ sen O dx,


§ 4 PROBLEMA ELEMENTAL 61

La fuerza resultante que actúa en la dirección positiva del eje Ox


es igual a
/ 1
F = f 4npo2 seoS 6.11 (1 + .11'2)2" dx.
o
Supongamos, para simplificar el problema, que

Entonces, la fuerza de resistencia será igual a


I
F= 4n:pU2 J
O
y'3y dx. (15)

El problema consiste en hallar la función y (x) en la que F alcanza


su valor menor posible siendo
.ti (O) = O e .ti (/) = R,. (16)
La ecuación de Euler para la funcional (15) tiene la forma
.11'3-3 :z (.11.11'2)=0. (17)

La solución particular .ti = O de esta ecuación debe ser rechazada en


virtud de las condiciones de frontera (16). La ecuación (17) puede ser
representada así:
v" +
3yy'y· = O. (18)
MuHiplicando p,or y' ambos miembros de (18), vemos que el primer
miembro es (y 3.11)'. 1ntegrando, encontramos
y'SU = q,
de donde resulta

y' C1
=-r=-
yy
e y=(C.x+C2)·. +
Utilizando las condiciones de frontera (16), obtenemos
4
R,3
Ct=-¡- y Ca=O
de modo que
62 CAP. 11. EXTREMO DE FUNCIONALES

o sea, el contorno Con ex iremos fijos que corresponde a la resistencia


mínima del cuerpo es una parábola de grado
4
1..
EJEMPLO 13. Hallar la extremal de la funcional
b,¡_
J lb' (x)J:.- J
r V 1 +y'2.
!I dx
el

que pasa por dos puntos fijos (o, A) y (b. B) pertenecientes al semiplano
su penar.
SOLUCION. Puesto que la función integrando no contiene explíci-
tamente x, la ecuación de Euler, según (14), da

ViT?2
11

Después de simplificar,
1
encontramos y VI + y'~ = C\. donde el =
= Cl. Integrando la última ecuación, encontramos (x + Ci)t. + !I' =
= Gr. o sea, una familia de circunferencias con centros en el eje Ox.
La extremal pedida será la que pase por los puntos fijos. El problema
tiene so lución única ya que por dos puntos cualesquiera del semi pi ano
superior pasa una y sólo una semicircunferencia con centro en el eje Ox.
OBSERvACiÓN. Según el principio de Fermat, el camino que recorre
un rayo de luz al propagarse con la velocidad v (x. y) en un medio bidi-
mensional no homogéneo constituye una extremal de la funcional

X'V~
J Iy (x)) = S ()(x, y) dx,
Xo
Como hemos visto en el ejemplo anterior, si la velocidad de la luz (1
es proporcional a y, los rayos de luz representan arcos de circunferencias
con centros en el e.je. Ox.
Sea q una curva. Denominaremos longitud óptica de la curva q
el tiempo T (q) que se precisa para recorrerla al moverse según esta
curva con la velocidad de la luz ti (x, y).
Supongamos que el semi plano superior !I > O es un medio óptico
en el que. la velocidad de la luz en todo punto es igual a la ordenada del
mismo: ti = y. Como hemos visto, los rayos de luz en este medio serán
semicircunferencias con centros en el eje Ox, Se puede demostrar que,
si uno de los extremos del arco AD de la semicircunferencia q se halla
en el eje Ox, su longitud óptica es infinita (rig. 5). Por eso, diremos
que los puntos del eje Ox están en el infinito. Consideremos que las
semlcírcúnferencias con centros en el eje O» son recias, que las longitu-
des ópticas de los arcos de estas semicircunferencias son las longitudes
de dichas rectas y que los ángulos entre las tangentes a las semicir-
cuníerencias ell el punto de intersección de las mismas son los áng~los
§ 4. PROBLEMA ELEMENTAL 63

entre dichas rectas. Obtenemos una Geometría plana en la que se


conservan muchas proposiciones de la Geometría habitual. Por ejem-
plo, por dos puntos fijos se puede trazar una recta y sólo una (ya que

o K

Fig.5

por dos puntos del semíplano se puede trazar sólo una semicircunfe-
rencia con centro en el eje Ox)_ Dos rectas se consideran paralelas si
tienen un punto infinito común (o sea, si las dos semicircunferencias

o x

Fig. 6

son tangen tes en un punlo B perteneciente al eje Ox). En tortees, por


todo punto A que no se halle en la recia q se pueden trazar dos rectas ql
y q2 paralelas a q. Las rectas que pasan por el punto A v que se encuen-
tran en los ángulos vertlcates 1 y IIl, cortan la recta q ínientras que las
rectas que se encuentran en los ángulos II y IV no la cortan.
Hemos obtenido el modelo de Poincaré de la Geometría plana de
Lobachevski (Iig. 6).
64 CAP. 11. eXTREMO DE FUNCIONALES

Hallar las extrema les de las funcionales:


b
SI. J[y(x)J= S l2xy+ (x2 + eV)y'Jdx; y (a) = A.
a
y(b)=8.
t
82. J(y(x)1= J
O
(eY+xy')dx; y(O)=O, y(l)=~.

83. J [y (x)j = l'tJ (y'2_ y2) dx; Y (O) = 1, y


(T). ="""2'
112
O

l(y'2-y2)dx;
1\

84. J[y(xH= y(O)=l, y{n)=-l.


O
1
85. J[y (x)l= J (x+y'2)dx¡ g(O)=I, y(1)=2.
O
1
86. J[y (x)J= J (y2+y'2)dx; y(O)=O, g(1)= 1.
O
i
87. J [y (x)l == S (y'2 +4y2) dx; y (O) = e2. y (1) = 1-
O
1
88. J[y(x)]= J O
(2ell-y2)dx; y(O)=l, y(l)=e.

b
89. J [y (x)] = J (xy' + y'2) dx,
(t

90. J [y(x)] = J" (Y+ y; ) dx,


a
91. Demostrar que no tiene extremos la funcional lineal
b
J[y(x»)= J [p{x)y'+q(x)y+r(x)]dx,
el

donde p (x) E el la. bl, q (x) E e [a, b) y r (x) E e [a, bJ.


PROBLEMA ELEMENTAL 65

92. Consideremos la funcional


b

J [y (x)1 = J F(x, Y, y')dx


a

con las condiciones de frontera y (a) = A e y (b) = B. Demos-


trar que la ecuación de Euler subsiste al agregar al integrando
F (x, y, y/) dx la diferencial total de cualquier función u =
= u (x, y).
b

93. J fy (x)l = J (y2+ s" +2yeX)dx,


a
11/2
94. J (y(x)} = J (y2_ y'z-8ych x) dx;
1)

y (O) = 2, Y ( ~ ) = 2 eh ;.
95. Hallar las extremales de la funcional
b
J (y (x)l = ) xny,Z dx
el

y probar que para n ~ 1 no existen extrernales que pasen


por dos puntos situados a distintos lados del eje Oy.
PROBLEMAS VARIACIONALBS EN FORMA PARAMETRICA. En muchos
problemas es cómodo, y a veces imprescindible, emplear la representa-
ción para métrica de las lineas
x= <p (1), }
to~ t ~ IJ
Y=\Il (r),
donde <p (t) Y IV (/) son funciones continuas con derivadas continuas
a trozos siendo, además, (~~) 2 + ( ~~) 2 ;f= O.
Consideremos la funcional
ti

Jc= Je FU, x, YI;' y)dt=


~
J F(t, x, !J, .;, y)dt, (19)

• dx dy
donde x =-¡¡¡ e y= di'
~-01367
CAP. ii. Ex'tk.eMO bE I1tJNCIONALES

Para que los valores de la funcional (19) dependan sólo de la


línea, y no de su parametrlzacíón que puede efectuarse de distintos
modos, es necesario y suficiente que la función integrando no contenga
explícitamente el parámetro t y sea positivamente homogénea de grado
uno respecto a Jos argumentos X e y:
F (x, y, k~, ky) = kF (x, y, x, y), k> O.
Por ejemplo, en la funcional

Je = Je x dy -!I dx

la función Integrando es positivamente homogénea de primer grado.


Efeclivamente, tenemos
F(x, Y. ;, y)=xfi-Y~
y es obvio que
F(x, Y. k~. kit) =kF{x, y, X. y).
Si la curva e

ofrece el extremo a la funcional J e en la clase de lineas e que unen los


puntos fijos (xo. Yo) y (Xli Yl). las funciones tp (t) Y 'i> (t) satisfacen
las ecuaciones de Euler

F,- : (Fil=O'} (20)


FV-(j[(F.) =0.
11

Una de las ecuaciones (20) es consecuencia de la otra.


Las ecuaciones de Euler se pueden representar en la forma de
Weiersfrass
F,-F,
XII 1/X
-=
r :1 '
(21)
FI (;2+y2)'2

donde r es el radio de curvatura de la extremal y FI es el valor común


de las razones
P,. F .. F.,
f 4. PROBLÉMA ÉLEMENtAL

EJEMPLO14. Hallar las extrernales de la funcional


(Xl. 11'1)

Je = J 1/2y'Z ds,
(O, O)

SOLUCIONo Puesto que puede haber extremales que se cortan con las
rectas paralelas al eje Oyen más de un punto, consideraremos el pro-
blema en forma paramétrica.
Poniendo x == % (t) e !I = Y (t), encontramos que la función
ya •
integrando tiene la forma y' ';""x, o sea, es positivamente homogénea

de primer grado respecto a JC e y.


. .
x2

La primera de las ecuaciones (20) da

_dtd ( l/2-y%)

=0 ,
X2

de donde

yl ( :!) 2 = q.
Integrando la última ecuación, encontramos
y' = 2Ct% + Ca·
Puesto que la extrema! debe pasar por el origen de coordenadas,
tenemos Ca = O. La segunda condición de frontera da Cl = i;"•
o sea, en definitiva,

EJEMPl.O 15. Hallar las extremales de la íuncional


ti
[e= J [y X2+y2 +aa (Xy-yx)J di.
to

SOLUCIÓN. Poniendo

F(x, !J. ;. y)= Y;D+V2+a'J.(xY-l/;),


vernos que la función F es positivamente homogénea de primer grado
respecto a ~ e y.
Empleemos las ecuaciones de Euler en la forma de

68 CAP. 11. eXTREMO DE PUNCIONALE~

Weierstrass. Tenemos
F ••
%%
F .::0: F I =---;----
-02 = ------,,3-
y
l/x
y2 (~Z+g2)2
Por eso. la ecuación (21) tiene en nuestro caso la forma
1
-=202•
r
Es decir. la curvatura..!.. de la extremal es constante. Por lo tanto. las
r
extremares son arcos de circunferencias; en particular. se tienen cir-
cunferencias completas sí
x(to)=x(t.), }
y (lo) =y (ti).
Hallar las extremales de las funcionales:

96. Je = J
(XI. 1/1) •
yZ_yzx'l.

dt.
te. O) x

97. J e = J
(1, 2) •
y2_3e1J1Xx2
• '.
di.
(o. O) x
(1. O)
98. J c= ) (K V~2+y2 -~y) di,
(-1. O)
donde K> O es una constante.

§ 5 Generalizaciones del problema elemental


del Cálculo varlaclonal
1°. funcionales que dependen de derlvedes de órdenes superiores.
Supongamos que. se tiene la runclonal
"'1

J [y (x)l= S F {x, y (x), g' (x) •.. " yln) (x)) dx, (1)
q
donde F es una función diferenciable n +
2 veces respecto a todos los
argumentos e y (x) E en [%0. XtJ. y supongamos que las condiciones de
írontera tienen la forma
=
!I~(Xo) Yo. !I (%0)=!ló' ... , y( 1\-1) (xo) = n- 1), } Yb
(2)
y (xl) =!/it !I (Xl) = 111" •• " yen-t) (xl) = y\,.-o,
GENERALIZ "ClONES OEL PROBLEMA ELEMENTAl. 69

Las extremales de la funcional (1) con las condiciones (2) son las
curvas integrales de la ecuación de Euler - Poissaa
d d2 dn
Fv--¡¡; Fv'+ dx2 F¡¡w- ••• +(_I)n dxn F¡¡(n)=0.
EJEMPLO 1. Hallar la extrema! de la funcional
I
J fy (x»- J (360x2y-y"Z) dx;
O
!I (O) = O. y' (O) = 1, Y (1) = O, y' (1) = 2,5.
SOLUCIÓN. La ecuación de Euler - Poisson tiene la forma
tP
360X2+ dxz (-2y")=O, o sea, ;v = 180x2,
y su solución general es
t
/1='2 x6-l' CjX3+CZXZ+C3X+ C,.
Empleando las condIciones de lrontera, encontramos
3
Ca="2' C2=·--3. CJ=1 y C,.--O.
La extrernal pedida es
l 3
Y=2 x6+2 x3-3x2 +x.
Consideremos el caso cuando en la frontera no se dan todas las
condiciones (2) sino un número de las mismas de modo que. después
de emplear las condiciones de frontera, en la solución general de la
ecuación de Euler - Poisson contienen todavía constantes arbitrarias.
Para resolver este problema es preciso hallar la variación de la funcio-
nal (1). transformarla tomando en consideración las condiciones de
frontera dadas y obtener condiciones complementarias en la frontera
igualando la variación a cero.
EJEMPLO 2. Hallar la curva y = y (x) que ofrece valor extremal a
la funcional
b
J (U (x)J = -} J y''I. dx (3)
a
con las condiciones
y (a) = O e y (b) ,= O. (4)
SOLUCIÓN. La ecuación de Euler - Poisson tiene la forma.
plV = o.
70 CAP. 11. EXTREMO DE FUNCIONALES

Su solución general
ti = CI + CZX + Cax2 + C4x3 (5)
contiene cuatro constan tes arbitrarias C¡ (i = 1. 2. 3 y 4) Y las con-
diciones de frontera (4) no bastan para determinarlas. Por eso, como
hemos explicado, calculamos la variación de la funcional (3). Tenemos
b
6J = J
Q
yW{jyW ax. (6)

Integrando (6) por partes dos veces, obtenemos


b
BJ = g" (x) {jy' (x) 1:- J U"'6U' dx =
~
b
= y" (x) 6y' (Xli: - y" (x) By (Xli: + ) yIV6y dx, (7)
a

La expresión (7) debe anularse en la extrernal ti (x) de la funcional


(3). Debido a la arbitrariedad de la función 6y, resulta que glV = O;
ésta es la ecuación de Euler - Poisson para la funcionar (3). Pero si
la integral del útimo miembro de (7) se anula, la expresión. de frontera

lb'" (x) {jy' (xl - !I'" (x} 6y (X)ll:


también debe ser igual a cero idénticamente. Puesto que 6y (a) =
= Óy (ó) = O (extremos fijos), resulta que debe ser

y" (b) 6y' (b) - y. (a) 6y' (a) = O.


En virtud de la arbitrariedad de las magnitudes ay' (ó) y fJy' (a),
obtenemos necesariamente
y" (a) = O e y" (b) = O. (8)
Las condiciones (8) conjuntamente con las condiciones (4) determinan
unívocamente la extremal en la familia (5): g ::¡ O.
2°. Funcionales que dependen de m funciones. En el caso de una
funcional que depende de In funciones Y1 (x), 1:Ia(x), ••.• !1m (x)
Xl'"

J (Ui> Y2 •... , Yml = J F (x, y" JI'/-, ••• , !I'th ElÍ. y;, .... U~) ax

'1 con las condiciones de frontera de tipo


Yh (xo) = y~, IJk (xt) = vi (k = 1, 2! ... , m)
§ s. GENERALIZACIONES DEL PROBLEMA ELEMENTAL 71

las extremales se determinan del siguiente sistema de ecuaciones dife-


renciales de segundo orden
d
F - -F ,=0 (k= 1, 2, ... ,m) 1.91
l/k dx Vil

que se denomina sistema de ecuaciones de Euler.


EJEMPLO 3. Hallar las extremales de la Iuncional
2
J[y(x), z(x»)= j (y'2.+Z2+Z'2)dx
t
con las condiciones de frontera
y (1) = 1, y (2) = 2, z (1) = O, z (2) = 1.
SOLUCION'. En este caso el sistema de ecuaciones (9) tiene la forma
y'=O, }
z-z"=O.
Resolviendo este sistema. encontramos
y = Clx C2 y+ Z = Cse" C~e-:c. +
En virtud de las condiciones de frontera. tenemos
1 e2.
Ct=l, C2,=O, e3=e2=I y e"=-e2=I
de modo que la extrema) pedida
y=x, }
sh (x-I)
z= sh 1
es una curva alabeada que constituye la intersección de dos superficies
ci 1índricas.
EJEMPLO ". Hallar las extrernales de la Iuncíonal

J fy (x), z (x)J = 1
n

O
(2yz - 2y2 + y'2 _Z'2) dx

si
y (O) = O, 1, z (O) = O
!I (n) = y z (n) = -1.
SOLUC10N. El sistema
de ecuaciones (9) tiene la Iorrna
y' +2y-z=O, }
z'+g=O, .
de donde, eliminando la función z, obtenemos
ylV 2y· y = O. + +
La solución general de esta ecuación tiene la forma
fI = el tOS1: el sen x +X (Ca tOS" +
C" sen z}, +
72 CAP. u. E XTREMO DE FUNCIONA LES

En virtud de las condiciones de frontera 11 (O) = Oe 11(n) = 1, tenemos


1
Cl = O Y C3 = - - de modo que
1t

y=C2 sen x+C,x sen x_!' cosx.


n
La función z se determina de la condición 2 = 1/" + 2y. Tenemos
I
z= Cl! ~nx+C~(2cos x+x sen x)+- (2 sen x=» cos x).
n
Las constantes Coa y C4 se determinan de las condiciones de frontera
2 (O) = O y z (n) = -1, de donde, resulta que C4 = O Y que Cl! es
arbitrarlo. Entonces.
I
z=C2 sen x+- (2 sen x-x cos x).
1t
La familia de extremales es
y = C2sen x- _x_cos x, ~

+
11:

Z = C2 sen x+ (2 sen x-x cos x), J


donde Cz es una constante arbitrarla.
SO. Funcionales que dependen de funciones de varias variables
independientes. Consideremos la funcional
J [z(x, 1/))= j ~F (X t 1/, 2, :~, ~~) dxay, (10)
D
donde F es una función diíerenciable tres veces respecto a sus argumen-
tos, y supongamos que se pide hallar Ia función z = 2 (x, 1/) que sea
continua conjuntamente con sus derivadas hasta de segundo orden
inclusive en el recinto D, que tome valores fijos en la frontera r del
recinto D y que realice el extremo de la Iuncicnal (10).
Si el extremo de la funcional (10) se alcanza en la superficie
z = z (x, y), la función z = z (x, y) satisface la ecuación de Euler -
Oslrogradski
o
F:z.~ aX. {Fp}-
iJ
ay {Fq} =0, (11)
fj
donde OX {Fp} y
a
ay {Fq} son las derivadas parciales completas
respecto a x e y, respectivamente:
a az op oq
-ax{Fp} = Fpx+ FpzTx + Fpp ax +Fpq OX '
() OZ op aq
ag-{Fq} = Fqy+Fq'Z. ay +Fqp Ty+Fqq ay ~
aquí se ha lomado, para abreviar,
az
Pi" =p y
iJz
aV =q,.
GENE~ALIZACIONES DEL P~OBLEMA ELEMENTAL 73

La ecuación (11) representa la condición necesaria de extremo de


la funcional (10). Es una ecuación en derivadas parciales de segundo
orden; se busca su soluci6n z = z (x. y) que toma valores fijos en la
frontera r.
EJEMPLO 6. Escribir: la ecuación de EuJer - Ostrogradskí para
la Iuncíonal

11%(x, y)] = J J [( ~: )2_( ~~)2]dXdY.


D

SOLUCION. Tenemos F(x, Y. z, p, q)_p?_qZ y, según (II), encon-


tramos -
a (2p)- oya
ax (-2q) =0, o sea,

02z fJ2z
OX2 - iJy? = O.
Para la funcional

J[2'(X., "'a, .... xn») = JI··· J


D
F(Xh Xa•••• , "'n. Z, Pi,

iJz
donde Pk=;¡--- (k= 1, 2, ... , 11). la condición necesaria de extremo
VXIl
viene dada por la siguiente ecuaciónde Euler- Ostrogradski
n

Fz- ~ iJ~t {Fpi}=O,


¡...
1
0, en forma desarrollada,
n 10 iJ
FT.- ~
,... 1
(FXíPi+F:ZPiPt +~
1....
1
FpíPj a:: ) =0. (12)

La función z = Z (Xl. X, •••• , xn). solución de esta ecuación, debe


satisfacer en la frontera r del recinto n-dimensional D las condiciones
de frontera dadas.
EJEMPLO 6. Hallar las condiciones que debe cumplir la función
Z (Xl. X" ••• , xn) para Que la integral de Dirlchlet

n
D [l (Xt, Xa, •.. , xn)! =)j ...J ~ ( ::, ) 2 dXl dx2 ... dxn
Q i=1

alcance en ella su mínimo si dicha función toma valores c:le~erminados


en la Irontera r
del recinto Q.
74 CAP. 11. EXTREMO DE FUNCIONALES

SOLUCiÓN. En este caso F= ~


{...j
( ::i.) 2, o sea, F no depende

explícltamente de Xt. X2. • •• , Xn I z. Por lo tanto,


Fz=Fr-Pi =FxiP¡=O,
F .. ={ 2 si 1 = 1,
Ptp) O si i =1= i
y, aplicando la f6rmula (12), obtenemos
n
,... a2z
.4J ox! =0 ó !.1z=O
;=-1 )
(ecuación n-dimensional de Laplace),
OBSERVACION. Si bajo el signo de la integral figuran las derivadas
de la función z (x, y) hasta de orden n, la ecuación de Euler - Ostro-
gradski tiene la forma
a a ¡)'l.. a'J.
Fz- ox {Fzx}- iJy {FzlI}+ ax'J. {Fz:lCx}+2 oxoy {Fr-XII}+
0
+ ay2
EJEMPLO7. Escribir
funcional
2
{FZIIII}-'" +(-1)1'\

la ecuación de Euler -
an
--
011' {FzlIlI ... II}=O.
n
Ostrogradski
(13)

para la

J [z (x, y)] = J ) [( g;~) + ( :~ r+


D
2

+2 ( 02z
ox ay
)2 - 2zf (x, y)
] dx dy.
SOLUCIÓN. Tenemos

F=
iJ2z ) 2 + ((j2Z)
( oxll ay2
2
+2
((jZZ)
éJx ay
2
-22' (x, y).

Aplicando la fórmula (13). encontramos

-2t (x, y)
a2
+ ox2 ( a2z) + ay2
2 ox2
iJz ( i)2Z)
2 ay2 +
02 ( 2 oxfJ2Z)ay
+2 ox oy =0,
1) sea,
p. GENERALIZACIONES DEL PROBLEMA ELEMENTAL 15
La última ecuación se representa brevemente así:
11M. = f (x, y).
Hallar las extremales de las funcionales siguientes:
J

99. J [U (x}) = 5 (u +2y'2 +JI2) dx;


2
o
U (O) = 0, !I (1) = o, y' (O) = 1 e y' (1) = - sh 1.
o
100. J[y(x)l= J (240Y-UI'I2)dx;
-1
y(-l)= 1, y(O}=O, u' (- 1) = - 4,5,
y(O)=O, y"(-1)=16, y" (O) = O.
b
10t. J [U (x)] - } (U +y") dx;
a
y (a) = Yo, y(b) =Yit Y (a) = v: y' (b)=y;.
b
102. J [U (x)} = J (y2 + Uy") dx;
cJ

U(a) =A,. U' (a) =Az. y(b) = B.. y' (b) = Ba.
t
103. J {g (x)) = J (y'2+ y"2) dx;
o
g(O)=O, U(l)=shl, y'(O)=l, y'(l)=chl.
104. Hallar la extrernal de la fundonal
1
J (U (x)} = -} } y"2 dx
o
con las condiciones
y (O) = O, y' (O) = O. y' (1) = 1.
'11/"
105. J[y(x), z(x)) = J (2z-4y2+y'2_Z'2)dx;
o
V (O)= O, !I ( ~ ) = 1~ z (O)= 0, z ( ~ ) = l.
76 CAP 11. EXTREMO DE FUNCIONALES

106. J[y{x), z(x)]= J


-1
1
(2XY_Y'Z+ Z;3 ) dx;

y(I)=O, y(-I)=2, z(I)=I, z(-I)= -1.

107. J[y(x), z(X)J= 1


o
2(y'2+Z'2_2yz)dx;

y (O) = 0, 9( ~) = 1, z (O) = 0, z ( ; ) = 1.
t
t08. J[y(x), z(x)]= J (y'2+Z'2+2y)dx¡
o
g(O)=l, y(I)=2'
a 2(0)=0, z(I)=1.
109. Probar que la ecuación de EuJer de la funcional
b
J[y(x), z(x)] = I
4
F(x, y, z, g', z')dx

tiene las siguientes primeras integrales:


-JF
1) ay' = e si F no comprende !Jj
2) F - y ,oFoy' - zJ oz
iJF
= c' SI F no compren de x.
Escribir la ecuación de Euler=-Ostrogradskl para las
funcionales:
110. J [z(x, y)] = J ~ [( ~:)4-1-
r+
D

+( ~; 12zf(x, y)JdxdY,

It 1. J [t (x, g)] = S J ( ~:~ + ~:~r dx d!J.


D
t12, J [z (XI> X2' "', xn)J =
n

=)J ". J r~
D j....
1
aJ (X" XZ •••• , Xn) ( :;J V-
.-C(X" X2 •...• xn)z2+2zf(XI' X2, .. '. xn)jdxtdxa, .. dxn•
§ ti. íNVARiÁNC¡A ce LA ECUACION DE EULER 11

113. Deducir la ecuación diferencial de las superficies


de área mínima.
114. Hallar la extrema! de la funcional
1 1
J [z (x, y)] = Io 1o etll sen 211 dx dy

con las condiciones z (x, O) = O y 2 (X, 1) = 1.

§ 6. Invariancia de la ecuación de Euler


Si la funcional
b
J [y (x)1 = J F (x, y, g') as
a

se transforma efectuando una sustitución de la variable independiente


o una sustitución simultánea de la función incógnita y de la variable
independiente, las extremales continúan determinándose de la ecua-
ción de Euler que se obtiene a partir del Integrando transformado. En
esto consiste la lnvariancia de la ecuación de Euler.
Sea X = x (u, v) e y = y (u, e) con la particularidad de que

I Xu
l/u
Xl)
Yo
I =F o.
Entonces
J
J F (x, y, y') dx= J F [ x (u, e), y (u, [/),
!lu+Yllv~
Xu
+x"vu' X

X (xu +XDt1~) du = J (JI (u, (J, v~) du

y las extrernales de la funcional inicial se determinan de la ecuación


de Euler para la funcional J
<D (u, (J, v~) du:
d
«>1)--
du
<Il ,=0.
11

EJEMPLO 1. Hallar las extremales de la lunctonal


"'1
J [r(<p)]= J y,2.+r'2d<p.

"'o
18 cAP. 11. ÉXT~ÉMO bE pUNcioNAlES

SOLUCION. La ecuación de Euler para esta funcional es


r d r
o.
V,2+r'2 -~ y,2+r'Z
La sustitución de variables x=rcosep e !I=r sen q> da

y lleva a la funcional
b
J fJl (x)] = J -v +
4
1 11'2 dx

cuya ecuación de Euler es !lit = O de modo que


11 = C1x + Cs·
Por consiguiente, las extremales de la funcional inicial vlenen dadas
por la ecuación
r sen <p = CI,'cos c:P + C.,
donde CI y C, son constantes arbitrarias.
EJEMPLO 2. Hallar las extremales de la funcional
ID 2
J [y (x)} = Jo (e-~1I'2-e"yZ) dx,

SOLUCION. La ecuación de Euler'para la funcional considerada tiene


la forma
yW _ y' + et'XlI = O.
Realicemos la sustitución de variables
x= lnu, }
!I =o.
La funcional inicial se transforma entonces en
2 2
J do (u)J= J
1
(e-In Uu2{/2_e1n"v2) ':: = J
1
(v'2_ v2)du

Y su ecuación de Euler v· + ti = O se íntegra fácilmente:


ti = el cos u + C, sen u.
Volviendo a las coordenadas iniciales JC e 11. obtenemos la ecuación
de las extremales en la forma
JI = el cos eO! + e~sen eX.
§ s, íNVÁiÜANCIA DE LA ECUACIÓN DE. eUtER

115. Ha llar las extremales de la funcional


q:t(

J (r(<p)) = J r sen qi Vr2+r,2,d<p.


11'0
116. Probar que las extremales de la funcional
epi
J [r (q»1 = 1f (r sen ep) V ,.a +,'Zdq>
'Po
se determinan por cuadraturas.
117. Hallar las extremales de la funcional
b
J(y(x)l= J Vx2+y2Vl+y':ldx.
a
Igual que en el caso de una variable, la ecuación de Euler -
Ostrogradski es invariante respecto a las transformaciones de coorde-
nadas.
EJBMPLO 3. Escribir la ecuación de Laplace
iJ2z 02Z .
. ox:& ay2, =0 + (1)
en coordenadas polares.
SOLUCIONo Consideremos la funcional

D(z(x, y)] = 1~
G
(z~+z~)dxdy.

La ecuación de Euler - Ostrogradski para esta funcional es precisa-


mente la ecuación (1). Pasemos en la funcional de las coordenadas
cartesianas (x, y) a las coordenadas polares (p, Ip): .t = P cos Ip, !I =
= p sen IJ). Tenemos
op op alp sen Ip o<p cos IJ)
ax=COSCJI. Olj=senlp, Tx=--p-' ay=-p-
Y. por eso,

D (z (p, CJI)}= J J [(
G
zp :~ + zq¡ ~~ ) 2 +
+(zp ~~ +zcp :: )2]PdPd<P=J
G
J (pz~++Z~)dPd<p.
Formando la ecuación de Euler - Ostrogradsk l para esta última
Integral, obtendremos la ecuación de Laplace en coordenadas polares:
1
Zcpq> p. +
pZpp zp =O. +
CAP, n. ~XTREMó DE I'IvNClúNALRS

§ 7. Campo de extrema les


La familia de curvas y = y (x, e) forma un campo propio en el
recinto D del plano xOy si por cada punto (x, y) de este recinto pasa
una y sólo una curva de la familia y = y (x, e).
El coeficiente angular p (x, y) de la tangente a la curva de la
familia y = 11 (x, e) que pasa por el punto (x, y) se denomina inclinación
del campo en el punto (x, y).
La familia de curvas 11 = 11 (x, e) forma un campo central en el
recinto D del plano xOy si estas curvas cubren sin cruzarse lodo el

Fig. 7J

recinto D y arrancan de un mismo punto (xu, Yo) que no pertenece al


recinto D. El punto (xo, Yo) se llama centro del haz de curvas.
EJEMPLO l. Dentro del circulo x~ +
y2 ~ 1 la familia de curvas
11 = ce=, donde e es una constante arbitraria y, en particular, e = O,
forma un campo propio ya que estas curvas no se cortan en ningún
punto y por todo punto (x, y) del CÍrculo pasa una y sólo una curva
de esta familia (lig. 7). La Inclinación del campo en un punto cual-
quiera (ot, y) es igual a
p (x, y) = Ce= = y.
EJEMPLO 2. La familia de parábolas y = (x
propio dentro del círculo X2 + y2 ~
+
C)' no forma campo
I porque distintas curvas de la
familia se cortan dentro del circulo y no cubren todo el recinto (Hg. 8).
EJEMPLO 3. La familia de curvas y = Cx forma un campo central
en el recinto x > o.
§ 7. CAMPO DB EXTREMALES 81

Fig. 8

¿forman campo (propio o central) en los recintos indi-


cados las siguientes familias de curvas?
t18. y=Ctgx;
119. y=C cos x:
Ji
a) Ixl<~; b) 2<x~n; e)
x2 y2
120. y=(X-C)3j -¡-+9::;;;;1.
121. y=C(x2-2x);
a) O::;;;;x<lj b) -1::;;;;x~3;

122. Y = e sen (x - : ) ;
7t 1t n n 2
a) 4~x~2; b) 3~x~n; e) 8~x~ 1t.

123. Y = ex+c; X2 + y2~ 1.


Si el campo (propio o central) está Iorrnadc por una familia de
ex tremales de cierto problema var lacional , se denomina campo de
extrema les.
6-01387
82 CAP. 11. EXTREMO DE FUNCIONALES

EJEMPLO f. Consideremos la funcional


i
J fu (x)) = J 11'2 dx,
o
Sus extremales son las rectas y = C1x +
Cl' La familia de extremales
y = Ca forma un campo propio y la Iamtlla de extremales y = C1x
forma un campo central con centro en el origen de coordenadas.
124. Determinar para la funcional
a
J [y (x) 1 = Jo (y'2 + ya) dx
los campos de extrernales propio y central.
125. Lo mismo para la funcional
n/"
J Jy(x)J = j (y'2-y2+x2+4) dx.
O
Supongamos que la curva y =y (x) es la extremal de la funcional
XI

J fy (x») = J
F (x, y. y') ox
xo
que pasa por los puntos A (xo. Yo) Y 8 (x~, !ll)'
Se dice que la extrernal y = y (x) está incluida en un campo propio
(Ú extrema/es si existe una familia de extremales y = y (x. C) que forma
un campo y que comprende la extremal !I = y (x) para cierto valor
C = Ca Ysi, además, esta extremal y = IJ (x) no pertenece a la frontera
del recinto D en el que la familia y = y (x, C) forma campo.
Si existe un haz de extremales, con centro en el punto (xo. Yo),
que forma un campo en una vecindad de la extremal y = y (x) que
pasa por dicho punto. se dice que se ha encontrado un campo central
que incluye la extremal considerada y = y (x), Como parámetro de la
familia y = y (x, C) se toma el coeficiente angular de la tangente a
las curvas del haz en el punto (xo. Yo).
EJEMPLO 5. Consideremos el problema variacional elemental para
la funcional
2
J (y (x)] = JO
(y'3+sen2 x) dx.

a) Sea Ij (O) :..:: 1 e y (2) = 1. La familia de ex trernaies de nuestra


funcional viene dada por la ecuación Y = C,x +
C2• La extrernal que
satisface las condiciones de frontera es y = 1. Dicha extremal se
puede incluir en el campo propio de extremales y = Cs. donde es
es una constan te arb j traria.
tAMPO bE EXTREMALES
• i.

b) Sea y (Ó) = O e y (2) = 4. La extremal qúé responde a estas


condiciones de hontera es la recta IJ = 2x que puede ser incluida en
el campo central de extremales UI =: C¡x (Cl es una constante arbitraria)
con centro en el punto O (O, O).
y

I
¡

J(

Fig,9

EJEMPLO 6. Consideremos el problema variacional elemental


1

J (1/ (x)) = J !J' (2X- -} !J' ) dx;


-1
I
y(-l)=O, Y (1)="2'

La solución de la ecuación de Euler t íene la Iorma y=x7.+Ctx+


+Cz' La extremal de este problema + ~-! se puede
y=x2 incluir

en el campo propio de extremalcs U=x2+ ~+Cz (rig. 9).


84 CAP. II. EXTREMO DE FUNCIÓNALES

Probar que las extrernales de los siguientes problemas


variacionales elementales se pueden incluir en un campo de
extrernales (propio o central).
I
126. J(y(x)]= J (y'2-2xy)dx¡ y(O)=y(l)=O.
o
1
127. J[y{x)l= 1o (2eXy+y'2)dXj y(O)=l, y(l)=e.

a
128. J[y(x)] = J (y2-y'2)dx (a:;z!:kn);
O
y(O)=O, y(a)=O
2.
129. JIy(x)]= J (y'2+x2)dx i y (O) = 1, y(2)=3.
O

DEFINICI()N. Sea «I> (x, 11, C)=0 una familia de curvas planas. Se
llama Cvdtscrtmtnante de esta familia el lugar geométrico de Jos puntos
determinado por el sistema de ecuaciones
<D (x, y, C) =0, }
o<D (x, y, C) _ O (1)
ac -.
En el caso general, el C-discrimlnante comprende la envolvente de la
familia, el lugar geométrico de los puntos múl tiples y el lugar geomé
trico de los puntos de retroceso.
La envolvente de la familia cIl (x, y, C) = O es la curva que en
cada uno de sus puntos es tangente a cierta curva de la familia consi-
derada y tal que cada una de sus partes es tangente a un conjunto íníí-
nito de curvas de la Iarnil ia.
Si se tiene un haz de curvas con centro en el punto A (xo, Yo), el
centro del haz pertenece al e-discriminante.
EJEMPLO 7 Hallar el C-djscriminante de la familia de curvas
y = (x - C)2.
SOLUCIÓN. Las ecuaciones (1) tienen en este caso 1a forma

y- (X-C)2=O, }
2(x-C)=O,
de donde y = O Es fácil ver que la línea y = O es la envolvente de
esta familia. Efectivamente, en cada uno de sus puntos x = Xo la
línea y = O tiene tangente común con la curva correspondiente y =
p. CAMPO DE EXTREMALES 85

= (x - XO)2 de la familia. Además. si tomamos una parte de la línea


y = O, por pequeña que sea, habrá un conjunto infinito de curvas de
la Iamilta tangentes a esta parte. En el caso considerado el e-discrimi-
nante consta de la envolvente nada más.

En los problemas siguientes hallar los C-discriminantes


de las familias dadas.
lS0. y = Cx + O.
131. y (C - x) - C2 = O.
132. (x - C)2 + y2 = 1.
Si el arco AB de la curva y = y (x) tiene un punto común A",
distinto del punto A, con el e·discriminante del haz y = y (x, e) que
tiene su centro en el punto A y que. comprende la curva considerada, se
dice que el punto A * es conjugado del punto A.
EJEMPLO 8. Consideremos la familia monoparamétrtca de curvas
y = e sen x. El e-discriminante de esta familia se determina por las
ecuaciones
y-Csenx=O, }
-!¡en x=O,
o sea, representa un conjunto discreto de puntos (1m. O), k = O, ±l.
±2, ... (que son los puntos de intersección de la sinusoide y de] eje
y

2Jr J(

rlg. 10

Ox). Tomando, por ejemplo, e = 2, obtenemos la curva y = 2 sen x


que pertenece al haz de sinusoides con centro en el punto O (0, O).
Si el otro extremo B (Hg. 10) del arco de la curva y = 2 sen x tiene
la abscisa x E (n, 2n), el arco DB tendrá otro punto (a parte del punto
O (O, O» perteneciente al e-discriminante, a saber el punto 0* (n, O),
que será conjugado del punto O (O, O). Si es O
no habrá puntos conjugados del punto O (O, O).
<x < ni en el arco 08
I
86 CAP. n, EXTREMO DE FUNCIONALES

133. Se tiene la familia de curvas y = e (x - 1) x.


Hallar el punto conjugado del punto O (O,O).
134. Se tiene la familia de curvas y = e sh x, Hallar
el punto conjugado del punto O (O, O).
1°. Condl<:ión suficiente de Jacobl para poder IncluJr la extremal
en un campo central de extremales, Condición suficiente para que el arco
A B de una extremal pueda ser incluido en un camfo central de extremales
con centro en el punto A (xo, Yo), es que Id punto A conjugado del punto A
no pertenezca al arco A B.
EJEMPLO 9. Consideremos la funcional
a
J (g (x)) =) (g'2_9y2+eXl_l) dx;
O
ti (O) = 0, ti (a) = O.
Analizar la posibilidad de incluir la extremal y = O en un campo
central de extremales con centro en el punto O (0, O).
SOLUCiÓN. La ecuación de Euler para la funcional considerada tiene
la forma y. + 9y = O y su solución general esg = Cl sen 3x Ca cos 3x. +
Si a =1= k; , donde k es un número entero, la extremal que satis-
face las condiciones de frontera es la recta g = O. Consideremos la
íamílía monoparamétrica de extremales y = Cl sen 3x; es fácil ver que el
C-dlscrlminante de esta familia consta de los puntos donde (k; • O), k
es un número entero; por eso, si a < i-,
en la extremal !I = O no
habrá punto conjugado del punto O (O, O) y entonces esta extremal se
podrá, obviamente. incluir en un campo central de extremales con
.n;
centro en eJ punto O (O, O). En cambio, si 11 ~ '3' en la extremal
y = O habrá como mínimo un punto conjugado del punto O (O. O) Y
no se cumplirá la condición suficiente de J acobi; en este caso las extre-
mates y = Cl sen 3x no forman campo.
FORMA ANALrTICA DE LA CONDICiÓN DE JACOBI. Consideremos el
problema variaclonal elemental
:le I

J(y(x)]= J F(x, y, y')dx; y (xo)=Vo, Y (X¡)=-=Yt-


:ce
Si la solución u=u (x) de la ecuación de Jacobí

(FIIII-* FIITI') u- :x (FJI'II'u/)=O (2)

que satisface la condición u (xo) = O se anula también en algún otro


punto del intervalo Xo < x < Xl' el punto A· conjugado del punto
A (xo. Yo) pertenece al arco AB de la extremal (el punto B tiene las
coordenadas (Xlo 91»'
§7. CAMPO DE EXTREMALBS 87

Si existe una solucién. u (x) de la ecuación de Jacobi que satisface


la condición u (xo) = O 11que ItO se anula en ningún otro punto del semi-
intervalo Xo < x ~ x¡, en el arco AB no habrá puntos conjugados del
punto A. En este caso el arco AB de la extrema! se puede incluir en Un
campo central de exiremales con centro en el punto A (xoo y{).
En la e¡uací6n (2) hay gue tomar en las funciones Fy'fl (x. Y. y'),
F 1111' (:c. y, 11 ) Y F '1/'11' (x, y, 11 ) en lugar de 11 (x) el segundo miembro de
la ecuación de la extremal IJ = 11(x, C~).
EJEMPLO 10. ¿Se cumple la condiciÓn de Jacobi para la extremal
de la funcional

J (y (x)] = r
a

O
(y'2+xZ) dx

que pasa por los puntos O (O, O) Y B (a. 3)?


SOLUClON. En este caso la ecuación de Jacobl tiene la íorma
u" = O. Su solución general es u = C1x +
e,. De la condición u (O) =
= O encontramos e, = O de modo que u = Clx. Estas soluciones
u = CIx (el =1= O) no se anulan para ningún valor de a > O. Por con-
siguiente, en el arco OB de la extremal no habrá punto conjugado del
punto O (0, O). Es decir, este arco se puede incluir en un campo central
de extrernales con centro en el punto O (0, O). Es fácil ver que la extre-
mal buscada es la recta y = ~x que se puede incluir, obviamente, en
a
el campo central de extremales y = e,x.
EJEMPLO J1. ¿Se cumple la condición de Jacobl para la extrema]
de la funcional
a
J[y(x)]= J (y'2_4y2+e-:x )dx z (a-4= (n+}) n)
O
que pasa por los puntos A (0, O) Y B (a, O)?
SOLUCIONo La ecuación de J acob l tiene la forma u· 4u = O Y +
su solución general es u = Cl sen 2x +
C~ cos 2-". De la condición
u (O) = O encontramos Ca = O de modo que u = Cl sen 2-". Si a <~ t

la función u no se anula para O < x .;;;;a y la condición de J aeob i se


cumple; en cambio, si a >~ t la solución u = CI sen 2;c de la ecua-

clón de J acobi se anula en el punto x = ; perteneciente al segmento


{O. al y en el arco de la extremal y = O (O ~ x ~ a) hay un punto
conjugado del punto A (O, O). Por consiguiente, si a > ~ , no existe
campo central de extr emales que comprenda la extremal dada.

En los problemas siguientes analizar si se cumple o no la


condición de J acobi,
88 CAP. IJ. EXTREMO DE FUNCIONALES

1
135. J Iy(x)]= J (12xy+y'2+x )dx; 2
-1

y(-l)= -2, y(I)=O.


a
136. J[y{x»)= J (y'2+9y2-3x)dx;
O
y(O)=O, y(a)=O.
1
137. J ly(x)] = J (1 +y(2)dx; y(O)=y(l)=O.
O
n

J38. J ly (x)] =) y' eY' dx; y (O) = 1, y (a) = b.


o
2n
J 39. J Iy (x)] = Io (y'Z - y2) dx; y (O) = O, y (2n) = 1.

140. Demostrar que si la función integrando de la fun-


cional
b
Jfy(x}J= J F(x, y')dx
a
no contiene y explícitamente, cualquier extrernal puede ser
siempre incluida en un campo de extremales.

funcional J Ir
oBSe~vACI()N. La condición de Jacobi es necesaria para que la
(x)l alcance su valor extremo: o sea. cuando la extrema!
A8 realiza e extremo. el punto conjugado de A no puede estar en el
intervalo XI) < x < Xl' Por ejemplo. la funcional
a
J[y(x)J=.\ (y'4+I)dx; y(O)=g(a)=O;
O
alcanza su valor mínimo en la exíremal y == O. En esta extremal no
hay puntos conjugados del punto O (O. O).
EJEMPLO 12. La Iunclonal
!I
T1t
Jlv(x)l= J O
<V2-V'Z)dxi '1(0)=0, !I(: n)-o;
§7. CAMPO DE EXTREMALES 89

no alcanza extremo en la ex trernal y == O porque en el intervalo


(O, !Jt) está el punto O· (a, O) conjugado del punto O (O, O) (ya que
la solución de- la ecuación de Jacobl que se anula en x = O es u =
=C1 sen x y u se anula también en el punto x = tt E (O, n) ) . !
Efectivamente. lomemos como curva «próxima» a y == O la curva
4
sen "5 nx
I/n (x) = nll ; para esta curva se cumplen obviamente las con-

diciones y (O) = Y ( 5: ) = O e y~ (x) = 5~ cos n ~ x, Entonces


tenemos J [O) = O y

J[
sen
4
5TlX
n2.
J= J 1. lt
4 sen2 (

n4
i;-x)
dx-
o
5
T"
_ Jr (_4_)2
5/t
cos2 (~x)
5
tlx= 8n2
5n (_l__ ~)
25
<O 112
o
para todo entero n ;;;.2. Por consiguiente. la extrema! y == O no realiza
el mínimo de la funcional considerada ya que existen curvas próximas
a U == O en las que son negativos los valores de la funcional. Considere-
mos abora la familia de curvas Un (x)
y == O en el sentido de proximidad
= + sen ~ x cercanas a la curva
de orden cualquiera. Es facil ver que
5 ~
T7t sen2~.I: T.!t
J IUn (x)] = J
r 5
.n2. dx -
r
J 25ñ2C'OS"5
16 2. 4
xdx=
9tt
40n2 > O.
O O
Por consiguiente, la extremal y !::!!I O tampoco realiza el máximo
de la funcional considerada.
141. Supongamos que en la funcional
b

J fy(x)] = J F (x, y, y') dx


a
la función integrando F tiene derivadas parciales acotadas
de tercer orden respecto a las variables U e y' en cualquier
90 CAP. 11. EXTREMO DE FUNCIONALBS

recinto acotado de variación de y e y'. Sean y = y (x) e y =


= y (x) + '1 (x) dos extremales cercanas. Demostrar que la
función '11 (x) satisface la ecuación de Jacobi
FIIII'YI+FIIII,T]'- :x (FlIlI'T]+FII'v'T]')=O
salvo una infinitésima de orden superior respecto a la dis-
tancia de primer orden entre estas extremales.
2°. Condiciones suficientes de l.egendre. Condición suücíente para
que la extrema} de la funcional
XI
J [y (x)l ~ J F (x, y, !J') dx; y (xQ) = Yo,
xo
se pueda incluir en un campo de extremales es que se cumpla la condi-
ción reforzada de Legendre, Esta consiste en que la desigualdad
FII,y' >O
se cumpla en todos los puntos de la extrernal considerada (o sea, para
todos los x E [xoI xll).
EJEMPLO 13. Consideremos la funcional
2
J(y(x)]= J (y"+y'2)dx; y(O)=I, y(2)=5.
O
+
Sus extremales son las rectas y = C1x Ca. La extremal buscada que
satisface las condiciones de frontera es la recta y = 2x 1. +
En este caso F 11'11' = 12y'i +
2 Y en todos los puntos de la extre-
mal y = 2x + 1 tenemos F IJ'IJ' = 50 > O. Se cumple la condición
reforzada de Legendre y, por consígulente, la extremal 11 = 2x I +
se puede incluir en un campo de extremales.
Esto se ve también directamente. La extremal 11 = 2x 1 queda+
comprendida en la Iarnllia monoparamétrica de extremales 11 =o:
;:: 2x+ a. (o: es el parámetro) que lorma un campo propio.
EJEMPl.OH. Consideremos la funcional

J (y (x)] = !
-1
(xlly'lI + 12y2) dx;

y(-I)=-I, y(l)=I.
La ecuación de Euler para esta íuncional tiene la forma
x'yN + 2:ty' - 12y = O
Y su solución general es
11= C1x:' + Cax ....
58. CONDICIONES SUFICrENTES DE EXTREMO 91

La extremal que satisface las condiciones de frontera consideradas es


g = :x8.
No puede ser incluida en un campo. La (mica familia monopararnétrtca
de extremales que la contiene es y = a:x8. Pero esta familia no cubre
el recinto que contiene el punto de abscisa x = O (porque las extremales
de esta familia no pasan por los puntos del eje Oy con ordenadas distin-
tas de cero).
En este caso tenemos F 11'11' = 2x2 y la condición de Legendre no
se cumple para x = o.
Analizar la posibilidad de incluir la extrernal en un
campo para las funcíonales siguientes
i
142. J (y(x)l= ~ (y'Z_yy'3)dx; y(O)=O, y(l)=O.
o
a
143. J[y(x)J= J y'3dx; y(O)=O, y(a)=b>O.
O
XI
144. J[y(x)1=) n(y)Vl+y'Zdx;

y (xo) = Yo, y (Xi) = Yt, n (y) > O.


a
145. J [y (x)) = ~(6y'2 - y'4.) dx;
'o
y(O)=O. y (a) =b, b>O.
§ 8. Condiciones suficientes de extremo de una funcional
Se considera el problema variaclonal elemental, o sea, se considera
la funcional
:1:,
J [y (x)) = 1 F (x, 'l. !I'l dx (1)

con las condiciones de frontera


y (Xo) = Yu, IJ (Xl) = 111' (2)
l0. Condiciones suficientes de Welerstrass. Se denomina función
de Weierstrass E (x, y, p, y') la función definida mediante la igualdad
E (x, y, p, y') = F (x, y, y') - F (x, y, p) - (!I' - p) Fp (x, y. p)

donde p = p (x, y) es la inclinación, en el punto (x, y). del campo de
extrema les del problema variaclonal (1) y (2).
92 CAP. [1. EXTREMO DE FUNCIONALES

CONDlCIONES SUPICIENTC.S DE EXTRE"'O DaBIL.


La curva e realiza el extremo débil de la funcional (1) si;
1) la curva e es una extremal de la [uncionai (1) que satisface las
condiciones de frontera (2), o sea, es la solución de la ecuacián de Euler
para la funcional (1) que satisface Las condiciones (2);
2) la extremal e puede ser incluida en Un campo de extrema les
(esto tendrá lugar. en particular. si se cumple la condición de Jacobi);
3) La función de \reíerstrass E (x, y, p, y') conserva su signo en
fodos los puntos (x, y) próximos a la extrema! C y para valores de y'
próximos a p (x, y). La funcional J ly (x)) tendrá máximo en e si E ~ o
U mínimo si E;;;" O.
CONDICIONES SUFICIENTES DE EXTREMO FUERTE.
La curva e realiza el extremo fuerte de la funcional (1) si:
1) la curva e es una exiremal de la funcional (1) que satisface. las
condiciones de frontera (2);
2) la extrema! e puede ser incluida en un campo de extrema les,
3) la función de Weierstrass E (x, y, p, y') conserva su signo en
lodos los puntos (x, y) próximos a la extrema! e y para valores cualesquiera
de y'. Si E ~ O. se tendrá máximo y, si E;;;" 0, se tendrá mínimo.
OBSERVACióN. La condición de Weiersirass es necesaria para que
exista el extremo en el sentido siguiente: si para ciertos valores de U'
la función E tiene signos opuestos en los puntos de la ex trernal, no se
alcanza el extremo fuerte; si esto ocurre para cualesquiera valores de
U' por próximos que sean a p, tampoco se alcanza el extremo débil.
EJEMPLO 1. Analizar el extremo de la funcional

1
Jlu{x)j= J {y'3+U')dx; y (O)=0, U(I)=2.
O

SOLUCIÓN. La ecuación de Euler para la funcional considerada tiene


la forma g'l' = O de modo que las extrernales son las rectas y =
::::::~G.l~+ C,.La extremal que satisface las condiciones de frontera
es la recta U:::. 2x. La inclinación del campo en los puntos de esta
t~jIema.l es p = 2. Es evidente que la extremal y = 2x se puede
ioclu ir en el campo central de. extrernal es y = C» con centro en el
punto O (O, O). Es fácil probar así mismo que en este caso se cumple la
con dlción de Jacobi. La ecuación de J acobi es en este caso - :,. (6y' u')=
= O, donde, debido a la ecuación de la extremal, hay que poner y' == 2.
Por consiguiente. la ecuación de Jacobi toma la forma UN = O de modo
que u = C1x +
C2• De la condición u (O) = O obtenemos e~
= O.
Puesto que para el :::¡i= O esta solución u = el): no se anula en ningún
punto a excepción del punto x = O, la condición de J acobi se cumple.
Formamos la función M Weierstrass
~ (XI y, p, {) = y'3 + y' _ p3 _ P _ (!J' _ + 1) =
p) (3p2
= (t - p}3 (V' + 2p).
p. CONDléIONE.S SUFICIENTES DE EXTREMO 93

El primer factor es no negativo cualesqu lera que sean los valores de


y' y el segundo es positivo para valores de y' próximos él 2. Por con-
siguiente, se cumplen todas las condiciones de mínimo ddlil. Además,
es fácil ver que para y' < -4 la función E será negativa y no se cum-
plirá la condición suficiente de extremo fuerte ya que para el extremo
fuerte se exige que la función de Weierstrass f] conserve su signo para
valores cualesquiera de y'. Si se
tiene en cuenta la observación de
la pág. 92. se puede llegar a la con- ¡I
clusión de que en este caso no hay
extremo fuerte.
EJEMPLO 2. Analizar el extre-
mo de la funcional
1

Jlg (x)] = Jo (X+2Y+! y 12) dx;


y(O)=O, y(I)=O.

SOLUCIÓN. La ecuación de
Euler para esta funcional tiene la ---'o*--"""'7'--_''--~>-:--~'''/(
forma y~ = 2. Las extrernales son
las parábolas y = X2 +
elx e2• +
La extrernal que satisface las
condiciones de frontera es y =
= X2 -)C. Formamos la ecuación
de J acobí - :x u' =- O. o sea,
u' = O. Su solución general es
u = e¡x + e2• La condición
u (O) = O da e2 = O Y la con di-
clón J acobi se cumple ya que
u = e1x con el '=1= o no se anula
en ningún punto del segmento [O, 1]
a excepción del punto x = O; es de- Fig.. 11
ctr, la ex tremal !I = X2 -)C se
puede incluir en un campo central
de extrernales con centro en el punto O (O. O). a saber. en el campo
y = X2 + ex (fig. 11). La función de Weierstr ass tiene la forma
E (x, y, p, y') = ~ (!J' - p)2. Se puede ver de aquí que E =

= ~ (y' - p)2 > O para cualesquiera valores de y' Por consiguiente,


la funcional considerada alcanza en la ex trernal y - X2 - x mínimo
fuerte igual a J (x';\ - x] ~ ~ .
CAP. 11. EXTltEMO DE FUNCloNÁLt:S

Analizar el extremo de las funcionales siguientes.


1
146. J[y(x)l= 5 ~(y2+ ; y'2)dx; y(O)=l, u(l)=e.
a
1
147. J IY(x)] = Ja e'IIU'2dx; g(O) =0, u(I)=ln4.

2
148. J[y\x») = Jr y'Z
x3
dx; g(I)= 1, g(2)=4.
i
el

149. J[U(x»)= Jo :~ ; U(O)=O, g(a)=b, b>O.

150. J(U(x»)= J (1+x)y'2dx; y(O)=O, y(I)=1.


o
n/2
151. J[y(x)l= J (lI'-!j2)dx; y(O)=I, y (~)=1.
o
2
152. Jly(x)1= 5
-1
+
y' (1 x2y') dx; y(-I)=l, y(2)=4.

t
153. J fu (x)) = J (y'3+ y,a) dx; y( -1) = -1, y(l) = 3.
-1

2°. Condldones suficientes de legendre. Supongamos que la


derivada parcial F 11'11' (x, y, y') de la función F (x, y, y') es continua
y que la extrernal e está incluida en un campo de extremales.
SI en la extrema! e se tiene F1/'11' > 0, en la curva e se alcanza
mínimo d~bll; si en la extrema! e se tiene FlI'lI' < O, en ella se alcanza
el máximo d~bil de la [uncional (1). Estas condiciones se llaman condi-
ciones reforzadas de Legendre.
En el caso en que F JI'IJ' (x, y. y') ;;;;.O en todos los puntos (x. y)
próximos a la exttemal e y para cualesquiera valores de y' , se tiene mlní-
mo [uerte y en. el caso en que FfI'y' (x. y. y') ~ O para estos valores de
los argumentos, SI! tiene máximo uerte.
t 8, éONDJCU)NES SlJPICJENtES DE EXtREMO 9$
EJEMPO 3: Analizar el extremo de la funcional
1
J(Y(X}l= ~ (y'3-o.y')dx; y (0)=0, y(I)=-2
o
(a. es un número real cualquiera).
SOLUCIONo Puesto que la [unción integrando depende sólo de !J',
las extrernales son las rectas y CIl: =
Cs. La extrernal+ que satis-
face las condiciones de írontera es la recta g = -2x que se puede incluir
en el campo central de extrernales !J = Cx. La inclinación del campo
en esta extrema! es p = -2. A continuación, calculamos F 1/'1/' = 6y'.
En la extremal tenemos Fu'u' = -12 < 0, o sea, la funcional alcanza
máximo débil en la línea y = -2x. Para valores arbitrarios de y'
el signo de FI/'u' no se conserva y, por consiguiente. no se cumplen las
condiciones suficientes de máximo fuerte.
En este caso la función de Welerstrass E (x, y. P. g') tiene la forma
E (x. y, p, !J') = (J/ - p)2. (JI' + 2p)
y para determinados valores de y' tiene signos opuestos. Teniendo en
cuenta la observación de la pág. 92, deducimos que no hay máximo
fuerte.
EJEMPLO 4. Analizar el extremo de la funcional

2
J Iv (x)] = J (eY' +3) dx¡ y (O)=0, V (2) = I.
o
SOLUCIONo Las exlremales son las rectas y=C1x+C2.' La extre-
mal que satisface las condíclones de frontera es la recta y = ; ; puede
ser incluida en el campo central de extremales y = Cx. Tenemos. ade-
más •. FII'II' (x, y, y') = ell' > O para cualesquiera valores de y'. Por
consiguiente, la funcional tiene mínimo fuerte en la extremal !J=1.

EJEMPLO 5. Analizar

J(y(x)}= Jo Vr el extremo de la funcional

y
dx¡ y (0)=0, y (a)=Yi'

SOLUCION. La función integrando no depende explícitamente de


x y, por eso, tenernos F - y' FU' = el o, en nuestro caso,
CAP. 11. eXTREMO DE FUNCIONALES

de donde
J
~-=-~==-=CI - 6 y {l··ry'2)....eh
VyYl+y'2
donde CI = (~I )2 . Pongamos y' = ctg 2t . Tendremos I
y = ej sen'2.2" =

= ~1 (I-cos/), Además.

dx =.......!!.J!_ = el sen t dt el sen2..!.. dt


t t 2
clg"2 2ctg"2
e, integrando, encontramos
_e r
x- I J
(1 - cos t) dt
2
el
=T(t+se.nt)
+ C2•
Tenemos pues (c = ~ )
x =C (t - sen t) + C2, }
y= e (l-cos t),
o sea, las ecuaciones paramétricas de una lamilia de cicloides. De la

J(

Fig. 12

condición y (O) = O encontramos C2 = O. El haz de icJoides


x= C (t -sen r), }
g=C (l-cos 1)
forma un campo central con centro en el punto O (O, O) que comprende
la extremal
x=R (t -sen t), }
y ~.R (1 -cos t),
donde R se determina de la condición de que la cicloide pase por el
segundo punto frontera B (a • .vI), sí a < 21tR (fig. 12).
§S. CONDICIONES SUFICIENTES DE EXTREMO 97

Empleamos la condición de Legendre. Tenemos

FU'lI'= 3 >0
Yv (1 +y'2)2
para cualesquiera valores de y'. Es decir, para a < 2nR la funcional
considerada tiene mínimo fuerte en la cicloide
x=R(t-sent), }
g=R (l-cos t).
Empleando la condición de Legendre, analizar el extremo
de las funcionales siguientes:
1
J54. J[y(x)]= S (y'2+x2)dx; y(O}=-I, y(I)=l.
o
3
155. J (y (x)] = J
2 Y
x~s dx; y (2) = 4, y (3) = 9.

2
156. J{y(x))= 5 (xy''''-2yy'8)dx; y(I)=O, y(2)= 1.
1
a
157. JIY(x))= 5
O
(l-e-II'S)dx; y(O)=o, y(a)=b.

1
158. J(y(x)]= J yy'
O
2dx; y(O)=p>O, y(l)=q>O.

159. Analizar el extremo de 1a funcional


1
J lY(x)} = ) (ey,z+y2+x2)dx; y(O)=O, y(t)=];
o
para distintos valores del parámetro ~"
EJEMPLO 6 (problema de Euler), Una barra vertical de longitud 1
Se somete a una carga axial P. Para un valor determinado de P (fuena
crítica de Euler) la barra se comba. Se pide determinar el valor mlnimo
de la fuerza P que¡rovoca la flexión longitudinal.
SOLUCIÓN. Sea el módulo de elasticidad, sea I el momento de
inercia mínimo de las secciones transversales de la barra, seá p el radió
de curvatura y sea q¡ .el ángulo entre la tangente y el eje.
1-01387
9B CAP. Il. EXT~EMO DE FUNCIONALES

La energía potencial de la flexión se determina mediante la


fórmula
1
I f dS
Ut="2 El J V'
o
Si el ex tremo de la barra desciende en
I

O" = J
O
(I-cos cp) as,

la energía potencial de la barra disminuye en


l
Uz= Pa=Pl-P ~ coscpdS.
O
Si antes de la deformación la energía potencial es igual a cero,
después de la deformación estaré dada por la Iórrnula
l

U=U,-U:l= J (~
O
El p~ +P~oscp)dS-Pl.

dS cpu
Puesto que p = dcp Y (para pequeños valores de q» cos q>:=:::: 1- T'
se tiene
I l

U= fI [El ( ~; ) 2- P<pll] dS:=::::~ j [E


f ( ~~ ) 2_ P!p2] d»,
o O
En el caso de equilibrio, la energía. potencial toma su valor mini
mo. Por eso. el problema se reduce a la determinación del mínimo de
la integral

En este caso
F=EI (!~) _Pcp~
2

Y la ecuación de Euler tiene la Iorrna

q>" +a.'aq>=O, donde a.2 = :1 .


La solución general de esta ecuación es
cp = Cl sen a.x Ca cos + a.x.
58. CONDICTONES SUP1CIENTES DE EX1REMO 99
Puesto que tg q¡ ~ q> para valores pequeños de q¡ y como, además,
tg q> = y', se tiene
y' = Cl sen ax + el cos ax,
de donde
g=
Ctcosez,x+C2senax C.
ex a +
Si el extremo inferior de la barra está en el origen de coordenadas,
será y = O para x = 0, o sea, Cl == C = O:
e
y= -1. sen ex.
a.
Veamos si se cumplen las condiciones de Legendre y de J acobl,
Es obvio que la condición de Legendre se cumple:
(JzF
oq>'1! = 2EI > O.
La ecuación de J acob tiene la forma
í

E l z" + Pz = O Ó z" + a.'z = O


con la particularidad de que z (O) = O. Por eso, la solución de la
ecuación de J acob i será
z = A sen ax.
La función z se anula para Xli =.!!!:..
a
(k = 1, 2, ... ) de modo que la

condición de J acobi se cumplirá si l,;;:a.!!...


a. De aquí
n3
P>-¡¡-El.
El valor míntrno de la fuerza critica de Euler será
ni
Prnfn=-¡¡-EI
y la ecuación de la curva de Ilexión será
Cz nx
y = -;:;:-
sen -1- ,

3°. Figuratrl2. Sea dada la funcional


b
J IY (x)1= } F (x, y. y') dx .
a
Tomando x e y como parámetros, consideremos la función y =
= F (x, y, y') en tanto que función del argumento !J'. El gráítco de
esta [unción en el plano de las variables (y'. Y) se denomina figuratriz.

100 CAP. rt. EXTREMO bE PUNCIONA LES

Es fácil probar que la función de Weierstrass E (x, y. p, !J') representa


la diferencía entre las ordenadas de la figuratriz y las ordenadas de la
tangente a la liguratriz trazada por el punto de abscisa y' = p. Si la
{unción de Weíerstrass conserva su signo para ciertos valores de g'.
ello significa que la figuratriz está por encima o por debajo de la
tangente para esos valores de !J'. En este caso hay mínimo débil. Si
la Ilguratrtz está a un lado de la tangente para todos los valores de
y' y para los valores de los parámetros x e y próximos a los puntos de
la extremal, hay extremo fuerte.
La condición suficiente de Legendre se expresa en estos términos
así: si para todos los puntos (x. y) próximos a la extremai la ftguratriz
es cóncaua hacia las y positivas o negativas, hay extremo fuerte.
EJEMPLO 1. Analizar el extremo de la funcional
(1

Jly(x»)= ~ y''Iodx.; y (0)=0, y(a)=b, b>O.


O
SOLtxrON. Las exlremales son las rectas g = C1x +
C2• La extre-
mal buscada viene dada por la ecuación y = ~ x, Puede ser inclulda
ti

Fig. 13

en un campo central de extremales, La figuratriz es la parábola


y = y'~ (f1g. 13). Es fácil ver que toda 1a figura triz está por encl ma
S 8. CONDICIONES SUFICIENTES DE EXTREMO 101

de la tangente trazada a la misma en el punto p = s: cualesquiera


a
que sean a y b (a =F O). Por consiguiente, la funcional considerada tiene
mínimo fuerte en la extremal y = ~ x_
EJEMPLO 8. Analizar el extremo de la funcional
a
Jlu(x)l= I
O
g'Sdx; y(O)=O, y(a}=b, b>O.

b
La extremal buscada es la recta y = - x que se puede
SOLUCiÓN.
a
incluir en el campo central de extrernales y = ex con centro en el

Fig. 14

punto O (0, O). La fuguratriz. es la parábola cúbica Y = y'S (fig. 14).


Para valores de y' suficientemente próximos al valor p = !!..
a
la rigu-

ratriz está sobre la tangente a la misma en el punto de abscisa y' = !...


a
De la lig. 14 se puede ver que la figuratrlz corta la tangente en el punto
de abscisa y' =- 2b ya la izquierda de este punto aparece por debajo
a
102 (.AP. JI. ~XTRf.MO DE PUNCIONALl,S

de la tangente. Por' lo tanto. hay mínimo débil en la extremal


b •
=-x.
a
Nótese que para p = O (esto corresponde al caso fJ == O en el que
la extremal es un segmento del eje Ox) la tangente a la figuralriz es
el eje Oy' y el punto O (O, O) es un punto de inflexión de la figuratriz.
Teniendo en cuenta la observación de la pág. 92. vemos que en cual-
quier vecindad del punto O (O. O), por pequeña que sea, la Ilguratrlz
tiene ordenadas tanto positivas como negativas. Por lo tanto. la
función de Weierslrass E tiene signos opuestos para valores de JI'
tan próximos a p = O como se quiera y. por consiguiente, en esto caso
no se alcanza ni siquiera el extremo débil.
EJEMPLO 9. Probar que la ex trernal JI = O del problema variacional
I
JIY(Xil=) (y't-yy'3)dx; y(O)=y(I)=O;
O
realiza el mínimo débil de la funcional.
SOLUCION. En este caso la condición de Legcndre da

FlI'II,I/la.e()= (2-6yy') -.: 2 > O.


1/1'-"'0
o sea, se alcanza mínimo déb il en la extremal y = O. Demostremos
(fue en esta extrema! no se alcanza el mínimo fuerte. Consideremos la

y'

Fig. 15

figuratriz y = y'" - !JY" para los valores y >


O (fig. 15). De la fig. 15
se ve que la tangente a la Ilguratrtz en el punto de abscisa p = O corta
la figuratriz en el punto y' = _!_.
y
Es decir, para los puntos (x, y),
con y > O. próximos a los puntos de la extremal y = O. la función de
Weierstrass E es positiva para valores de g' menores que ~ y es negatl-
g
§ 8. CONDICIONES !'íUP(CJENT~S DE FXTl~F.MO 103

va para y' > 2...


y
Según la observación de la pág. 92, no hay mlnlmo
fuerte. Una situción semejante Se tiene también para y < O.
Lo que destaca este ejemplo es que en él la condición FI/'Y' > O
se cumple en la extremal para cualesquiera y' y, sin embargo, ello
no implica la existencia de extremo fuerte.
Empleando la figuratriz analizar el extremo de las funcio-
nales siguientes:
j

160. J [y (x)] = Jo (1+x)y'Zdx; y(O)=O, y(l)=-2.

2
161. Jly(x)l= J y (l+x y')dx; 2 y(-1)=y(2)=1.
-1
a
162. J[y(x))= J (1-e-II'4)dx¡
O
y(0)=0, y(a)=b, (b>O).
a
163. J(y(x)J= J (6y'2_y'''+yy')dx;
O
y(O)=o, y(a)=b (1»0).
OBSERVACIÓN. La 110 negatividad de la segunda variación es con-
dición necesaria, pero no suficiente, para que la funcional J [y (x»)
alcance mínimo en la curva.
EJEMPLO 10. Consideremos la funcional
1
1[y (x)1= J y' (x-y) dx
o
en el espacio e [0, l], La ecuación de Euler tiene la forma FII = O
Ó y = O. Para O ~ x ~ 1 la segunda variación de la funcional en la
extremal y = O
1
{Ji! (O, 6Y1 = J X (<'Iy)3 dx
o
es positiva para todo ay ::¡6 O. SIn embargo, en cual quier vecindad del
cero la funcional J [9 (x)l toma también valores negativos; basta fijar
lO4 (.;AP. 11. EXTREMO DE FUNCIONALES

8 >- O Y considerar la función


-x+s,
!J~ (x) = { O, x:;;;"e.
8'
Entonces tendremos J (Ya (x)] = -6 < O para cualquier e> O.
DEFINICION. Se dice que \a funcional cuadrática L, (h] definida en
un espacio narmado es fuertemente posi/lva si existe una constante
k> O tal que
para todo h.
CONDICION SUFICIENTE DE MtNIMO. Condición suficiente para que
la [unclonal J 111 (x») definida en un espacio normado tenga mínimo
en el punto estacionario y = Yo es que su segunda variación sea fuerte-
mente positiva en y = Yo. o sea, que se cumplo. la condición
(J'J I Yo. Óy) ;;;, k 11 6y IIz•
donde k = consto k > O.
4°, Supongamos que se busca el extremo de la funcional
%1

J [lIit Uz, . oo, Ynl= j F (x. !lit yz. '''' {In. !ll, Yí. oo', y;'> dx, (3)
:1:0
que depende de n funciones Yl (x), Ya (x) ••.. , !/n (x), con las condi-
ciones de frontera
YA (Xo) = YkG. YIt (Xl) = YIt] (k = 1, 2 ••. o. n).
La condición reforzadcl de Legendre consiste en que las desigualda-
des

F~.",
1 n

FI/'II'
In >0 (4)

n1 F~.",
FII'''' ni
. o o F",,,,
nn
se cumplan en todos los puntos de la extremal considerada de la Iun-
cional (3).
La oondicion reforzada de Jacobi consiste en que el segmento
(.to. "J1 no contenga punto conjugado del punto XO°
f 8. CONDICIONES SUFICIENTES DE EXTREMO 105

La condición reforzada de Legendre (4) conjuntamente con la


condición reforzada de J acobl garantizan por lo menos la existencia
del mlnlmo débil de la funcional (3).
EJEMPLO 11. Analizar el extremo de la funcional
1

J fu (x), z (x)] = J (y'B+2'2) dx; (5)


o
y (O) = 0, z (O) = o, } (6)
y(l)=t, z(I)=2.
SOLUCION. Las ecuaciones de Euler para la funcional (5) son
y- = o, z· = O.
de modo que
y=Cj +C2x, } (7)
z=Cd C/ox,
Empleando las condiciones (6), obtenemos
CI = O. C. = l. Cs =O Y C, = 2.
La ex tremal buscada
y=x, } (8)
z=2x
representa una recta que pasa por el origen de coordenadas.
Tenemos
Fy'Y' = 2. Fy'z' = 0, FZ'II' = y ° FZ'%' = 2.
La condición reforzada de Legendre se cumple:

FY'y,=2>0. IFr.'Y' FY'~'1=12


FY'I/'
Fz'z' 10 2
°1=4>0.

Veamos si se cumple la condición reforzada de Jacobl.


Una de las definiciones de punto conjugado es la siguiente (véase
[3)).
Supongamos que se tiene una familia de extremales de la funcio-
nal (3) que arrancan del punto inicial (xo. YIO. ' , ., YnO) en direc-
ciones próximas rero linealmente independientes. Se dice que el
punto x" E [xo, Xl es conjugado del punto Xo si existe una sucesión
de extrema les, que arrancan todas del punto inicial y que son tan
próximas como se quiera a la ex tremal considerada, tal que cada una
de estas extremales corta la extremal considerada con la particulari-
dad de que las abscisas de los puntos de intersección convergen hacia
el punto X·,
En nuestro caso las extremales son las rectas (7). Todas las extre-
males que arrancan del punto (O, O, O) cortan la extremal (8) en este
106 c,'\I'. 11. EXTREMO DE FUNCIO .'¡ALES

punto solamente Por lo tanto. el segmento (O IJ de variación de x


no contiene punto conjugado del punto Xo = c. Es decir, se cumple
tanto la condición relorz ada de Legendre como la condición reforzada
de Jacobí de modo que la extremal (8) realiza el mínimo débil de la
funcional (5).

Analizar el extremo de las funcionales siguientes:


t
164. J[y(x). z(x»)= J 'Vl+y'2+z'2dx;
o
y(O) ·,,,0, y(I)=2. z(O)=O, z(1)=4.
1
165. J [y (x), z (x)] = ) (y'2 +Z'2 + 4z) dx;
o
y(O)=O, y(I)= 1, z(O)=O, z(I}=O.

§ 9. Extremo condicionado
l0. Problema Isoperlmétríce. Sean F (x, Y. [1') y G (x, y, y') dos
funciones.
El problema isoperimétrico consiste en lo siguiente: entre todas
las curvas y = y (x) E el [':<0' Xl] a lo largo de Ias cuales la funcional
;tI

K [y (x)] = JG (x, y, !J') dx


xo
t iene un valor fijo! hallar la curva en la que la funcional

!lel

J [y (xli =) F (x., y, y'l dx


xo

alcanza su valor extremo.


Suponemos que las funciones F y G tienen derivadas parciales
continuas de primer y de segundo órdenes para Xo ~ x ~ Xl Y para
valores cualesquiera de las variables y e y'.
TEOREMA DE EULER. Sí la curva y = y (x) realiza el extremo de la
funcional
Xl

J !y (x)l-= j F (X, y, y') dx


Xo
p. EXTREMO CONDICIONADO 107

con las condiciones


Xi

K[y(x)l=) G(x, y, y')dx=l, Y (Xo) =Yo. Y (Xt}=Yt.


xo
y si U = Y (x) no es extremal de ta funcional K existe una constante A
tal que la curva y = !I (x) es extremal de la funcional
XI

L (11 (x)) = J
[F (x, y, y')+W (x, y, .11'») dx,
:ro
EJEMPLO I (problema de Dido). Entre todas las curvas cerradas
de longitud 2l hallar la curva que comprende el área máxima.
SOLUCION. Observemos, ante todo, que dicha curva debe ser con-
vexa. Efectivamente, de lo contrario encontraríamos una recta L

A E

Fig. 16

(Iig. 16) tal que, al reflejar en elJa la parte BCD de la lrontera, obten-
dríamos un recinto de área mayor que el inicial siendo la longitud
de la frontera la misma que antes.
Observemos también que toda recta que div ide por la mitad la
cerrada que comprende el área máxima ha de dividir por la mitad la
propia área. Efectivamente, supongamos lo contrario y sea L1 una recta
que no cumple esta propiedad. ReUejando en L1 la parte de la figura
de área mayor, obtendremos una curva de idéntica longitud pero que
comprende un área mayor.
Tomemos como eje Ox cualquiera de las rectas que dividen por
la mitad la curva; llegamos entonces al problema siguiente.
Hallar la línea y = y (x), Ij (-a) = y (a) = 0, de longitud fija
1 > 2a que conjuntamente con el segmento -a ~ x ~ a del eje Ox
encierre el área máxima. Por lo tanto. el problema se reduce a hallar
el extremo de la funcional
el

J[y(x))= J ydx; y(-a)=y(a)=O;


-a
108 CAP. 11. EXTREMO DE Pl:1NCIONALES

con la condición complementarla de que


a
K(y(x))= f
-4
VI+y'3dX=1 (/>2a). (1)

Formemos la (unción auxiliar

H=F+AG=y (X)+A 'Vl+y'2 (x)


y consideremos la Iuncicnal auxlllar
a
L ty (x)) = JH (x, y, y') dx, (2)
-a
La ecuación de Euler para la funcional (2) tiene la forma

-d ( AY' ) -1
dx VI +y'a - ,
de donde
Ay'
VI +y'"
Resolvrendo la última ecuación respecto a y', encontramos
dy x+e¡
(3)
-¡¡¡= 'V),II_(x+e¡)i .

Integrando la ecuación (3). obtenemos


(x+cl)a+ (y+e2)2=A:a,
o sea, la cireunlerencia de radio ~ con centro en el punto (-el. -es).
Las constantes el y C~ así como el parámetro A se determinan de las
condiciones de frontera y (-a) = y (a) = O y de la condición isoperi-
métrica (1). Tenemos
C¡=A2-(C1-a)2, }
q= ),3_ (e. +a)Z,
de donde

de modo que

Y=V~-V~ e
~xt~eMÓ CóNDICIÓNADÓ 109

La condición (1) da entonces

1 ')..
a
l= _dx =~arcsenT X !x-a =2J..arcsenT el
·V),2.-XZ x--01
-a
o sea,
a 1
-¡-=sen 2~ .

Resolviendo respecto a ), esta ecuación trascendente, encontramos un


valor determinado A = 1..0 Y después encontramos el valor de C2 =
= V),g - a~.
Es fácil persuadirse de que la ecuación ~ = sen ;').. tiene siem-

pre solución. Efectivamente, tomando ~J.. = t, esta ecuación quedará


reducida a sen t = ~a t, donde ~ =«<1 por hipótesis del problema.
La íncllnaclén de la tangente a la función y = sen t en el punto t =O
es : mientras que la inclinación de la función y =«1 es menor. Por
consígulente, los gráficos de estas funciones tienen un punto de ínter-
sección como minimo, a parte del punto O (O, O).
PRiNCIPIO DE RECIPROCIDAD EN EL PROBLEMA ISOPERIMETRICO.
Las extrema lesde la funcional
%1
J 19 (x)I= J F (x, y, y') dx

Con la condición complementaria


%t
K I.Y (x)]= ) a (x, Y. y') dx= consl
"'O
coincIden con las extremales de la funcional K [1/ (x») ron la condición
¡ (y (x)] = const.
Basándonos en el principio de reciprocidad. deducimos del proble-
ma de Dido el resultado siguiente: entre todas las curvas cerradas que
comprenden un drea fiia. la circunferencia es la curva de longitud
mlnima.
Es Iácll obtener este resultado directamente si se recurre 8, la
forma paramétrica del pro blema var íactona 1.
Sean
x=X{t), x (to)=x(tt), }
!I = y (/), y (to) =u (t1), to~ t '" tit
110 CAP, 11. EXTREMO De FUNCIONALES

las ecuaciones de una curva cerrada. El problema consiste en hallar


el extremo de la funcional
1
f (~2+y2/ dt
con la condición
J (xy-yx)dx=C.
Introduciendo la función
1
F={X2+y2)2+ ~(x¡'-y~),

encontramos (véase la pág. 6S) que la curvatura de la curva que


r
realiza el extremo es constante:
.!.=A.
r
Por consiguiente, la extremal buscada es una circunferencia.
Utilizando el principio de reciprocidad. se pueden resol ver, sin
realizar cálculos, algunos problemas cvarlacionales» de la Geometría
elemental.
EJEMPLO 2. Demostrar que: 1) entre todos los triángulos de base
y perímetro fijos. el de área máxima es el triángulo isósceles; 2) siendo
fijas el área y la base, el triángulo isósceles es el 'triángulo de perí-
metro mlnimo.
SOLUCION. 1) Tomemos una elipse cuyos focos son los extremos
de la base de los triángulos considerados (fig. 17). De la propiedad
y

CO(o,b)

Fig. 17

de 1a elipse deducimos que todos los triángulos A CB tienen el mismo


perímetro. Es evidente que el área máxima corresponderá al triángulo
n. EXTREMO CONDrcrON¡\DO 111

de altura maxirna lo que signHica que el ver tice del triángulo debe
coincidir con el vértice e
o de la elipse. El triángulo ACoB es Isósce-
les.
2) Según el pr incip io de reciprocidad, siendo fijas el área y la
base. el perímetro mínimo corresponde al triángulo isósceles.
EJEMPLO 3. Hallar el mínimo de la integral

"
J [y (x)) = J
l)
y'2dx

It

con las condiciones) y2dx= 1, y(O)=y ln),=O.


O
SOLUCiÓN. Formemos la funcional auxiliar
l't

L [y (x») = J (y'2+Ay2) dx
O

f consideremos su ecuación de EuJer

2A.y- :x (2y'}=O. o sea. y"-ly=O. (4)

Su ecuación característica es ,2 - 1.. := O, de donde '1,2 = ± Vl.


Está claro que A debe ser menor que cero: si aceptamos que A > O,
la solución general de la ecuación (4) tendrá la forma y = ele V~x +
+ Cae- V;:x, las condiciones de frontera y (O) = y (n) = O se cumpli-
rán sólo para Cl = 0, C2 = O, o sea, resultará y == O Y no se cumplirá

en este caso la condición J" ¡f dx = 1; de la misma forma, si J.. = O.


O
la solución de la ecuación de Euler (4) que satisface las condiciones
de frontera también será la función !J e; O. Por eso, consideramos que
A. < O de modo que 'l.a == ±i V=Xl y la solución general de la
ecuación (4) es !I = el sen -1...:< V
C2 cos +
h.x:. La condición1f
y (O) = O da Ct = O Y la condición !J (n) = O da -A = kl
(k = 1, 2, ... ). Es decir, y = Cl sen k.x, donde el no se ha determi-
n
nado aún. Utilizando la condición J y2 dx = 1, obtenemos
O
1'1

J Qscn kxdx= 2 1,
o
11.2 CAP. 11. EXTREMO DE flUNC10NALES

de donde el = ± V !.o sea, IJ == ± V! sen kx. Todas las extre-

males y =± V! sen kx pasan por los puntos (O, O) Y (n, O), pero

sólo para dos, a saber, y=± Ji ! sen x, se cumple la condición


de Jacobi. En estas dos extremales se tiene

J [y (x)l =Jr y'2 dx = f 2 cos2


J -;t X dx= J.
o o
EJEMPLO 4 (problema de Kelv in . Supongamos que en el plano
xOy está distribuida una masa de densidad continua !1 (x, y) y supon-
gamos que se tiene en el plano una curva e suave a trozos y dos puntos
P1 y P2 sobre la misma. Entre todas 1as curvas de longitud rija t
que unen los puntos Pi y P'1. hallar la curva que conjuntamente con
el arco PIP'/. de la curva e forme un recinto D de masa máxima. Los
puntos P1 y P.! pueden coincidír.
SOLUCION. Consideremos la función

v (x, y) = J !1(x, 11) dx,

Según la Iórrnul a de Green, tenernos

J J l.ttx,
D
y)dxdy= j J ~~ dXdy=fVdYt
D r
donde el contorno r se compone de la curva L y de la parte PIP,
de la curva C. A lo largo de esta última parte la integral toma un
valor determinado que designaremos por K. Aceptando que la curva
L está dada M forma paramétrica
x=x (/), }
lo ~ I ~ th
11= Y (t),
tendremos entonces
tI

J J !1(x, y)dxdy= J V (x, y)Ydi+K.


D t~
Por consíguiente, el problema ha quedado reducido a la determi-
nación del máxtmo de la funcional
It
Ir,= JV (x, 1/) y dt
t2
S 9. EXTREMO CONDICIONADO 113

con la condición de que


tl
~ Y;Z+y2dt=1.
¿,
Consideremos la función auxiliar

F=Vy+" V;:>. +yZ


y empleemos la forma de Weierstrass de la ecuación de Euler. Tenemos
av
F • = iJx' F. = O.
xv yx
F ••
F, = -==- = --__" 2

yZ • •
A
3
(x2+y2)
de modo que la ecuación de Euler en la forma de Weierstrass (:l>

1 1 av
7='1' iJx
o, recordando la expresión de la función V (x. U'J.
I ~t (x, y)
7= A.
donde r es el radio de curvatura de la curva pedida.
En el caso en que fA. (x, y) = const resulta que la curvatura de
la curva pedida es constante y, por consiguiente, las extrernales son
circunferencias, Queda claro que realizan el máximo de la funcional
JL'
También se denominan problemas {SorerimétricoS los problemas
variacionales en los que se pide hallar e extremo de la funcional
Xl

¡ (Yi> I/z, .. '. Yn]= J F (x, Yt. !lz ••. " Un, Yi, Yí, ., '. Un) dx (5)
:reo
con las asl llamadas condiciones isoperimétricas
:>:1

..~ G¡ (x, Ul' !lz•.... Yn. y~. y;, . '" y~)dx=ll l6)
:>:0

(i=I.2, ... , m),


donde 1, son unas constantes.
Para obtener la condición necesaria fundamental en el problema
ísoperimétrico sobre la determinación del extremo de la luncional (5)
8-01R67
114 CAP 11. EXTREMO De FUNCIONALES

con las condiciones (6) hay que formar la lunclonal auxiliar


Xl m
4l> IY¡, Y2, ... , Unl = J (F + ¿, ¡,¡G¡ ) dx ,
xo .=1
donde Al son unas constantes y escribir sus ecuaciones de Euler. Las
constantes arbitrarias Ch C2, ••• , C2n de la solución general del
sistema de ecuaciones de Euler así como las constantes Al' Az, •••
. . . , Am se determinan de las condiciones de frontera
!JII (.to) = !111o. YII (Xl) = YII1 (k = 1, 2, ... , n)
y de las condiciones isoper imétricas (6)
XI

IOidx='t (1=1,2, ... , m).


X(I

EJEMPLO 5. H aliar la ex tremal en el problema isoper imétrico


sobre el extremo de la lunctonal
1
J IY (x). z (X») = III
(y'2 + Z'2_ 4xz' -42) dx;

y(O)=O, z(O)=O. y(I)=I, 2(1)=1;


con la condición
1
J (y'2-xy' _Z'2) dx= 2.
U
SOLUCION. Formamos la funcional 3uxi liar
1
a>IY(x), z(x)j= J [y'2+Z'Z-4xz'-42+)" (y'2-xy'_z'2») dx
(1

y escribimos para ella el sistema de ecuaciones de Eu ler

- d~ (2y' + 2)..y' - ).x) = O. }

-4 -.!!_ (2z' -4x-21.z') =0'


dx '
.esolviendolo, encontrarnos
S 9. EXTREMO CONDICIONADO 115

Las condiciones de frontera dan


_ 3J..+4.
Ct 2 C2=O, C3=2 {J -A} y

de modo que
J..x2+(31..+4) x }
y= 4 (1 +J..) ,
z=x.
Para determinar 1.. recurrimos a la condición Isopertmétrica (7). Puesto
• 2)..x+3A+4 •
que y = 4 (1 +i..) y l = 1, obtenemos
I
r [(2).X.+3A+4)2 (2A.x+3A.+4)x ,] dx=2.
J 16(1+).)2 4(1+/..)
~
de donde, después de unos cálculos sencillos pero voluminosos, obte-
nemas para i.. la ecuación siguiente:
! (23).2 +46"-+24) = 48 (1.2 +21..+ 1).
De aquí resulta 1.,1 = - :~ y 1..2 = - !~. Introduciendo en (7), vemos

que A.t= - :~ satisface y "2= - ~~ no satisface la condición ;so-


perimé tri ca.
La extrema! buscada se determina por las ecuaciones

y= 7X-;_5X2 , }

z=x.
Hallar las extrernales en los siguientes problemas isope-
rirnétr icos.
166. Entre todas las curvas planas de longitud 1 con
extremos en los puntos fijos Mo (xo• Yo) y MI (xh Yl) hallar
la curva con ordenada mínima del centro de gravedad (pro-
blema sobre fa forma de equilibrio que toma un cable pesado
homogéneo por acción de la gravedad).
,
J67. J{Y(X)l=.l y'2dx; y(O)= 1, y(I)=6; con la
o
1
condición J' y dx = 3.
o
116 CAP. rt. EXTREMO DE FUNCIONALES

168. J [y (X) I= J (X2+y'2)dx; y(O)=O, y(l)=Oj con


o
1

la condición io y2 dx = 2_

I
169. J [y (x)) = Iy''l. dx; Y (O) = 0,
1
Y (l) = '4; con la
o
t

condición ~ (y - y'2) dx = l~ ,
71
20. También es un problema vartactonal de extremo condicionado
el problema de Lagrange en el Que se pide hallar el extremo de la
funcional J [YI' Y'i" ... , Yn] con la particularidad de que se imponen
ciertas condiciones de enlace a las funciones de las cuales depende
la luncional J,
El problema se plantea así. Hallar el extremo de la luncional
XI

JI% Y'l., ••. , Ynl= J F(x. 111. Y2, ... , Un, Yí, y~, .. -, y~)dx; (8)
xo
YJ(XO)=Yio, yj(x1)=YJI (j=1,2 ..... rt);

con las condiciones


<p, (x, Yl. Y2. •••• , Un) =O (9)
(i = 1,2, ... , m; m < n)
que se consideran independientes,
TEOREMA. Las funciones !JI. Y2' •..• Yn que realizan el extremo
de la funcional (8) con las condiciones (9) satisfacen, siempre que los
factores A.l (x) (i = 1, 2, . . .• m) se escojan debidamente, las ecuaciones
de E uler de la funcional

In

1[ + ~
Xi

J. = ¡: "j<Pi J áx •
>'"0 ' ... 1

In
Para abreviar pondremos F+ 2: X¿<p¡=ct>(x, Yt. y'}., ,. -, Yn' YÍ,
i-1
yí. . .. , y~). Entonces las funciones Xl (x) e y¡ (x) se determinan de
S 9. EXTREMO CONDIC10NAUO 117

las ecuaciones de Euler


ID' -~<ll',=o (j=I,2, ... ,n)
IIJ dx 'Ji
'f de las ecuaciones
q>i (x, YI' 112, ••• , y,.) = O (1 = 1,2 •. , " m).
Las ecuaciones !PI = O también se pueden considerar como ecuaciones
de Euler para la funcional J* si se acepta que los argumentos de esta
funcional son tanto las funciones !l1' !/2•. ' ., Ij" como las funciones
A." (z), hl! (x], .•.• Am (x).
EJEMPLO 6. Hallar la distancia mínima entre los puntos
A (1, -1, O) Y B (2, 1, -1) que pertenecen a la superficie 15x-
- 7g + 2 - 22 = O.
SOLUCiÓN. Comosesaben la distancia entre dos puntos A (xo, !lo; 20)
y B (Xl' lit> 21) en la superficie q> (x, y, z) = O se determina por la
fórmula
Xi
1= J 'Vi +y'2.+z'2.dx,
"O
donde y = y (x). % Z (x), =
Se pide. pues, hallar el mínimo de 1 con la condición q> (x, Y. z) =
= O. En nuestro caso,
.lO = 1, X:! = 2. q> (x, y, z) = 15x - 7y +z - 22.
Formamos la funcional auxiliar
2
J. = j lVI +y'2+z'2+),,(x) (15x-7y+z-22)1 dx
1
y escribimos para ésta las ecuaciones de Euler
). (x) (-7)--dd ( 'tí
X
y'
V 1 +y'Z+%'2
) =0. 1 (lO)

d ( Z' ) ( 11)
A.(X}.l--d-x VI+y'2+z'Z =0.
Resolvamos el sistema de ecuaciones (lO) y (11) empleando la condi-
ción de enlace
15x - 7y z - 22 = O, + (12)
Las funciones incógnitas y = y (x) y z = z (x) satisfacen las siguientes
condiciones de frontera:
ti (1) = -1, y (2) = 1, z (1) = O, z (2) = -1. (13)
Multiplicando por 7 la ecuación (11) y agregándola a (10), obtenemos
_:!_ ( y' -1- 7z' ) _ O
dx VI+- tI'~+ z'~ - ,
118 CAP. 11. EXTREMO DE FUNCIONALES

de donde
y'+7z'
( 14)

De (12) tenemos
z' = 7y' - 15. (15)
Introduciendo este valor de 2' en (14) y resolviendo la ecuación dife-
rencial obtenida, encontramos 11 = C1X +
Cz• Las condiciones de
frontera (13) dan e
1 = 2 Y C~ = -3 de modo que
y = 2x - 3. (16)
Teniendo en cuenta (16), de (15) resulta
z= 1 - x (17)
(es obv io que la función (17) satisface las condiciones de frontera).
De (lO) o de (11) obtenemos /,. == O. La distancia buscada es
2

1= J lfl+y'2+2:'Zdx=V6.
1
Este resultado se puede obtener inmediatamente de considera-
ciones geométricas evidentes.
30, Líneas geodésicas. Sea
r = r (u, v) (18)
la ecuación vectorial de una superficie.
Se llama línea geodésica la línea de menor longitud que pertenece
a la superficie considerada y que une dos puntos rijos de la misma.
Las ecuaciones de las líneas geodésicas se pueden obtener como
las ecuaciones de Euler correspondientes al problema v ar iacional
sobre 1 a distancia mínima en la superficie entre dos puntos rijos:
Toda línea perteneciente a la superficie r = r (u. v) se puede
representar por 1as ecuaciones paramétrícas
u = u (/), ti --=: ti (t).
La longitud de su parte comprendida entre los puntos correspondientes
a los valores lo Y t1 del parámetro t es igual a

J[u(l), v(t)[= J VEU'2+2Fu'v'-I-Gv'2dt, (19)
!(l

donde E, F y G son Jos coef lcien tes de la primera forma cuadrática


de la superficie (lB), o sea,
E
=
(orou' Tu'
a,· ) F
=
((Ir
au'
Qr )
Tu ' G= (.El:...
ou' ~).av
Aquí (a, b) es el producto escalar de los vectores a y b .
§ 9. t::XTREMO CONDICIONADO 1I9

Para la funcional (19) el sistema de ecuaciones de Euler tiene


la forma
EuU'3 +2Fuu'v' Guv'~ +d 2 (Eu' +Fv') =0
VEu'·¿+2Fu't.l-; Gv'2-·dt·VEu'Lt-2Fu't)'~Gv'l!. '
Evu'LJ2Fvu'r./+Gvu'll _~ 2(Fu'··J··Gv') =0.
VEu''J.+2Fu'u'.+Gv'2 dt VEu'?.+2Fu'v'+Gl.'2
EJEMT'LO 7. Entre todas las curvas que están sobre una superficie
esférica de radío R y que unen dos puntos fijos de la misma. hallar la
curva de longitud mínima (la curva ~codésica).
SOLUCION. Sean (jJ y O las coordenadas del punto en la esfera
y sea q> -"c q> (e) la ecuación de la CUTva pedida. Tenemos entonces
r = r (<p. e) = x (ep. e) 1 + y (ep. e) j + Z (If. fl) /l,
donde
x ,...,.R cos <p sen S, s > R sen {f' sen e.
z = R cos e.
Por eso,
E = (rlp' r~) = Rf. sen2 O; (J = (re, rij) = R.2, F = (re. rq¡) = O.
De aquí. según la fórmula (19). tenemos
91 el
J[<p(6)1=R. ~ Vd6z+sen ijacp'¿=R
ll J VI + sen 6q>'z(O)
2 d6.
eo 60
El integrando no conttene la función incógnita q¡ (O) y, por eso, la
ecuación de Euler será

de modo que

VI + seoS 6q>'2 (9)


dedonde
<p' (e) = C1
sen6Vsen26-q

CI
sen!a 'v 1--- q
senzO
Cid ~etgO)
sen2 9 V (1- Cn- q ctg2 e - V (1 - cn-
q ctg* o'
Integrando, obtenemos
el etg e
q¡ (6) = arccos .. r
y I-q
+ C?,
120 CAP. 1I EXTREMO DE FUNCIONALES

ó
cp (e) = arccos (e ctg 9} + c2, donde

De aquí
e ctg a= cos (q> (9)- e:!)
ó
ctg e = A cos cp (O) +8 sen ep (O), (20)
donde

8 = sene2
e .
Multiplicando por R sen a ambos miembros de (20). obtenemos
R cos B = ARcos ep sen e + B R sen <p sen e
o. pasando a las coordenadas cartesianas.
z = Ax + By.
Esta es la ecuación de un plano que pasa por el centro de la esfera
y que corta su superficie según un circulo máximo. Por consiguiente,
1 a línea más corta (línea geodésica) es el arco del circulo máximo.
EJEMPLO 8. Demostrar que para una superficie de revolución
es constante en cada uno de los puntos de las geodésicas el producto
del radio del paralelo por el seno del ángulo entre la geodésica yel
meridiano (teorema de Clairaut).
SOLUCION. En coordenadas cilíndricas la ecuación de una super-
ficie de revolucion tiene la forma
x = p cos ep, y = p sen «P, z = f (p).
Determinemos los coeficientes E, F y O:
E = 1 (2, +F = O, G = p2.
. Por eso, la dilerenclal ds de la longitud de arco en +a superficie de
revolución tiene la forma
ds= V p2+(1 + ,~.)pf2 dcp.
En la superficie de revolución las líneas geodésicas serán extremales
de la funcional
11>1

) V p'+(1 +f~9)p'Zdcp.
qJo
Como la función integrando no contiene explícitamente «p. obtenemos
lnrnedia tarnen te
p~
const,
V
p2+(1 +fp'} pfi
§ 10. PROBLEMAS CON FRONTERAS M6vll_ES 121

o sea, p9 ~~ =const. Observando que p :; =senw (fig. 18), obte-


nemos p sen o ee const que es lo que se queda demostrar.
170. Hallar
la distancia más corta entre los puntos
A (l. O. -1) Y B (O,-1, 1) en la superficie x y -1- z = O. +
171. Hallar las líneas geo-
désicas del cilindro circular
r = R.

§ 10. Problemas vartacionales


con fronteras móviles
1°, Problema elemental con fron-
teras móvlles. Sea F = F (x, y, U') una
función diferenciable tres veces respec-
to a sus argumentos y sean
11 = q> (x) e U = IV (x) (1)
donde p (x) E CI fa, b] Y - '\ji (x) E
E CI la, b], dos curvas en el plano xOU.
Consideremos la funcional

J [U (x)} = J F (x, y, g') dx (2)


fig. 18
'V

definida para las curvas suaves U = 11 (x) cuyos ex tremos A (xo, Yo)
y B (Xl. 111) se encuentran en las curvas (1) de modo que 110 = <p (xo)
e UI = W (XI)' Se pide hallar el extremo de la funcional (2).
TEOREMA. Supongamos que en la curva Yo : Y = (x) se alcanza Y
el extremo de la funcional
J (y (x)! = J
y
F (x, y, y') dx

entre lodas la curvas de la clase el que unen dos puntos arbitrarios de


dos curvas fijas y = <p (x) e y = W (x). Entonces la curva 'Yo es una
extremal y en los extremos A (xo. Yo) Y B (Xl' Yl) de la curva 'Yo se cumplen
las condiciones de transversalidad
[F+(Ip'-t}FII,¡ 1;0:_:0:0=0, }
(3)
[F+('IjJ' -1)
Fy,J Ix~;q =0.
Es decir, para resolver el problema elemental con lronteras
móviles es preciso:
1) Escribir y resolver la ecuación de Euler correspondiente.
Como resultado, se obtiene una Iamtlla de extremales !I = f (x, el. C3),
que depende de dos parámetros el y C3•
122 C,\P. 11. E XTREMO DE FUNCIONALES

2) Determinar las constantes CI, Cz• Xo y Xl de las condiciones


de transversalidad (3) 'j de las ecuaciones
{(xo, e 10 C2) = q¡ (xo), }
(4)
{(XI, C¡, C2,)=\II(Xt).
3) Calcular el extr emo de la funcional (2).
EJEMPLOl. Hallar la condición de transversalídad para la fun-
cional
'q
J (y (x)1 = Jf (x. y) earctg 1/' V I -1- g'~ dx, f (x, y) -* O.
XI>

SOLUCIÓN. Supongamos que el extremo de la izquierda de la extre-


mal se ha fijado en el punto A (xo. Yo) mientras que el extremo de la
derecha B (Xl> YI) puede desplazarse por una curva y = lj1 (x). Tendre-
mos entonces
[f "j- ('IjJ' - g') F 11,1Ix_xI = O.
En nuestro caso es
1 -+-y'
F = f (x. y) e~\rctg ,¡' V 1 + y'Z y F 1/' = l (x. y} (;'arc.tg 1/' -~'-"-_
Vl+y'~
r
La condición de transversalidad se representa así
f (x, y) earctg 1/' V 1-1- y'2 +
+W-y')/(x, y) eflrctg 1/'
VI +y ~
I-i-Y', ] I
x=X¡
=0.

De aquí obtenemos, debido a la condición f (x, y) =1= O,


'I>'-y' -l. (5)
I+1j¡'y'
Desde el punto de vista geométrico, Ia condición (5) , lgniüca que las
extremares y = y (x) deben cortar la curva y = ~ (x) por la cual
se desplaza el punto extremo B (XI> yd de modo que el ángulo entre
n
estas curvas sea de T'
Efectivamente, la relación (5) se puede transformar del modo
siguiente: supongamos que la tangente a la extremal en el punto
8 (Xl. Yl). que pertenece a la curva y = 'IJl (x), Iorrna ángulo a. con
el Ox y que la tangente a la curva rija y = q¡ (x) forma ángulo ~
(fig. 19); entonces, se tiene tg a. = y', tg ~ = '11>' y el primer miembro
de la Iórrnula (5) da tg (~ - 0.); pero como -1 = tg ( - : ), resulta

~ - a. = - : ' de donde a. = ~ + ~ que es lo Que se quería demos-


trar.
§lO. PROBLEMAS CON FRONTERAS MOVILES 123

EJEMPLO 2. Hallar la distancia de la parábola y = x2 a la recta


x-y = 6.
SOLUCION. El problema consiste en hallar el valor extremo de la
integral
:11:1

J ry (x)) = J V 1 +y'Z dx
Xi)

con la condición de que el extremo de la izquierda de la extrema! se


puede desplazar por la curva y = Xl mientras que el extremo de la

Fig. 19

derecha, por la recta y = x - 5. Por consiguiente, tenemos en este


caso <p (x) = X2 y ljl (x) = x - 5. La solución general de la ecuación
de Euler será y = e1x + e2, donde el y el son constantes arbitrarias
que deben ser determinadas.
Las condiciones de transversalidad (3) tienen la forma

donde y' = Cl' Las ecuaciones (4) lienen en nuestro caso la forma

C1XO+e2=X~, }
CtXt+Cz=xl-5.
124 CAP, n. I?XTREMO DE FUNCIONALES

Tenemos, pues, un sistema de cuatro ecuaciones con cuatro incógni-


tas Cl• C2• Xo Y Xl:

VI+C:+(2xo-Ct) -.í
v I+C:
el =0, 1
VI +C:+(i-Ct),í v l+q
C. =0, j
CtXo+C2=X~,
CtXt+Cz=Xt-5;
resol viéndolo, obtenemos
3 I 23
Ct=-l. C2=4' xO="2 y x(=g'

Es decir, la ecuación de la extrema! es y = -x + !y la distancia


de la parábola a la recta es igual a
2.3
T 23

l= J V +<-I)ldX=V !:
1 2X =19y'2.
1 í
2'
{72. Hallar la distancia más corta del punto A (1, O)
a la elipse 4X2 + 9y'i = 36.
173. Hallar la distancia más corta del punto A (-1. 5)
a la parábola y2 = x.
174. Hallar la distancia más corta de la circunferencia
X2 + y2 = 1 a la recta x y = 4. +
175. Hallar la distancia más corta del punto A (~1, 3)
a la recta y = 1 - 3x.
176. Demostrar que en el caso de la funcional
«'1

Jfy(x)J=) h(x, y)V1+y'2dx,

donde h (x, y) '*


O en los puntos frontera, las condiciones
de transversalidad tienen la forma
y' (x) = - epll(X) e y' (x) = - 1j),I(X) ,

o sea, las condiciones de transversalldad se reducen a las


condiciones de ortogonalidad.
S JO. PROBLEMAS CON FRONTERAS MOVILES 125

2°, Problemas con fronteras móviles para funcionales de la forma

1
:el
J [y (x). z (x)] = F (x, v. z, !J', z') dx. (6)
:ro
Al analizar el extremo de la funcional (6) aceptamos que por lo
menos uno de los puntos frontera A (xo. Yo. %0) o B (Xl. Yl. 21) se
desplaza por una curva fija.
El extremo de J [y (x), z (x)1 se puede alcanzar sólo en las curvas
integrales del sistema de ecuaciones de Euler

F....--F
dx 11
,=0,
d }
Ft-~F.dx :t
=0.

Supongamos que el punto A (xo. Yo, %0) está lijo mientras que el
otro punto frontera B (XI, !/l. 21) se. desplaza por una curva definida
mediante las ecuaciones
y=q>{x), }
z ='Ijl (x).
En este caso, la condición de transuersalidad tiene la forma

[F+(q>' -!J') FII ,+ (1jJ'-z') Fz.11:t_:t1=0.


Análogamente se escribe la condición de transversa1!dad para el
extremo de la izquierda (si éste también se desplaza por una

EJEMPLO 3. Hallar la distancia más corta del punto M (xo. Yo, 20)
a la recta
y=mx+p, }
z =nx+q.
SOLUCIONo El problema se reduce a la determinación del extremo
(mínimo) de la integral
~1
J Ig (x), Z {x)J= J VI
%0
+ g':l+z'2dx (7)
\26 CAP. 11. EXTaEMO DE FUNCIONALES

con la condición de que el extremo de la derecha de la extremal puede


desplazarse por l a recia
y=mx+p, }
(8)
z=nx+q.
o sea, en nuestro caso las funciones rp y IP tienen, respectivamente.
la forma
rp (x) = mx +p y 'i1 (x) = nx + q.

La solución general del correspondiente sistema de ecuaciones de


Euler será
y=G1X+G2, }
(9)
Z=G3X.+C~,
donde e¡ (i = l. 2. 3 Y 4) deben ser determinadas.
La condición de transversalidad (en el extremo de la derecha)
tiene la forma

[ VI +y'3+z'2+(m-y') VI+y't+z'~
y' +
+(n-z') Z'
VI +y'2+ z'Z
] I
X-XI
=0,

de donde, puesto que y' = el y z' = Gil' obtenernos


I -1- mC} + /lC:! = O. (lO)
La relación (10) expresa la condición de perpendicularidad
la entre
recta buscada (9) y la recta dada (8).
Empleemos el hecho de que la recta buscada (9) pasa por el punto
M (xo. Yo, zo):
(11)
y también el hecho de que e 1 extremo de la derecha se desplaza por
la recta (8):
elXl+e2=mXl+P. } (12)
C3Xl +C,=nxd-q·
De las cinco ecuaciones (Iü, (11) Y (12) debemos determinar CJ. C.,
Ca. e, y Xl (.to, Yo. 2o. m. n, p y q son números dados). Para calcular
la integral (7) basta conocer Xl. el y C,. Tenemos
x _ ;_;xo::..,+..:......_m_{:;y,::,o
---=p,-!-,},....:.+_n"...(.:....~~o
-~q)
1- l+nZ+mZ t

C _ mxo+m/l (2'0-9)-(1+n2) (yo-p)


t- m (yo-p) +n (zo-q) -(m2+n2) Xo t
e _ rlXo+mn(Yo-p)-(l+mZ)(zo-q)
3 - m (Yo-p)+n(2o-q)-(m2+nZ}Xo •
§ 10. P~OBLEMAS CON FRONTERAS MOVIlES 127

Introduciendo estos valores en (7), obtenemos


h = mín J (y (x), z (x)l =
-V/'
-
x +(y -p)z+(z
.o
2
o o
_ q)2_ rxo+m(!lo-p)+n{zo-Q»)2
1 + n2 + m2 •

Si el punto frontera A (xo, Yo, 20) está fijo y el otro punto lronter a
B ('~1' YI' Zl) pue-de desplazarse por una superficie Z ~..::.<p (x, y), las
condiciones de lransoersalidod serán
(F -11' Fu,+(q¡~-Z') F~,.lLI:;;xl =_0, }
(13)
IF!J' + F~'CfllJ 1:x_xl - O.
Las condiciones (13) conjuntamente con la ecuación 2 = <p (x, y)
permiten determinar, hablando en términos generales, dos constantes
arbitrarias de la solución general del sistema de ecuaciones de Euler
(las otras dos constantes se determinan de la cond ición de que la
extremal ha de pasar por el punto fijo A (xo, Yo, 20»'
Si el punto móvil es el punto frontera A (xo, Yo, 20), obtenemos
para x ;-:. Xo unas condiciones análogas completamente a las condi-
ciones (13).
EJEMPLO 4. Hallar la distancia más corta del punto A (1, 1, 1)
a la superficie esférica
X2 + + = y2 Z2 J. (14)
SOLUCION.El problema consiste en analizar el extremo de la
lunclonal
1
J[y(.~), z(x)J=
XI
IV 1+!I'2+z'2dx. (15)

donde el punto B (XI> fll. 21) debe estar en la superficie esíértca (14).
Las ex tremales de la funcional (15) son las recias
y=Ctx+C2, }
(16)
z =C3x+C(o.
De la condición de que la extr emal (16) pase por el punto A (1,.1, 1),
obtenemos
(17}

tienen la lorma
128 CAP. n. EXT~EMO DE FUNI,;(ONALES

teniendo en cuenta (16), después de unos cálculos sencillos encontra-


mos de aquí
ZI-CaXt=O, }
CtZt-C3I1t=O. (18)
donde Xl> 111 'i Zl son las coordenadas del punto buscado B.
De la condición de que la extremal (16) pasa por el punto
B {Xl' 1/1' %1) tenemos
Yl = C¡X¡+Cz, }
ZI =C3X¡+C4• (19)
De (17), (18) Y (19) encontramos
el = 1, C,,= O. C3=
de modo que la ecuación de la extremal es
y=x. }
(20)
z=x.
Puesto que el punto 8 (XI. 111. Zl) debe estar en la superficie esférica
(14), obtenemos, tomando en consideración (20). que x~ + x~ xl = +
= 1, o sea. que XI = ± va .
1

Por consiguiente, obtenemos dos puntos


B (_1 _J _1) B¿ (
I I
-V'j' -'V'3' -V3 .
1 )
I V3' V3' ~í3 Y
Es fácil ver, por razones geométricas, que en la extremal (20) que
une los puntos A y Bl la funcional (15) alcanza su mínimo igual a
1

Jm1n= J VI+I+ldx=V3-1
1
Va
mientras que en la extremal (20) que une los puntos A y B~ esta fun-
cional alcanza su máximo
1
Jméx= \ lf3dx= V3+ l.
•1
- 113
OBSERVACiÓN l. Al deducir las condiciones de transversalidad (18)
hemos considerado que <p (x, g) == VI - x2 - 1/2. Es fácil ver que
las condiciones (18) subsisten si q> (x, y) == - VI _ x'l _ y2.
OBSERVACIÓN 2. Queda claro, por razones geométricas, que la
extremal (20) es ortogonal a la superficie esférica x2 + y'J + i" = 1.
10. PROBLEMAS CON FRONTERAS MóVI LES 129

EJEMPLO 5. Consideremos el mismo problema sobre el extremo


de la funcional (15) pero tomando como A el centro de la esfera
O (O, O, O).
SOLUCION. Las ex íremales de la funcional son las rectas (16) y la
condición de que la extrernal pase por el punto O (O, O, O) da inme-
diatamente C~ = C, = O.
Las condiciones de transversalidad serán las mismas
Zl-C3Xl=O, }
(21)
CiZt-C3!/[ =0,
y las condiciones en el extremo móvil serán
Jlt =CIXt, }
(22)
z1 =CaXl'
Por último,
(23)
Para determinar las cinco magnitudes Cl• C" Xl. Yl Y z. tenemos
cinco relaciones (21), (22) Y (23) de las cuales solo tres son indepen-
clientes:

(24)

Empleando las relaciones (24), encontramos


l 1
x1=±1(1+c:+q' Y¡=±Y'I+q+q'
1
zl = ± -::i'~~¡¡:::::::::~
y I-I-Cl+q'
donde Cl y Ca son unas constantes arbitrarias.
Consideraciones geométricas aclaran esta arbitrariedad: la distan-
cia del punto O (O, O. O) a la superficie esférica (14) es la misma en
cualquier dirección, o sea, para cualesquiera valores de Cl y Cs.
El valor de la funcional J [y (x), z (x)] en las extremales
!/=Ctx. }
z=Csx
es igual a

Vl+chci
J !y (x),Z (x)l= ~ VI +ct+qdx= l.
'O
9-01381
130 CAP. 11. EXTRflMO OS f1UNCJONALE.S

EJ.EMPLO 6. Hallar la condición de transversalidad para la íun-


cional

Jf
%1

J[V(x). z(x)J= (x. 11. z)"VI +y'2.+z'2dx, (25)

si el punto A (xo. !lo, to) está fijo y el punto B (Xl. !JI, Zt) se encuentra
en la superficie z = q> (x, y).
SOLUCIÓN. En este caso las condiciones de transversalidad serán

(1+q>~z/)I'-'~1==O, }
(y' +
q>Íf2')I;a:..Z1=0,
o sea,
I 1
q>; ,.... «1 = q>v
y' I :r.=o""1=-=1
~ 1«-XI'
Representan la condición de paralelismo del vector T {l. y', z'}
tangente en el punto B (x .. (1' %1) a la extremal buscada y del vector
11 {q>;, <p~, -1 } de la norma a la superficie z = q> (x, 11) en este mismo
punto. Por consiguiente. para las funcionales de la forma (25) las
condícícnes de transversalidad se reducen a las condiciones de ortogo-
h'alidad.
177. Demostrar que, si la condición de transversalidad
y la condición de ortogonalidad coinciden para todos los
datos iniciales, la función integrando F tiene la estructura
siguiente: •
F = ¡(x, Y, z) VI y'2+Z'2, +
donde f (x. y, z) es una función diferenclable cualquiera de
x, y y z.
t78. Hallar la distancia más corta del punto M (0,0, 3)
a la superficie z = Xl + y'¡.
t79. Hallar la distancia más corta del punto M (2, 0, 5)
a la superficie z = XZ yz. +
180. Hallar la distancia más corta entre las superficies

~+ r; +~= 1 Y x2+UZ+z2=4.
181. Analizar el extremo de la funcional

J fy (x)J Z (x)l =
~.
J (y'2+z':&+2yz) dx
o
S 10. PROBLEMAS CON PRoNtERAs MO\lI LES 131

si y (O) = z (O) = O Y el punto B (Xl. Yl. 21) se desplaza por


el plano X = Xl'
3°, Distancia geodésica. El valor de la integral
B
J [y (x)) = .~F (x, y, y') dx, (26)
Á

ca1culada según una linea T


desde el punto A hasta el punto B, se
denomina !-longitud de la ínea y. Si y es una extremal se dice que
J [[1 (x)] es la distancia geodésica entre los puntos A y B, o simplemente
J·distancla, y la propia extrernal se demomina J-recta.
EJEMPLO 7. Hallar la distancia geodésica del punto A (O, O) al
punto B (1, 1) si esta distancia se define mediante la funcional
B
J [y (x)) = J
A
g2,/,'J. dx,

SOLUCION. La distancia geodésica del punto A al punto B es Igual


al valor de esta funcional en la extremar que une dichos puntos. La
ecuación de Euler es

2yy'Z - :x (2YZy') =0 6 yyo" +y'2= O.


Es Hici1 ver que
d
l/JI" + {J'! = '7lX (yy'),

de modo que 2yy' = CI e ya = C1x +


C:a. Empleando las condiciones
de frontera y I:r=o = O e !l1:':=1 = 1, obtenemos el = 1 Y C. = O.
Por consiguiente, la extremal que une los puntos A y B es la parábola
ya = x.
Tenemos ahora 2yy' = 1, yy' = ~lI. por consiguiente, (yy')2 = !.
Por definición, la distancia geodésica entre los puntos A y B es igual a
1-

1 (A. B)= J ! !.
O
dx=

Supongamos que se tiene una línea Z: q> (x, y) = O.


La distancia geodésica entre un punto B que no pertenece a Z
y esta linea se define como la distancia geodésica del punto B a un
punto A E Z que se obtiene calculando la funcional (26) según la
extremal 'i que une los puntos B y A con la particularidad de que y
corta transversalmente la línea :c
en el punto A.
9*
CAP. 11. EXTREMÓ DE FUNélONAL2S

Se denomina ¡-circunferencia (o circunferencia geodtsica) la línea


formada por los puntos que están a una misma distancia geodésica
de un punto rijo. Análogamente se definen los conceptos de ¡-elipse
y de J-bipérbola,
EJEMPLO 8. Hallar la ¡·circunferencia de radio R y con centro
en el punto O (O, O) si la distancia geodésica se define mediante la
funcional
B
J [U (x)] = J y2/1'2 dx,
A.

SOLUCION. Las extremales de la funcional cortan transversal-


mente la clrcunlerenclegecdésíca. Para las extremales tenemos (véase
el ejemplo anterior)
y. = el". 2yy' = el
y. por consiguiente.
fJ'_ y
" _ 2:c •

De la condición de transversalidad
y'y' (2ep' - yr) = O
encontramos Que el coeficiente angular de la tangente a la ¡·circun·
ferencia es ep' = U; y, por eso, la ecuación diferencial de la ¡·circun-
íerencía es y' = ¿ . de donde resulta la ecuación de la ¡-circunfe-
rencia: u' = Cx. Para determinar el valor de e observemos que el
punto (CS, C) está en la circunferencia geodésica y que la ecuación de]
radIo geodésico (o sea, de la extremal) que pasa por dicho punto es
tf = ~ . De aquí tenemos !lU' =~ y, por lo tanto,

es es
R= j (lIy')2dx=
o
Jo 4~2 dx= ~ •

Es decir, C = 4R y la ecuación de la circunferencia geodésica de ra-


dio R y con centro en el origen de coordenadas es ya = 4Rx.
EJEMPLO 9. Hallar la ¡·circunferencia de radio R y con centro en
el runto O (O, O) si la distancia geodésica se define mediante la funcio-
na
B
Jry(x)J=
A
J lfl+!l dx.2
§ 10. PROBLEMAS CON FRONTERAS MÓVILES 133

SOLUCION. Las extremales de la funcional son las rectas y =


= CIx + C,. De la condición de que la extremal deba pasar por el
punto O (O, O) encontramos Ce = O de modo que y = C1x y, por consi-
guiente, y' = .!L •
x
La condtcíón de lransversalldad coincide en este caso con la
condición de ortogonalidad y, por eso, el coeílcíente angular de la
tangente a J-circunferencia es -q>' = - :' . Por consiguiente, la

ecuación diferencial de la J-círcuníencía es y' = _..:...


y
De aquí resul ta
la ecuación de la ¡-circunferencia: XII + y'l = Cll• El punto (C, O)
está en dicha circunferencia. La ecuación del radio geodésico que pasa
por este punto es y = O de modo que y' = O y
e
R= ) dx=C.
o
Es decir, C = R y la ecuación de la circunferencia geodésica buscada
de radio R es la ecuación de la circuníerencia corriente ;(2 + y2 = Rll.
OBSERVACION. Los conceptos introducidos permiten hablar de la
Geometría no euclídea con la diferencial de arco
ds = F (x, y, g') d:c.
Si F = VI + y'2, las J -rectas se convierten, como hemos vlsto, en
las rectas corrientes y nuestra Geometría se convierte en la euclídea
corriente.
Si F es una {unción arbitraria, que s610 satlsíace las condiciones
habituales de ser continua y derivable respecto a los tres argumentos,
la Geometría construida muy poco recuerda Ia corriente: no siempre
se puede trazar una J -recta por dos puntos y puede suceder que por
dos puntos pasen varias J -rectas y, por consiguiente, que la J -dlstancta
entre dos puntos no sea una función unívoca de las coordenadas.
182. Hallar la distancia geodésica del punto A (O,O)
al punto B (1, 2) si esta distancia se define mediante la
funcional
J(y(x)J=) (!f+ y'2.) dx.
183. Hallar la distancia geodésica del punto A (O, 1) al
punto B (1, 1)si esta distancia se define mediante la funcional

J [y (x)) = ) (l2xy+ y'2) dx,


184. Hallar la J-circunferencia de radio R = 8 y con
centro en el punto O (O, O) si la distancia geodésica se define
134 CAP. 11. EXTREMO DE. FUNCIONALES

mediante la funcional

J [y (x)] = J y'3 dx.

§ t 1. Problemas discontinuos. Variaciones unilaterales

1°. Problemas discontinuos. La extrema! y=y (x) de la funcional


:<1

J lb' (x)l = j F (x, y. !I) dx (1)


Xo

es una función que tiene dos derivadas continuas siempre que la deri-
vada F V'II' (x, y (x), y' (x» sea diferente de cero. Sin embargo, existen
problemas variacionales en los cuales el extremo se alcanza en una
curva suave a trozos solamente.
o) PROBLEMAS DISCONTINUOS DE PRIMERA ESPECIE. Consideremos
el problema sobre la determinación del extremo de la funcional (1)
aceptando que las curvas admisibles satisfacen las condiciones de
frontera
y (xo) = Yo, y (Xl) = Yl (2)
y pueden tener un punto angular en el punto de abscisa e (xo < e Xl)' <
Este punto angular puede darse 5610 allí donde F 11'11' = O (véase el
teorema 2 de la pág. 53). En el punto angular la extrema] debe satisfa-
cer las condiciones de Weierstrass - Erdmann

FlI,I:l:=c-o-FII,I:l:=c+o=O, }
(3)
(F - ti' FII,) /x ... c-o-(F- y' FV'} 1:1:""'4)+0=0.
Conjuntamente con las condiciones de continuidad de la extremal
buscada, estas condiciones permiten determinar las coordenadas del
punto angular.
En cada uno de los segmentos [xo, el y [e, Xl! la extremal debe
satisfacer la ecuación de Euler, o sea, una ecuación diferencial de
segundo orden. Al resolver estas dos ecuaciones se obtienen cuatro
constantes arbítrarias que, hablando en términos generales, se deter-
minan de las condiciones de Irontera (2) y de las condiciones (3) en el
punto angular.
EJEMPLO J. Hallar las extremales quebradas (si es que existen)
de la funcional
a
J Iv(x)J= J (y'2_ y2) dx,
11
§ n. P~OBLEMAS DISCONTINUOS 1.35

SOLUCION. Escribimos la primera de las condiciones (3) que deben


cumplirse en el punto angular:

F 11' I:.:"",c-o = Fu' b:=e+o (O <:: e < a).


En nuestro caso tiene la forma
y' (e - O) = y' {e O) +
= c. Por conslguíen-
Y significa Que la derivada y' (x) es continua en x
le. no hay puntos angulares. Esto se puede ver también de que en
nuestro caso F 1/'V' = 2 > O en todo punto. Por lo tanto. en el proble-
ma considerado ~~ extremo puede alcanzarse sólo en curvas suaves.
EJEMPLO 2. Hallar las extremaJes quebradas de la funcional
2
Jly(x}j= S (y'4_6y'2}dx; y(O)=O. y(2)=0;
O
aceptando que y' puede Ser discontinua en el punto correspondiente
a la abscisa x = c.
SOLUCIONoEn este caso F11''''' = 12y'2- 12 se puede anular, y, por
eso, puede ocurrir Que la extremal tenga puntos angulares. Puesto
que la Iuncíén integrando depende s610 de y', las extrernales son las
rectas

Pongamos
y_ = mx + 11 (O X -< <
e) e U. = p» q (e x ~ 2). + -<
De las condiciones de frontera encontramos n = O y q = -2p de
modo que
y_ = mx e 11+ = P (x - 2).
La condición de continuidad de ta extremal da
me =p (e - 2). (4)
Escribamos las condiciones de Welerstrass - Erdmann. Tenemos
r",,=4y'S-12y' }
F-y'FII,= -3y"+6y'2.
Puesto que y:" = m e y: = p, obtenemos
4m3-12m=4p3-12p, }
-3m'+6m2= -3p4+6p2,
o sea.
(m-p) (m2+mp+p2-3)=O, }
(5)
(m2-p2) (m2+p2-2~=O.
136 CAP. 11. EXTREMO DE FUNCIONALES

La segunda de las ecuaciones (5) da inmediatamente m = p. m = -p o


m" + pZ -2 = O.
La solución m = p debe ser excluida: en este caso la extrema! tiene
derivada continua y de la condición (4) obtenemos m = O, o sea, la
extremal es un segmento de) eje Ox.
Por consiguiente. para resolver el sistema (5) hay que resolver
dos sistemas de ecuaciones:
m=-p. } 6)
~+~+~=3 (
y
m2+p2=2, }
(1)
m2+mp+p2=3.
La solución del sistema (6) es: m=V3, p=- y In = - V3, 113
p = V3. La solución de! sistema (7) es m = p y debe ser excluida.
Es decir, m::: -p y la condición de continuidad (4) da e ,= 1.
Por consiguiente, las extrernales buscadas son:

y= {
V3x,
-]Í3 (x-2),
e
-vax, O~x< 1,
g= { V3 (x - 2). 1 '" x < 2.
185. Hallar las extremales con punto angular para la
funcional
2
J (y (x)] = 1o y'2 (y' - 1)2 dx; y (O) = O, y (2) = 1.

186. Hallar la solución c.on un punto angular en el pro-


blema sobre el mínimo de la funcional
4
J(y(x)] = J (y' _1)2. (y' + 1)2 dx; y (O) = O, y (4) = 2.
o
187. ¿Existen soluciones con puntos angulares en el
problema sobre el extremo de la funcional
Xl

J [y (x) 1 = J (51'2+ 2xy- y2) dx; y (xo) = Yo, Y (Xl) = Yi?


xg
~ 11. PROBLEMAS DISCONTINUOS 137

188. Hallar la solución con punto angular en el problema


sobre el extremo de la funcional
I
J{Y(X)I=) y2(l_y'2)dx; y(-I)=.O, y(I)=l.
-1

189. Hallar la solución con punto angular en el problema


sobre el mínimo de la funcional

f
;le.

(y" - 2y'2) dx.


%1

190. En el problema sobre el extremo de la funcional

(""r 1/1)
J sen y' dx
(o. O)
hallar la solución continua y la solución con punto angular.
OBSERVACION. Las condiciones (3) de Weierslrass-Erdmann
admiten la siguiente interpretacíón geométrica.
Consideremos la fíguratriz o sea. la curva Y = F (x, y, y') en
tanto que función de y'.
Las condiciones (3) signHican entonces que para los valores de los
parámetros x = e y y = el, que corresponden al punto angular, la
figuralriz debe tener una misma tangente en los puntos de abscisas
y:" = y' (e - O) e y.j. = !J' (e O). +
Al mismo tiempo se obtiene una Interpretación clara de la condí-
ción FU'I/' =¡60 que excluye la posibilidad de puntos angulares en las
extrernales. Efectivamente. si, por ejemplo, es F I/'Y' > O, la fíguratriz
es cóncava hada las Y positivas y no podrán coincidir sus tangentes
trazadas en dos puntos distintos. Es decir, en este caso la extremal
no podrá tener punto angular.
Consideremos de nuevo el problema sobre la determinación de las
ex trema les quebradas de la funcional del ejemplo 2 de este parágrafo.
Tenemos
2
J!U (x)] = J
O
(y'"_6y'2) dx; U(O)=O, y(2)=O.

Las extremales son rectas. En este caso la Iiguratr iz y = y'. -


- GIf'2 no depende del punto (x, y). Tiene una tangente común en
138 CAP. JI. EXTREMO DE FUNCIONALES

los puntos de abscisas y' = ± V3 (fig. 20). Por eso, las condiciones
de Weierstrass-Erdmann quedarán cumplidas si como extrernales

Fig. 20

quebradas tomamos las quebradas cuyos lados formen ángulos de


± ~ con el eje Ox.
En la quebrada !11 con un punto angular (Hg. 21) la funcional
toma el valor J {ud = -18. Este mismo valor tendrá Jiu (x}) en la

Fig. 21

quebrada Y'J con dos puntos angulares, (Hg. 22), en la quebrada Ya


con tres puntos (lig. 23), etc.
b) PROBLEMASDISCONTINUOS DE SBGUNDA eSPECIB. Se denominan
problemas discontinuos de segunda especie los problemas sobre el extre-
S 11. PROBLEMAS DlSCONTI NUOS 139

mo de la funcional
XI
1 [(x») = J F(x, y, y')dx; Y (X¡)=Yf. y (X2)=Y2; (8)
;ll:f

en la que la funci6n Integrando es discontinua.


Supongamos, por ejemplo. que F [x, Y. y') es discontinua a lo
largo de la curva y = <!> (x) y sea F (x, y, g') igual a F1 (x, y, g')
a un lado de la linea y = dl (x) e Igual a F, (x, y, y') al otro lado.

2 x

Fig. 22

SI existe la extremal quebrada, deb.erá componerse de los trozos


de las extremares y = Yl (X) e !I = y, (x) que tienen un punto común
(c. <!> (e», e E (Xl. XI)' en la linea de discontinuidad. Para hallar la

2 x

Flg. 23

extrema! quebrada obtenemos dos ecuaciones diferenciales de Euler


cuyas soluciones generales contienen cuatro constantes arbitrarias Cl•
Cg• C, y C,. Para determinar estas constantes así como la abscisa e
del punto en el que la extremal encuentra la curva y = CD(x) tenemos:
1) dos condiciones de fronteras (8), 2) dos condiciones según las cuales
las ordenadas de los extremos de las ex trernales en' el punto de [unción
han de ser iguales a la ordenada de la curva.y = CD(x) y, por último,
3) la condición de la ¡un.ci6n ..

Fi +(q.' -!/) Fi¡¡' IlC"C-O""".F2.+(<D' - y') !'21!' 1~=e+O.. {9~


140 CAP< 11. EXTREMO DE FUNCIONALES

Hablando en términos generales, estas condiciones alcanzan para


determinar la extremal quebrada.
EJEMPLO 3 (problema de la refracción de un rayo de luz). La luz
se propaga con una velocidad constante Ell en el medio 1 y con una
velocidad constante VII en el medio 11. La curva y = (l) (x) separa los
medios 1 y II.
Deducir la ley. de refraccl6n del rayo de luz que va desde el punto
A del medio' 1 hasta el punto B del medIo 11 si se sabe que es mínímo
el tiempo durante el cual el rayo de luz recorre este camino.
SOLUCIONo El .problema consiste en hallar el mínimo de la inetgral

J [y (x»)= r Vf+Y'2
J 01
dx+ Jb v¡-::¡:Y;Z
V2
dx, (10)
(l e

ya que la primera y la segunda integrales de (10) representan, respecti-


vamente, el tiempo que necesita el rayo de luz para llegar del punto A
a la linea de separación y de la linea de separación al punto B.
Tenemos un problenia discontinuo de segunda especie siendo

Para determinar los trozos de las extremales debemos hallar las extre-
males de la funcional

JV 1 +1I'zdx
que, como se sabe, son rectas. Por consiguiente
Y1=mx+n e U2=P;<+q.
Escribamos la condición (9). Tenemos

F • aF 1 11¡-::¡::fiii gi'
1-91 a9~ = UI. t1tVI+Yí' (11 Vl+uí'
iJF2 I
=
Y 1+ !li' .
r
F'J.-yz-,
iJy, (12

Introduciendo estas expresiones en (9), encontramos

I + <Il'Yí 1+ ID'uí (11)


VI V 1+lIí' 02 Vl+!lá"
Sea y el ángulo que forma con el eje Ol' la tangente a la lInea de sepa-
ración en el punto de abscisa e, sea a el ángulo que forma con el eje
O« el rayo de la izquierda y sea fl el ángulo que forma con el eje Ox
el rayo de la derecha. Entonces, se tiene ~' = tg y, Yi = tg a, = y,
S n, P~08LEMAS DISéoNTINUOS 141
= tg ~ y la condición (11) toma la forma
1 + tg (¡ tg i' l + tg ~ tg i'
Vi VI + tgZ (¡ [1% V 1+ tg2~
o
cos (y-ex)
tJz
(ti

donde 'V - (¡ y 'V - ~ son los ángulos entre los rayos y la tangente
a la llnea de separación. Tomando en lugar de éstos los ángulos lp y e
entre la normal a la línea de separación y los rayos, incidente y re-
fractado, obtenemos
sen <p tll
--=-=consl,
sen e ClZ

o sea, la tey de refracción del rayo de luz.


20. Varlac:lones unilaterales. Se pide hallar el extremo de la
funcional

I
Xli

J [y (x)) = F (x, y, g') dx;

con la condición
y - O (ó y - lp (x) ~ O)
q> (x) ~ (12)
(las condiciones de limitación pueden ser de forma más compleja).
En este caso la extrernal buscada puede estar formada por trozos
de extrernales que pertenecen al recinto (12) y por trozos de la frente-
ra y = q> (x) de este recinto. En tos puntos de j unción de estos trozos
la extremal buscada puede ser suave y también puede tener puntos
angulares.
La condición en el punto de [uncián tiene la lorrna
[F(x,!J, y'l-F(x, y, tp')_(q¡/_!J') Fil' (.x, y, y')J1 -=0.
~-x
_Si_FIJ'JJ' ::;¡60, la extremal es tangente en el punto de [unclón
M (x, y) a la frontera y = q> (x) del recinto.
EJEMPLO 4. Hallar en el recinto y ~ X2 el camino más corto del
punto A (-2, 3) al punto B (2, 3).
SOLUCION. El problema consiste en hallar el extremo de la Iuncío-
nal
2
J [y (x) I= jV 1 +y' '1. d» (13)
-2
con las condiciones
!I ~ xi. y (-2) = 3, y (2) = 3.
i42 CAP. 11. extREMO DE FuNCIONALES

Las extremales de la funcional (13) son las rectas


y = el + Csx.
En nuestro caso

F".".= 1I '4= O
(1 + y'2)2
Y la extrernal buscada se compone de los trozos AM y NB de las rectas
tangentes a la parábola y = xt y del trozo MON de esta parábola
(Hg. 24). Representemos las abscisas de los puntos de tangencia por x
y -x (utilizamos la simetría del problema). En el punto de tangencia

A (-2,J)

Fig. 24

colnclden las ordenadas y los coeficientes angulares de la recta y de la


tangente a la parábola de modo se tiene

el+e~="i: } (14)
ez=2x·
Por otra parte, la tangente debe pasar por el punto B (2, 3) y, por
consiguiente,
CI se, = 3. + (15)
Eliminando CI y C, de (14) y (15), encontramos 4x 3 = 0, x' - +
de donde Xl = 1 y i. = 3. El segundo valor de X no sirve. Es decir,
i = 1 Y el = -1 Y C. = 2. La extremal buscada (única) es
-2X-¡ si -2<:.%<-1.
II=-
{
x? si -1 ,x< l.
2x-l si l,x<2.
Queda claro que la funcional (13) alcanza en ella su mínimo.
TEORtA DE HAMI L tON ..J ACOBl 143

191. Hallar las curvas en las cuales puede alcanzarse el


extremo de la funcional
10
J[y(X)]=) y'3dx; y(O)=O, y(IO)=O;
o
si las curvas admisibles no pueden pasar por el interior del
círculo que limita la circunferencia (x - 5)2 + y2 = 9.
192. Entre las curvas que unen los puntos A (a, Yo)
y B (b, y¡) hallar la curva que ofrece el valor extremo a la
funcional
b
J [y (x») = y VI-y2y'2dx J
(1

con las condiciones y~O, l-!ly'2~O.


§ 12. Teoría de Hamiltoo-Jacobi. Principios variado
nales de la Mecánica
1°. Forma canónica (hamlltoniana) de las ecuaciones de fulero
Las ecuaciones de Euler para la funcional

r
J [YIt Y2. "" Unl =

= F (x, Yt. !h.•... , !/n. !/i, y;, .... y~) dx (1)


Xi
tienen la forma
d
Fu --d F ,=0 (k= 1,2 •... , n). (2)
k X I/It

En el caso en el que el determinante


FJI'I/'
11
F 1/''''
la
•• , F 1/''''
in
FII,,,,F 11'11' ••• FII'IJ'
al tI! in ,*0 (3)

pondremos
F , = PIt
IIk
(k = 1, 2, . • .• n). (4)

De las ecuaciones (4) se puede expresar Jlk en términos de x, JI1, JI"


•.• , Un' PI, P'A•••• , Pn:
gÍt = <Pk (x, !l1. Y•••• " !In' Ph Ps ••••• Pn)'
i44 CAP. JI. EXTREMO DE FUNCIONALES

La (unción H de las variables x, Yl. y" ' . o, Yn' Pi, P2. , •.• Pro
definida mediante la igualdad
H=[-F(x. 1It, 11"}.."0' Un. !/í. y;, .... U,I)+
11

+ ~ YÁP r (x, !lb Y2, o oo, y,.. y;'. y~•..• , Y~)]l , ,


~1 ~ ~~
se denomIna hamiltonlana de la funcional (1).
El hamiltoniano satisface las relaciones sigu lentes
aH dyl!. 01{ dp",
{)Ph=--¡¡X' iJy",=-dX (k=1,2, ... ,n). (5)

Se dice que las ecuaciones (5) son el sistema canónico O hamiltoniano


de las ecuaciones de Euler (2); las varIables 111. !h, ...• Yn' P¡, PI •. , .
. . .• Pn llevan el nombre de variables canónicas,
OBSERVACiÓN 1. La condición (3) en el caso de la funcional
OCa
J (U (x)l= J F (x, y, y'» üx da FII'II,4: O en [XI. x2)'
:I:¡

OBSERVAcrÓN 2. Hablando en términos generales. las ecuaciones


(4) no se pueden resolver unívocamente respecto a Yi en todo el seg-
mento IX1> !Cal. Si se cumplen las condiciones del teorema de existencia
de la (unción Imptícita , las ecuaciones (4) admiten solución unívoca
localmente.
EJEMPLO l. Formar el sistema canónico de las ecuaciones de Euler
para Ia funcional

J
.1t

J [Ut. Y21= (2Y1Y2-2U~ +y? _Y~Z) dx.


o
SOLUCiÓN. En nuestro caso
F = F (x, !/lt Y2' !/í. Y2) = 2ylY'J. - 2yj + !/? - uí'.
Ponemos
F , = PI
lit
y F,
1/2
= PI'
Entonces
P1 = 2y'; y
Aquí el determinante
F V'l/'
11
F l/'V'
1 2
FI/''J' FI/'V'
2 1 a 2
Resolviendo respecto a y~ e Y2 Ias relaciones obtenidas. encontramos
' - Pi
!J1-2 e y' P2
'=-2'
§ 12. TEORfA DE HAMILTON-JACOBI 145

Formamos el hamlltoniano de la funcional considerada


H = (-F +Yi.Fr.+yíF y.)
1 a, Pi
"1-2'
II,=_PI
• 2.
p. pi
=(-2YIY2+Zgl+y;,a-Yit) , 1'1 =2Yf-2¡I1Y2+-;f---f,
111"'2'
II'__ P.!
2 2
Empleando las relaciones (5), obtenemos el sistema canónico de las
ecuaciones de Euler
dYt =.!!1. . d!l2=_~. }
dx 2' dx 2 '
dPt .
(.[i= -411t +2Y2; dP2_211
7iX- t· .

Aquí Y1 = III (x), y, = Y:I(x], Pl = PI (x) y PI = Pa (x) son funciones


Incógnitas de x,
EJEMPLO 2. Formar el sistema canónico de las ecuaciones de Euler
para la funcional

J rUi. 92)= J YIY¡ (x2+ lIi +lIí) dx,


SOLUCiÓN. Aquí
F = yfyª (x2 + uí + gil·
Determinamos las derivadas parciales
F 11'
1
= U~yl, y F y'
J
= !I~YI·
Ponemos
Pl=y~yl Y Pz=y~y:·
Estas relaciones no comprenden las derivadas lIi e Y:Í de las funciones
Incógnitas lI1 e Ya; por eso, no se puede expresar y' e Yí en términos
de PI y PI' Por consiguIente, no se puede formar e\ hamiltoniano de
esta funcional. En este ejemplo no se cumple la condición (3):

_1 OO O01 =- O •
EJEMPLO S. Formar
I F "iYí F "íliÍ \
FI/'v' Fy'Y' -
I 1 a
el sistema
I

canónico de las ecuaciones de


Euler para la funcional

J [y (x) I = J xgy'3dx.

10-01381
146 CAP. 11. EXTREMO De FUNCIONALES

SOLUCiÓN. Tenemos
F = xyy'3 y Fu' = 3xyy"J.

Pongamos p = 3xlIy'2, de donde

y' = - J/ ~: s 11'= V 3~g·

La ¡uncional considerada tíene dos hamll tonianos


Hi=(-F+yIF""j.
,,-- Vp-=
~

=2Xyy/31
1/'__ ' íP
=-~3y3 V p3 ,
xy
V3x"V
2 I"pa
H,_=(-F+y'F1/' , líp=3V3 V xy'
I1/- . "Jiü
En concordancia con esto obtenemos dos sistemas canónicos de las
ecuaciones de Euler:

dI! .. /p 1
dx = JI 3xy' l
dp
dX"="3 V
1 .. /7
3xy3'
J
Formar los sistema canónicos de las ecuaciones de Euler
para las funcionales siguientes:
193. J [y (x)} = J xyV y' dx,
194. J [y (x)J = J xyy'2 dx.

195. J [y «n = J ~í X2 + y Vl + x':>' dx,


2

t 96. J [111. Y2}:;:: J (y? + y: +y~2)dx,


S 12, 'l'EOIHA bE HAMILTON·JACOBI 147

197. J[Uft Yzl= J (xz+Yty;l+y,.y~~)dx.


198. J lYlt Y~J = J (2xYt - y~2+-} "!I~a) dx,
2°, Ecuación de HamUton-Jacobl. Teorema de Jacobl. El
sistema canónico (5) de las ecuaciones de Euler es el sistema de ecua-
ciones de Euler para la funcional
J (Ui, !/a, ., " !/n] =

= r[~
~, 1-1
Pk!/~ -H (x, Uit 112' .•• , !In' Pi' Pa, ..• , Pn)] dx

si 1/1- Y••... , Yn' p" PI' .• " Pn se consideran en tanto que fun-
ciones incógnitas de x.
Esta funelonal J es la solución de la ecuación en derivadas par-
cíales de primer orden
aw
~+H
( x, Ut, aw iJW iJW )
Uz, .•• , Un' -iJ • -iJ , .• -, -iJ =0,
ox !Jt 112 Yn
que se denomina ecuación tú Hamilton-Jacobi.
TeO~EMA DE JACOBI· Supongamos qu.e W es la integral completa
de la ecuación de Hamllton-Jacobi y satisface la condición
¿¡2W Q2W iJ~W
OYt ec, iJYlOCa
--, OY. oc;
a2w Ol-W a2w
oY2OC. iJYaiJC2 - .. iJY2iJCn :::/= O.

cJ2w ()2W cJ2w


aYn se t aYn ec; ... aYn iJCn
Entonces las igualdades
iJW aw
oy/¡, =p/¡, (k= 1, 2, .. _, n),
OCA =8k,

dJJnde Ch. y Bk son unas constantes arbttrarlas, determinan una solución


tUl sistema canónico (5) lÚpendiente tU 2" constan/es arbitrarias.
EJEMPLO 4. Hallar las extremales de la funcional

J [y (x)] = lit) V X2 + !l V 1 + y'2 dx


:tI
utilizando la solución de la ecuación de Hamilton-J acobí.
JO·
148 CAP. 11. EXTREMO DE FUNCIONALES

SOLUCION. Para obtener la ecuación de Hamilton-J acobi íorma-


rnos el hamiltoniano de la funcional considerada. Tenemos
H= -lf x2+yZ_pZ.
La ecuación de Hamilton-Jacobi tiene la forma

aw
-- I./: XZ+y2_ (OW)2
- =0
ex r' ay
ó

( OW)2.
OX + (dW)2
Tu = x2+y2. (6)

Representemos la ecuación (6) en la forma

(~~
r -x2 +( ~~)2 -
Y apliquemos el método de separación de variables.
y2 =0
Queda claro que
la ecuación (6) se ver illca si se exige que

( aw
ox
) 2. _ x2 =_C y (aw)
ay
2_ y2=C
I

donde e es una constante arbitrarla. De aquí encontramos

ox =Vx2-C
aw y
aW =lfyZ+c.
oy
La integral completa de la ecuación (6) será

W= J Vx2-C J
dx+ V y2+Cdy=
=+xlf.t2-C - ~ In)x+ 1fx2-C 1+; y V y2+C+
+; In ls+ yy2+cl+co1
donde C y Co son constantes arbitrarias.

De la relación
determinamos
~~ =+ I donde A es una
la solución general de la ecuación de Euler, Tenemos
constante arbitraría,

x ___!_¡ x+,VXCCI+ E. I +
4Vx2-C 2 4 (x+lfxz-C) lfx2-c
+TV;+c+i- In!y+Yy2+cl+
~ 12. TEORIA DE HAMILTON-JACOBI 149

Después de unas simplificaciones sencillas, obtenemos

In IY+Vyq=c
x+Vx2-C
I=A. o sea, u+Vyq:c
X+Vx2-C
=A

(A=± eA.),

de donde resulta definitivamente

es decir, una familia de hipérbolas.

Hallar las extrernales de las funcionales siguientes:


~2
199. J [y (x)) = j xy -vy' dx,
e
200. JIY(X)]=JxyyI2dx; y(l)=O, y(e)=l.

r
1

201. J [y (x») = a (y) V 1 +y '2dx.

202. Hallar el mínimo de la funcional


1
J [y (x)] = J ({-
y'2+ yy' +s' + y) dx
o
si se desconocen los valores en los extremos del segmento.
203. Hallar la función del campo p (x, y) y el propio
campo de extremales que pasan por el origen de coordenadas
para la funcional
(~.II)V--
J[y(x)l=) ¡:Y'Z-dx (y>O).
(O, O)

204. Entre las líneas que unen el punto x = O con el


punto Nf¡ (Xl, Yl), donde Xl> O e Yl> 0, hallar la línea
en la que alcanza su mínimo la funcional
150 CAP. 11. EXT"REMO DE FUNCIONALES

XI

J [y (X)] = ) vr:¡:y'2 dx (y> O).


O 11
Supongamos que se tiene la funcional
:rll
J [y (X)[ = J F (x, y, g/) dx
Xi
y se conoce su campo de extremales y = q> (x, C). Entonces en todo
punto del campo se conoce la dirección de la transversal del campo que
pasa por este punto. Todas las transversales del campo se obtienen
como las soluciones de la ecuación diferencial de primer orden

F!J' rx, 'P (x, C), q>~ (x, C)] :~ = H (x, (jl (x, C), q>~ (x, C)),

donde en lugar del parámetro e, que determina las ex tremales del


campo, hay que introducir su expresión en términos de las coordenadas
de los puntos de) campo. Aqul H (x, y. p) es el hamiltoníano.
EJEMPLO 6. Hallar las transversales para el campo de extremales
y = ex de la funcional

J [y (x)] = r
:I:{
y'2 dx,

SOLUCION. Formamos el hamiltoníano de la funcional considerada.


Tenemos
F = y'2 Y F 11' = 2g' (F 11'11,2 =1= O).
Poniendo p=FlI" encontramos y'=r~ y

H=(-y'2+2y'y') I
II'-~
= P: .

Las transversales se obtienen resolviendo la ecuación diferencial

F 11' II/-cx :~ = H 1,,-211' _ 2C t

donde en lugar de e hay que tomar su expresión en términos de las


coordenadas de los puntos del campo: C = Jf_. Tenemos
JI:

2y' Iv-cx :~ = ~ /P_2C Ó 2C ~~ = C2.

Puesto que e ~ o, se tiene 2 :; =C, o sea, 2!! = ~. De aquí


encontramos <jue la familia de transversales son les parábolas 1J2~ e",
§ 12. TEORIA DE HAM!LTON-JACOBI 151

205. Hallar las transversales del campo de extremales


y = ex de la funcional
~2
J [y (x)j = J F (y/) dx,

206. Hallar las transversales del campo de extrema les


y = x + e de la funcional
J (xy't. -
X2

J (y (x) I= 2yy'3) dx.


XI
207. Hallar las transversal es del campo de extrernales
x2
y=x-c de 'la funcional
"'2

J W(x)] = J Vy(I-y'2)dx (C>O, x>O, y~O).

Conocíen do 1a ecu ación de H amil ton -J aco b i


aw (
ax + H x, y.
¿¡W'
ay
) =o
de la funcional
:x:z
J fu (x)J = J F (x, y, ;y') dx,
=1

se puede reconstruir la función integrando F (.r. !J. y'). Esta es solu-


ción de la ecuación diferencial de primer orden
F - zF; = -H (,(, y, F~, (7)
donde H (.r, IJ, p) es el hamiltoniano de la funcional considerada
y F (x, !I, z) es 1a f unción inc6gni ta (se considera que x e y son pará-
metros). Después de determinar F (x, y, z) hay que tomar en ella la
derivada y' en lugar de z,
OBSERVAC.ION. La ecuación (7) es la ecuación de ClairauL Como
regla, la solución general de la ecuación de Clairau t se omite pues
en este caso la función integrando F (x, y, y') es lineal en y' y el
problema varíacional no siempre tiene solución (véase el § 4). Por
eso, se toma sólo la solución ingular de la ecuación de Claíraut que
será precisamente la función buscada F (x, y, z).
EJEMPLO 6. La ecuación de Hamilton-J acobi en el problema
Xa
sobre el extremo de la Iunclonal 1 [y (x)l = ) F (x, y, !J') t4 tíene
:lI4
152 CAP. 11, EXTREMO DI! FUNCIONALES

la forma
aW)2 + (aW)2
( ox Tu = ,~2+y2.

Hallar la función F (x, y, y').


SOLUCION, R.esolviendo la ecuación dada respecto a la derivada
aw
ox ' tenemos
aw '.V/
-ax= x2+V2-
(aW)2
iJy t

o sea,

Por constguien le, el hamiltoniano es


H= -Vx2+y2_p2.
La ecuacíon (1) para la determinación de la lunclón F tiene la
forma
dF
F-z-=
dz
Ji" ;c2+y2__(dF)2
dz •
(8)

Derivando respecto a z ambos miembros de la ecuación (8), resulta

dZF
Dejando a un lado el caso dz2 =0 (que da la solución general, tene-
mos
dF
dz
z =-,~
/====::;:dF;::::::;02 •
V ,\,2+V2- (dZ)
Resolviendo esta relacIón respecto a la derivada :: ' encontramos
dF _zV~ (9)
Tz- Vl+t2 .
§ 12. TEORrA DE HIIMILTON ..JACOBI 153

Introduciendo (9) en (8), obtenemos

F=z z vii+Y2
VI+:z2
+..Ví xa+ y 2_ 22 (x2+y2)
I+z~
·V:c2+y:qll +z:1.

Por consiguiente, la función integrando buscado tiene la forma


F=Vx2+y2 Vl+y'2.
En los problemas que siguen hallar las funcionales a partir
de sus ecuaciones de Hamiltoo-J acobi:

209. 4 aw aw _ . .2
(Ix oy - ,{,.-y •
..2.•

2JO. 4xy aw (aW)2


Tx+ a¡¡ = O.

21 J. aW)2
( X-ax +( y aW)2
ay =x2 +y2.
30. Principios variaclonales de la Mecánica.
a) PRINCIP(O DE HAMILTON - OST'ROORADSI<I.Supongamos que se
tiene un sistema de n puntos materiales Mil. (XII' 1111.. 2'1I.) (k = 1, 2, ,." n)
con masas respectivas mk (k = 1, 2, .. " n). Supongamos que el
movimiento del sistema está sometido a enlaces
(jlJ (x, y, z, t) = O (J = 1, 2, ... , m; m ~ n) (10)
y se realza bajo la acción dejas fuerzas PII. (X,." Yk' ZJ¡) (k = 1, 2, oo., n)
que tienen el potencial (funci6n de Iuerz a) U = U (x.\. Yk, zk. t):

Xh=;;-,
su eo
Y,,=:;--,
eo ..
ZJI.=~
IJxk ..yft "'k
La energía cinética de este sistema será igual a

Supongamos que este sistema pasa de cierto estado A correspondiente


al momento del tiempo t = to a otro estado B correspondiente al
momento de tiempo t = tI' Entre todos los desplazamientos posibles
del sistema de A a B se escoje la clase de movimientos admisibles que
concuerdan con los enlaces dados y que hacen pasar el sistema del
estado A al estado B en el Intervalo de tiempo dado (io, '1)'
El principio de Hamilton-Ostrogradski consiste en lo siguiente:
entre todo« los moulmienlO&admisIbles que hacensasar el sistema
¡54 CAP. 11. EXT~EMO DE FUNCIONALES

del estado A al estado B, el movimiento real se caracteriza por el


cumplimiento de la condiclén necesaria fjJ = O de ex/remo de la
funcional
ti
J= J (T-I-U)dt. (11)
to
A cada movimiento admisible del sistema le corresponden 3n
funciones Jek (1), !lk (/), %/1 (1} (k = 1, 2, ... , n) que están definidas
en el intervalo [to, ',], que satislacen las ecuaciones (lO) 'i que toman
determinados valores en los extremos del Intervalo [to, t11. Por consi-
guiente, tenemos un problema variacional con los enlaces (lO) y con
fronteras fijas.
Para resolver este problema formamos la función auxiliar de
Lagrange
m
F=T+U + íJ AJ (t) 'Pi
;=1
y escribimos para ella el sistema de ecuaciones de Euler-Ostrogradski:

•• m u'PJ
mkXk-XB - ~ J..j (t) axl>. =0,
j ... 1

•• m u'PJ
mkYIt-Yk- ~ AJ(t) U!lk=0, (12)
j-t
•• m O'PJ
mllzli.-ZIt- ~ AJ(t) OZh =0.
;=1
El sistema (12) conlclde con las ecuaciones diferenciales del movlmlen-
to real del sistema.
b) PRINCIPIO DE 1.A ACCIÓN ~MINIMA EN LA FORMA DE LAGRANOB.
Supongamos que Jos enlaces ''Pi y el potencial U no dependen del
tiempo t. En este caso tiene' lugar la integral de energía T - U =
= h = consto La integral
/1

J = J Tdt
lo
se denqmina I!c~i(l{l.De la integral (11) se deduce que

J
~
IJ
(T+U)4t=~
,~
ti
(Tdt-)
li
h dt,
§ 12. TEOR fA DE HAMI L TON·J ACOBl 155
El principio de la acción mínima en la forma de Lagrange consiste
en lo siguiente: para el mooimiento real. la integral de la acción debe
tomar su valor mínimo. O sea.
It

J= J Tdt=min.
to
El principio de la acción mínima puede ser representado en la
forma de Jacobl
~ V2(U+II) ds=mín
'\1
(ds es la diferencial del arco y) en la que no interviene el tiempo.
OBSERVACION l. Aqui se consideran admisibles los movimientos
que satisfacen las ecuaciones de enlace ~J (x. Y. z) = O (} = 1, 2•.••• m}
y la ecuación T - U = h con el mismo valor de h que para el movi-
miento real y que tienen los estados inicial y final rijos. siendo tam-
bién fijo el momento Inlclal lo del tiempo. El momento final del tiempo
no se fija para estos movimientos.
OBSERVACIÓN 2. La energía potencial figura no en la integral sino
en la condición complementaria T - U = h. Formamos la función
auxiliar de Lagrange
m
1 1 ~
F =2' T+2' (U+h)+ LJ "'JfPJ'
;-1
Después escribimos las ecuaciones de Euler-c-Ostrogradski para nuestro
problema
m
•• iJU a~¡
mkxk = ;¡--+2
vXh.
'" Xl -r-
.t:.J vX)t
t

;-1
m
•. au iJCFl
mhYlI.=-a -+2 .a
YIt
~ ¡"J;¡-,
v!lk
;-1
.. iJU m OCPJ
mhzll. = OZIl +2 ~ ¡"J iJzll
t
j ...

que representan las ecuaciones del movimiento real.


EJEMPLO 7. Basándose en el principio de la acción mínima, hallar
la trayectoria del punto material (de masa unitaria) que se mueve
por acclén de la gravedad.
SOLUCIÓN. Tomando ~l eje Oy hacia arriba. el potencial de la fuer-
za de la gravedad es
(l3)
156 CAP, 11. EXTREMO DE FUNCIONALES

Según el principio de la acción mínima, para la trayectoria bus-


cada 'Yo la integral
J=SV2(U+4)dS (14)
y
debe alcanzar su valor mínimo. Por consiguiente, la trayectoria será
una extremal de la funcional (14). Introduciendo (13) en (14), obtene-
mos
Xi

J= S V2(h-gY)Vl+y'2dx. (15)

La ecuación de Hamilton-Jacobi tiene la forma

-ax- JI
c)W -. /'
2h-2gy- ( aw ) 2. =0,
ay
o sea,
(~~) 2+ (~~ r =2 (h-gy).

Su integra l completa es
3
W=Ax+ Jr l/2h-2gy-A2
1-
dy=Ax- 3g (2h-2gy-A2) 2 +B,
donde A y B son constantes arbitrarias.
Determinamos las extremales de la funcional (15):
t
X+~(2h-2g!l_A2)2 =C,
g
o sea,
y= ; - :; - 2~2 (x-C)2; A y e constantes.
En particular, las exíremales que pasan por el origen de coorde-
nadas se determinan de la condición y (O)=0. Obtenemos una Iarni-
lia rnonoparamétr Ica de parábolas

s=>- 2A2 x2+


g :t:2ii=Ai
A x.

212. Hallar en el plano la trayectoria de un punto que


se mueve por efecto de una fuerza repulsiva que actúa desde
el eje Ox en dirección del eje Oy y que es proporcional a la
distancia del punto al eje Ox aceptando que la integral de
la fuerza viva tiene la forma ~ - ~s = O Y basándose en
§ 12. i1~ORtA DE HAMIL 'tON·JACOS) i 57

la integral de la acción

J y VI +y'2dx
X2

J [y (x)] = (y> O).

213. Un punto material describe la circunferencia p =


= 2R cos cr (p,q> son las coordenadas polares) de radio R
bajo la acción de una fuerza central :6
inversamente propor-
cional a la quinta potencia de la distancia al centro que se
encuentra en el origen de coordenadas. Demostrar que la
integral de la acción alcanza mínimo fuerte en cualquier
arco de esta circunferencia (- ~ < cri ~ q.l ~ (f1\J < ~).
214. Analizar el movimiento de un punto material por
efecto de una fuerza central de atracción proporcional a la
distancia al centro O basándose en el principio de la acción
mínima y aplicando el método de Hamílton-J acobí.
Capitulo ¡JI
M:ÉTODOS DIRECTOS EN EL CÁLCULO VARIACIONAL

§ 13. Método de diferencias finitas de Euler


Consideremos el problema variacional elemental: hallar el extre-
mo de la funcional
b
J [y (x)] = ) F (x, y. y') dx¡ !J (a)=A, y(b)=B, (1)
a

Según el método de Euler, los valores de la funcional (1) se toman


no en las curvas arbitrarias que admite este problema variacional,
sino en las quebradas compuestas por un número dado n de segmentos
rectilíneos cuyos vértices tienen abscisas fijas
b-a
a+6x, a+26x, , .. , a+ (rt-I) Sx, donde ax = --.
rt

En estas quebradas la funcional J [y (x)l se convierte en una Iunclén


(Yi, Y2, ... , YV-l) de las ordenadas g., y" ... , Yn-I de los vértices
(IJ
de la quebrada. Las ordenadas Yl, Y2••• " Yn-l se escogen de modo
que la función «I> (Yl, !/i, ... , Yn-l) tenga extremo, o sea, se deterrni-
nan del sistema de ecuaciones
o«I>
iJy? =0, ... , ~-o
oYn-l - .
La quebrada así obtenida es la solución aproximada del problema
varíacional (1).
EJEMPLO. Hallar la solución aproximada del problema sobre el
mínimo de la funcional
1
J{y(x)l= J (y'2+2y)dx¡ y(O)=y(I)=O.
o
1-0
SOLUCION. Tornemos Óx = -5- = 0,2 Y pongamos
Yo = Y (O) = O, !JI = y (0,2), Y2 = y (0,4),
Ya = Y (0,6), Yl = Y (0,8), Y5 = Y (1) = O.
MeTODO b~ DIFERENCIAS FINITAS 159

Sustituimos los valores de la derívada según la fórmula aproximada

Yk' -y' (x ) '"'" YIt+t -YIt


- k"" 6% •
Entonces

11' (0)---oy-,
lit-O y' (O2)-
, -
Y2-1I1
0,2 '
y' (0,4)= '13;;;112 ,
,

Y ' (0,0·.1)= 1I~~Y3,


, 11,'(0 8) =~.
O-Y4-

Sustituimos la integral por una suma empleando la íérmula de los


rectángulos:
o
) f (x) dx ~ [f (a) +t (.~1)+ f (x2) + ...+ f (xn.-t)J 6x.
(l

Tendremos

(l) =[
(Yt, 112,Ya, 11",) ( t.~) + ( 2 Y~~ Y1 ) 2 + 2Y1+
+ ( !/3-;:./1.)2 +2Y2+ ( Y";;1I3)2 +2Y3+ ( _ t.~)2 +2y ,J .0,2.
Formamos el sistema de ecuaciones para determinar las ordenadas
1111 g .. Ya e y!" de los vértices de la quebrada buscada;
1 ofI) 111 117.- lit
0,2 oYt = 0,02 -- 0,02 +2=0,
_1_ am = 112-Yt Y3-Y2 +2=0
0,2 oY'l. 0,02 0,02 •
_1_ a<D = Y3-Y2 Y4-YS +2=0,
0,2 aYa 0,02 0,02
I aa> y",-Ys 114
0,2 oy4. = 0,02 + 0,02 +2=0,
o sea,
2Yt-Y2=_O,04,}
-yd-2yz-ys= -0,04,
-Y2+2Ya-YI,,= -0,04,
-Y3+2Y4 = -0.04.
La solución de este sistema es lit = -0,08, 112 = -0,12, y, = -O, J 2
e Y4 = -0,08. Estos valores de la solución aproximada coinciden
con los valores que tiene en los puntos respectivos la solución exacta
x2-x
y= -2-'
160 CAP. J 11. MéTODOS OIRttCTOS

Hallar las soluciones aproximadas de los problemas sobre


el mínimo de las funcionales:
1
215. JIy(X)1=) (y'2+y2+2xy)dx; y(O)=y(I)=O.
o
SUGERENCIA. Tomar 8..1:=0,2.
j

216 • .1 [y (x)] = J (y'2+ 1) dx;


o
a) !J (O) = O, y (1) = O;
b) Y (O) = O, y(1)= l.

§ 14. Método de Rítz. Método de Kantorévích


10. Método de Rltz. La idea del método consiste en que al hallar
el extremo de la funcional J [y (x)) se consideran, en lugar del espacio
de las funciones admisibles, s610 fas funciones que se pueden represen-
tar como cornb lnaciones lineales de las funciones admisibles:
n
Yn (x) = ~ IX,IPI (x). (1)
j ... 1
donde !x,
son unas constantes y el sistema {<Pi (x)}. llamado sistema
de [unciones coordenadas, está formado por funciones lp, (x) que son
linealmente independientes y que constituyen un sistema completo
de funciones en el espacio considerado.
Hablando en términos generales. cuando pedimos que las fun-
ciones Yn (x) sean admisibles, Imponemos a las funciones coordenadas
<PI (x) ciertas condiciones complementarias como, por ejemplo, llrnlta-
cienes en cuanto a la derivabilidad o en cuanto a la verificación de
las condiciones de Irontera.
En estas combinaciones lineales la funcional J fu (x») se conv íerte
en una funci6n de los argumentos !Xl' IX2, •.. , Ctn:
J [gn (x)l = <D (Ct.¡, IX2••.. , IXn)·
Determinamos los valores !Xl. IX" ••• , IXn que ofrecen extremo 8 la
función <D (!Xl' !X2, •.• , !Xn); para ello resolvemos el sistema de ecua-
ciones
o<D
~=O (i= 1,2, ... , n),
UIX¡
no lineales, como regla. respecto a !Xh a2, •.• , ct,u e introducimos
en (1) los valores encontrados para !X,. La sucesión {Yn (x)} que asl
resulta es una sucesión minimlzante, o sea, la sucesión de los valores
de la luncional {J [Yn (xH) obtenida El partir de elJa converge hacia
J 14. MSTODO DE RITZ. MSTODO DE KANTOROVICII t!)t

el minimo o hacia la cota inferior de la funcional J fu (x)l. Sin embargo,


de
lim J {UIl (x)]
n ... oo
= min J 1.9 (x)J
no se deduce aún que 11m Yn (x) = U (x). La sucesión minlmtzente
n ... oo
puede no converger hacia la funci6n que realiza el extremo en la
clase de las funciones admisibles.
Se pueden indicar las condiciones que garanticen que el mínimo
absoluto de la Iuncíonal exista y se alcance en las funciones (l/n (x)}.
En el caso en el que se trata del extremo de la funcional
%1

J (y (x)i = JF (x, 11. y') dx;


%1

11 (XI) = Yit
V (x2) = 112:
estas condiciones son:
1) la función F (x, y, t) es continua respecto al conjunto de sus
argumentos para cualquier z y para (x, y) E D. donde D es un recinto
cerrado del plano xOy al que pertenecen las lineas Un (x);
2) existen unas constantes ex > O. p > I Y ~ tales que
F (x. y, z) ~ I z IP +p
cualquiera que sea z y para cualquier punto (x, y) E D;
. 3) la funci6n F (x, !I, z} tiene la derivada parcial continua
Fz (x. y, z) y esta derivada es una función no decreciente de
Z (- 00 < z < +(0) cualquiera que sea el punto (x, y) E D.
En particular. las condiciones enunciadas se cumplen para las
funcionales

J (y (x)) = r
~I
(p (x) y'2+Q (x) y2+2r (x) yl dx;

if.(xl)=a, y (x2)=b;
donde p (x), q (x) y r (x) son funciones dadas, continuas en [XI> xsI.
con la partlcularidad de que existe la derivada continua p' (x) de
p (x) y de que p (x) > O Y q (x) ~ O.
Si por este método se determina el extremo absoluto de la funcio-
nal, el valor aproximado de su mfnimo se obtiene por exceso y el
valor aproximado de su máximo, por defecto. Al aplicar este método,
el éxito depende en gran medida de la elección adecuada del sistema
{Q?t (x)} de funciones coordenadas.
En muchos casos basta tomar la combinación lineal de dos o tres
funciones 'l>J (x) para obtener una aproximación bastante satisfactoria
de la soluclón exacta. .
Si hay que determinar el extremo aproximado de la funcional
J (z (Xl' xs•. _ ., xn)) que dependen de las funciones de varias varia-
H-01381
162 CAP, 111. MeTODOS DIRECTOS

bies independientes. se escoge un sistema de funciones coordenadas


11'1(Xl. Xa•••• , xn),

Cj), (Xl' .ta, .. ". Xn) •.•. , Cj)m (XI. Xli' ••• , Xn), ...

y la solución aproximada del problema variacional se busca en la


forma
m
Zm(Xh X2, ••• , Xn)= ~ akll'k(x" Xz••.. , Xn).
11",,1

donde los coeficientes all son unos números constantes. Para determi-
narlos se íorrna, por analogía con lo que hemos explicado, el sistema
de ecuaciones !(1) = O (k = 1, 2,
VaA
... , a), donde Cll (ah a,l, ... , a n)
es el resultado de introducir 2m en la funcional J (z (Xl. Xli' .• "' x~)I.
EJEMPLO 1. Hallar la solución aproximada del problema sobre
el mínlmo de la funcional .
t
J (y (xH= J (y'2_ y2+ 2xy) dx; (2)
O
Y (O) = g (1) == O;
Y compararla con la solución exacta.
SOLUCiÓN. Como sistema de funciones coordenadas Cj),\ (x) tomamos
epA (x) = (J - x) x1l. (k = 1, 2, . " .).
Es. evidente que las funciones epk (x) satisfacen las condiciones de
frontera Cj)k (O) = Cj)1I (1) = 0, son linealmente independientes y for-
man un sistema completo en el espacio ello, 11.
Para k = 1 tenemos 111(x) = IX¡ (x - x'). Introduciendo esta
expresión de Yl (x) en la funcional (2), obtenemos
i
JllIt(x)J= J (a;f{I-2x)2+al{x-x2)2+2alX(X-x2)ldx=
o
1
= J [al (1-4x+4x 2- -,z+ 2z3-x4) + 2!li (x2-z3)] dx=
11

3.1 "')
=lO ai+"6l%l= ....(at.

Ei coelícíente af se determina de la ecuacíon


0Cll 3 1
OGt =-sat +"6=0.
§ ie , METODO DE RITZ. MarODO De KANTOROVICH 163

5
de donde resulta a1= -18' Por consiguiente,

5 5
= --x+-x
Yt (x)
18
2
18'

SOLUCION EXACTA. La ecuación de Euler de la funcional conside-


rada es
y' + 1'1= x.
Resolviendo esta ecuación lineal no homogénea, encontramos
y = el cos x + Ca sen x + x,
Empleando las condiciones de frontera y (O) =y (1) = O, obtenemos
definitivamente
sen x
y=x-señ!'
Comparemos las soluciones exacta y aproximada:
II SolucIón exacta Sotuclén aprexlrnade
0,00 O O
0,25 -0,044 -0,052
0,50 -0,070 -0,069
0,75 -0,060 -0,052
1,00 O O
EJEMPLO2. Hallar la solución aproximada de la ecuación no
lineal
3
1'1-=2' 1'12

que satisfaga Ias condiciones y (O) = 4, Y (1) = 1.


SOLUc,rON. A este problema de contorno le corresponde el proble-
ma variacional
1
J IY(x»)= J (y'2+y3) dx; y (0)=4. y (1) = 1-
O
Buscaremos la solución en la forma
Y1 (x) = 4 - 3x + al (x - )(2);
es evidente que 1'11 (x) satisface las condiciones de frontera dadas cual-
quiera que sea el valor de ~.
Tenemos
j

J [JI. (x)l= S (la. (t -2x)-3}2+ [4-3x+a. (x-x2)]3) dx,


O
164 CAP. 111. MerODOS DIRECTOS

de donde
I
oJ[y¡(x)} = ~{(1-2x)2[a.t(I--2x)-31+
aa.t •
()

+3 (x-x2) [4-3x+CXl {x-x2)l2} dx.


La condición (jJ [!JI (x)] =O toma la forma
oa.¡
9cx~ +
490CXl+ 1407 = O
y para O!.l = -3,0413
obtenemos la solución del problema
!JI (x) = 3,0413x1- 6,0413x 4 +
posi Uva en lodos los puntos.
Hallar las soluciones aproximadas y compararlas con las
exactas en los problemas que siguen sobre el mínimo de las
funcionales:
1
217. J[y(x)l= J (y'2+2y) y(O)=y(l)=O.
dx;
o
2

218. J [y (x)] = J + y2 + y'2) y


(2xy dx; (O) = Y (2) = O.
o
2t 9. Hallar la solución aproximada del problema sobre
el mínimo de la funcional
I
J(y(x)1= J (y'2-k if')dx; 2 y(-l)=y(I)=O;
-1
t
con la condición complementaria J yZdx = l.
-1
EJEMPLO 3. Hallar la solución aproximada del problema sobre
el extremo de la funcional

J [z (x, g»)= Jr Jr l ax
éJz )
e- (
+ (ayaz )
2 2
- 2z dx
J dy,
D

donde D es el cuadrado -a ~ x ~ a, -4 ~ Y ~ a. siendo z = O


en la frontera del cuadrado.
SOLUCIÓN_ Buscamos la solución aproximada en la forma
Zo (x, y) = 0.0 (x2 - aa) VI' - aa).
, 14. METOOO DE RITZ. M~TOOO DE KANTOROVICH 165

Es evidente que esta función lO (x, 1/) satisface las condiciones de


frontera planteadas. Introduciendo Zo (x. y). zó" (x, y) y Zóy (x, y) en
la funcional e integrando. obtenemos
11$ 32
J Izo (x, y)) = ~5 o;~as-9ctoas=!ll (etol.
Tenemos después
0(1) 512 32
oaooa
---ct - 8_-a
45 6-O 9 -,
5 5
de donde cto= 16az de modo que ZO('~I y)= 16a2 I.x2-02) (1/2_02).

220. Hallar la solución aproximada del problema sobre


el extremo de la funcional

J (z(x, y)J = ) J r(~: _y)2 +( ~~+xf J dxdy,


o
donde D es el recinto limitado por la elipse ~: -+ ~: = 1.
221. Hallar la solución aproximada Z3 (x, y) del proble-
ma sobre el mínimo de la funcional
J [z (x, y)] = j J [( ~:) + (~;) J dx dy.
2
2
D

donde D es el recinto: x> O, !J > O Y x + !J < 1, si la fun-


ción z (x, y) satisface en la frontera F: x = O, y = Oy x +
+ y = 1 la condición z Ir = x'l + y'l.
20. Método de Kanterovleh. Este método ocupa una posición
intermedia entre la resolución exacta y el método de R itx y se aplica
para analizar el extremo de las luncionales
J (z (Xl. xa, .•. , xn)) (3)

que dependen de funciones de varias variables independientes (11 ~ 2).


Igual que en el método de R itz, escogemos un sistema {<Pk (XI. X2 ... ·• XII)
de (unciones coordenadas y buscamos la solución aproximada en la
forma
(4)

pero considerando los coeficientes a)¡ (xJ) como Iunciones incógnitas


de una de las variables independientes.
166 CAP. 111. MerODOS DIRECTOS

En las funciones (4) la funcional (3) se convierte en una funcional


1[«t (xJ)' a2 (xJ)' , Clm (Xj)] que depende de m funciones
al (X/), a. (xJ)' , G'tm (xJ)' Estas funciones se escogen de modo
que la funcional 1alcance el extremo y se determinan de las condicio-
nes necesarias de extremo para la funcional 1.
Empleando el método de Kantorovích, se obtiene una solución
aproximada, como regla. más exacta que la solución que da el método
de ~ Hz con las mismas funciones coordenadas 'P" (Xl, x:, ...• x n)
y con el mismo número m de términos en la aproximaci6n.
EJEMPLO 4. Hallar la solución aproximada de la ecuación de
Poisson

~¡r= -1
,
en el rectángulo D:
{-a~x-<a
-b -< Y -< b'

si z = O en la frontera.
SOLUCiÓN. La ecuación ~z = -1 es la ecuación de Euler-Ostro·
gradski para la funcional

J [2 (x, y}J = j J [( ~: t + ( :; ) 2- 2Z] dx dy. (5)


D
Buscamos la solución en la forma

%1 (x, y) = (h' - y') a (x);


es evidente que la función Zl (x. y) satisface las condiciones de frontera
z = O en las rectas y = +b.
Intrcducíendo esta expresión de Zl en la funcional (5), encontramos
a
J [zt(x, g)l = J (:~
-o
b5a'2+ ! b3a2 ~ b3a) dx; (6)

La ecuación de Euler para la funcional (6) es


,,5 5
Ct - 2bZ Ct= -W· (7)

La ecuación (7) es una ecuación lineal no homogénea con coeficientes


constantes y su solución general es

Las constantes Ct y Cz se determinan de las condiciones de frontera


Ct (-a)-a (a)=0
§ 16. LOS VALORES Y LAS FUNCIONES PROPIOS 167

lo queda C2=OyC,= - Ji 5 a
de modo que
2ch --
2 b

l ch 'y/52b 1 JC

a(x)- - 1- .
2 ,/5aj
Es decir, obtenemos
1 chy 2b

..V/521) 1
x
2'1 (x , y) =
b2_gZ
2 t-
ch
Ji 5 a .
I eh --
2 b
Para obtener una aproximación más exacta se puede buscar la solución
del problema en la forma
~2 (x. y) = (bt - y'') a} (x) + (b' - y') a, (x).
222. Hallar en el recinto D la solución aproximada de
la ecuación de Poisson 6z = -1 que se anule en su frontera
si D es el triángulo equilátero formado por las rectas y =
= ± ~3 x y x b. =
223. Hallar en el recinto D la solución aproximada de
la ecuación 6.z = -1 que se anule en su frontera si D es el
trapecio isósceles formado por las rectas y = ± ~3 x, x =
y x= 3.

§ 15. Métodos variacionales para la determinación


de 108 valores y de 188 funciones propios

La ecuación de Sturm-Liouville
d
- dx (p (x) ti) +q (x) g = AY, III
donde p (x) > O tiene derivada continua y q (x) es continua, COl1 las
condiciones
y (a) = O e 11 (b) = O (2)
tiene la solución nula (trivial) !I El O cualquiera que sea el valor real
o complejo de A.
168 CAP. 111. MeTODOS DI ~ECTOS

El conjunto de la ecuación (1) y de las condiciones de frontera (2)


se denomina problema dé contorno tU Slurm-Liouoille (1)-(2).
Losvalores de'" para los cuales el problema de contorno (1)-(2)
tiene soluciones no triviales g == O se denominan va/ores propios
y las soluciones mismas llevan el nombre de [unciones propia, del
probleftlll de contorno.
La ecuación (1) es la ecuación de Euler para el siguiente problema
sobre extremo condicionado: hallar el mfnlmo de la funcional
b
J lu (x)! = J (PII'z+ qUa)dx (3)
o

con las condiciones (2) y la condición


b
J y2
()
dx= 1. (4)

Si IJ = Y (x) es una solución de este problema variacional, también


será una solución del problema (1)-(2) distinta del cero Idéntico en
virtud de la condición (4). Por eso, los valores propios y las funciones
propias del problema de contorno de Sturm-Liouville se denominan
también valores propios y íunctcnes propias de la funcional (3) con las
condiciones (2) y (4).
La función propia 11 = Ij (x) se denomina normada si

EJEMPLO 1. Hallar los valores propios y las funciones propia,


de la funcional
3
J (y (~)J = J «2x + 3)2 y'2 - y2J dx
O
con las condiciones
ti (O) = O, Y (3) = O, (5)

J y2dx= 1.
O
SOLUCION. La ecuación de Sturm-Liouville tiene la forma
d
-y-7iX ((2x+3)2tll ='J..y,

o sea,
(2x + 3)'y" + 4 (2x + 3) g' + (A + 1) Y = O. (6)
Sil. LOS VALORES y LAS PUNCIONES PROPIOS 169

Mediante la sustitución 2x +
3 = tI la ecuación (6) se reduce a la
ecuación lineal con coeficientes constantes
d2y dy
4 dt? +4dT+().+l)y=O. (7)

Su ecuación caracterfst ica

tiene las rafees


4k' + 4k + )..+ I =O
I 1 '1r-
kt,2=-'2±2"Y -A.
Consideremos tres casos.
1) A < O. Entonces la solución general de la ecuación (7) es

y = Cle"tl + e.e"tf•
donde k. y ka son números reales; en consecuencia. la solución genera)
de la ecuación (6) es
y = Cl (2x + 3)111 + el (2x + 3)"1.
las condiciones de frontera (5) dan

Ct3"'+C23111=O. }
e19~t+C2~=O,
de donde el = O. Ct == o e y 5' O.
2) A = O. Entonces '

y, por consiguiente,

De las condíclones de frontera obtenemos


Ct+CJln3=O.
Cl +Caln 9=0,
de donde Ct=O y e2=o, o sea, ysO.
3) A. > O. Entonces ki• 2=
1 V~ Y
-'2 ± i -2- la solución general
de la ecuación (7) es

y=e--r (CtCOS ~I t+C2sen v:); t)


170 ClIP. 111. MeraDOS DIRECTOS

Pasando 8 la variab le x, obtenemos

CIC<>Sr~ In (2x+3)] +c2sen[ ~ln(2x+3}]


y Y2x+3 .
Las condiciones de frontera t5) dan

e1cos (v:¡ In3)+C2s~n (~I In3) =0,)


V- y¡ } (9)
el cos ("-fIn 9) +Czsen (-2- In 9) =0. J
El sistema (9) tendrá soluciones no triviales si su determinante es
igual a cero
y- 1(-
cos ( -f In 3) sen ( -T- In 3)

cos (-2-
vX
·V~In9 ) sen (-2- In 9
) = O;

por lo tanto. sen (Y~ln3- ~¡: In3)=0, es decir, sen x

X
V~ In J) =-_ O, de donde -2-ln
(-2- Vi 3= m'C, Los valores propios serán
4nZn2
x, 1:::I'"i'ii2'3 (n= 1, 2, ... ).

Tomando cualquier ecuación del sistema (9), por ejemplo, la primera,


e introduciendo en ella /..n en lugar de A, obtenemos
Cl cos /I.Jt + e, sen ns: = O,
o sea, el (_I}n = O. de donde el = O. Tomando en (8) el = o y
4n2n2
AII = In23 ' obtenemos las funciones prop las del problema conside-
rado
AA In (2x+ 3) ]
sen [ In 3
(n= 1,2, , .. ).

Los coeftcíentes en se determinan de la condición de normación


3
) Y~l(x) dx= I
O
5 IS. LOS VALORES Y LAS PUNCIONES PROPIOS 17}

lo' que da
c.,. = :i:,Vr-21n3
y, por consiguiente,
M In (2X+3)J
2 sen [ 1n3
Un (x)=± V1ñ3 V2x+3 (n= 1,2, ... ).

Hallar los valores propios y las funciones propias en los


problemas que siguen:
1
224. J [y (x») = 1 (y2+
o
y'2) dx; y (O)= y (1) = O;
t

J !ldx=).
o
2
225. J(y(x)J= 1
t
X2y'2dx; y(1)=y(2)=O;

J y2dx= 1.
i
e
226. J{y(x)l= J (6¡l+X2y'2}dx;
1

y(I)=y(e)=O; ) yadx= 1.
1
2n
227. J (y (x)] = J (y2_y'2)dx; y(1C)=y(2n)=O;
JI
2n
J !ldx=
:n
J.
i
228. J[y(x)l= J 13!1-(x+ 1)2y'1l)dx;
O
1
y(O)=y(I)=O; J y dx=J.
2
Il
172 CAP. 11. MaTO DOS 01 R flCTOS

Los valores pror!OS y las funciones propias del problema varia-


cíonal (3), (2) Y (4 tienen varias propiedades importantes.
1) Si)..,., y t..n son dos valores propios diferentes de la funcional (3)
con las cori(ficiones (2) y (4) Y si 11m (x) Y !/n (x) son las funciones pro-
pIas que les corresponden, estas funcIones 11m (.x) e /In (x) son ortogo-
nales. o sea,
b

J Ym (x) IIn (x) dx= O (m =1< 1\).

2) Todos los valores propios "'n


de la funcional (3) son reales.
3) Si An es un valor propio de la funcional (3) e IIn (x) es la íun-
cl6n propia normada que le corresponde, se tiene
J [Yn (x)] = An'
4) El menor de los valores propios coincide con el mlnimo de la
funcional (3) con las condiciones (2) y (4).
BJEMPLO 2. Demostrar la desigualdad
n
J"o y'?' (x) >- Jo yZ (x) ox, y (O)=y (n)=U.

n
:SOLUCIÓN Determinemos el min J g't. (x) dx con las condiciones
O

'"
) 112(x) dx= 1, !I (O)=y (n)=O.
O

La ~w"~lon de Euler para la funcional


n
I (U (x)J = J (!I'a - Aya) d»
o
tiene la forma
Y· + 1.y = O; y (O) = O, y (n) = O.
Las funciones propias de este último problema son !In (x) = sen nx
y los valores propios son ~ = fll.
El valor propIo mlnlmo es ~ = 1. Por eso, en virtud de la propie-
dad 4),
n
mln J y'2 (x)dx= l.
O
§ 15. LOS VALORES Y LAS FUNCIONES PROPIOS 173

En consecuencia, para cualquier función y (x) tal que r


o
y2 (x) áx = 1

tenemos
n It

~ y';\ (x) dx> J y2 (x) dx.


o o
sen x
Esta desigualdad no se puede precisar ya que para Yl (x)= liñ
se tiene

jo Yi9(x)dx= J y?
o
(x)dx=1.

OBSERVACIÓN. Si 1 .1(

y'J. (x) dx = k2 -+ 1, el prob lema se reduce al

anterior introduciendo la función z (x) = y ~) •


Empleando la definición extremal de los valores propios. señale-
mos cómo pueden ser calculados aproximadamente a partir del método
de Ritz, Debe tenerse en cuenta que el método de Ritz da una aproxi-
mación por exceso del valor propio.
EJEMPLO 3. Hallar aproximadamente el primer valor propio del
problema
y" +
';.."y = 0,
y (-1) = y (1) = O.
SOLUCIÓN. El problema sobre el mlnirno de la funcional

r
t

-1
{J'2 dx

con las condiciones


1
Y ( .- 1) = y (1) =O y 5 y2 dx = I
-t

es un problema isoperimétrico y se reduce al problema sobre el mini-


mo de la funcional
1

JIy (x)) =
-1
J (y'2 - '}..2y2) dx
174 CAP. 111. M~TODOS DIRECTOS

cuya ecuación de Euler coincide con la ecuación dHerencial considera-


da y" +
Aay = O. y (-1) = fI (1) = O.
La solución general de la ecuaci6n es y = Cl cos'Ax Ca sen AX, +
De las condiclones de frontera encontramos

et cos 'A-Cz sen A = O, }


(lO)
C t cos A-\- C2 sen A = O,
de modo que la condición de existencia de una solución no nula del
sistema (lO) es la condición de que sen2A=0, o sea, A=n; •
Por conslqulente, para el primer valor propio tenemos A~=
= ( ~ )2 Y la primera arm6nica de la cuerda viene dada por la solu-
.. exacta
clan t y=cos-y,
1tx. ¡'=Ti
1t 1a segun da arrnonrca
' . es y=sen.nx,
~
A = .n; I a t ercera armonrca
. . es y 3nx
= cos -2-' ,
A = "23 n, etc.
A título de comparación, busquemos las soluciones pares (armóni-
cas pares de la cuerda) aproximadas en forma de un polinomio según
las potencias de x. Tomando las funciones coordenadas en la forma
'PI¡ (x) =x21l-2_x211 (k= 1,2, ... ) minimicemos la funcional J en las
m
funciones 11m(x) =~ Ck'P" (x). Limitándonos al término {JI (x) =
II-t
=c¡!p¡ (x), tendremos J ({JI (x)J=-c~ ( : - !~;.,z) y para determinar
el obtendremos
oJ IYt (x)]
.:Je1
Puesto que debe ser el =F O, resulta A' = 2,5. Tomando para y
Y2 (x) = C¡'Pl (x) + C¡!Pi (x).
encontramos

J [Y2('~)]=C: (~ - !~AZ) +2c l'2 ( 185- 1~5 A2)+

+ei ( 1:- 3\~;'2) ,


y para determinar Ct y C2 obtenemos el sistema

iJJ[yz(xH_
Oc! -Ct
(..!!-~A2)+
3 15 Ca
(~-.E_AZ)=O
15 105 •
...I
iJJ!yz{x))
OC2 el
(J2.._~A2)-j.
15 105 c2
(176_.2!.AZ)=O
105 .315 t
J
S 15. LOS VALORES Y LAS FUNCIONES PROPIOS 175

La condición de existencia de soluciones no nulas el Y c: de este último


sistema es que su determinante sea igual a cero; esto da A4 _ 28)..2 +
+ 63 = O. de donde Al = 2,46744 Y 11,1 = 25.53256. Comparemos los
valores aproximados obtenidos para AP y 11,: con sus valores exactos.
El valor exacto de ).: es ( ~ ) z ~ 2,46740 Y el valor exacto de A~es

(~ ) ~ 22,20661 de modo que el valor aproximado obtenido para


Al es de gran exactitud mientras que para el segundo valor prop ío se
Obtiene una aproximación tosca.
EJEMPLO.. Hallar el primer valor propio del problema
yW + )..
(1 + Xi) !I = 0, y (-1) = !I (1) = O.
SOLUCION. Tomemos como funciones coordenadas las funciones
CJ>A (x) = 1 - x'Jfl (k = l. 2•... ) que satisfacen, obviamente, las
condiciones de frontera. Poniendo
Yi (x) = Cl (1 - x') + c:a(1 - x4).
planteemos el problema sobre la mlntmizaclón de la funcional
1
1 lb' (x)) = J ly'2- h (1 +xZ) y2) dx
-1
que tiene la ecuación dada como ecuación de Euler. Tendremos

J[yz. (x») =cl ( ~ - :~~ ~.) +2clc2 ( 1: - ~: A) 7


2 ( 32 5&3~
+c2 T- 3465 A).
Para determinar CI Y el! obtenemos el sistema
oJ[!l2(X)]
OCI
=2
CI
(!_
3
128 'J..)..L2
104
(...!i_~i.)-O
5
,C2 45 -,
'\I
al [Y2 (x)]
élC2
2
5 45 '"
el
(~_~.)
C2 7
+2
3465 '" - .
(32 _ 5888,) -o J
La condición de existencia de solución no nula de este último sistema
da
52)..20- 1068j" 2079,.." O. +
de donde. tomando la raíz menor. encontramos AJ = 2,1715.
PRINCIPIO De RAJLEIGH. Supongamos que se tiene el problema de
valores prop íos
L(y)=- :'" [P(,X) :~J+q(x)y=~"r(x)y. <11'1

alY (a) ·HtY' (a) =0, cxH-~l > O, }


Ct2Y (o) +~2Y' (b) =O. cx~-Hi > 0, (12)
donde p (x), p' (x), q (x) y r (x) son continuas en [a, nI; p (x) > O
en [a, b).
176 CAP 111. MeTODOS DI'REeTOS

Diremos que la función y (x) es admisible (y E D) si tiene dos


derivadas continuas y satisface las condiciones de frontera (12).
Supongamos que para toda función admisible y (x) se cumple
la condición
b

J yL (y) d:c"> O.
u
En este caso el problema'de contorno (11) - (12) tiene solamente valo-
res propios reales A.
Podemos poner en correspondencia a este problema de valores
propios el siguiente problema varlaclonal:
en tre todas las r unciones admisib les y (x) tajes que
b

J r (:c) y2 dx > O. (13)


a
b
S IIL (y) dx
a
ha 11ar aque lla para la cua 1 b = mí n.
S r (x) g2 dx
a
Sea y = 1I>,'(x) la solución de este problema.
Si Al es el valor mínimo, o sea, si
ti ti

o
S gL (y) dx ~
a
"'tL ('1>.) dx
Al = mío ti ~~b ---
lIED
S r (x) y2 dx !r",~dx
a G
entonces Al es el menor valor propio positivo y "1>1 (x) es la función
propia que le corresponde.
Si a las íuncíones admisibles se impone, a parte de la condición
(13), una condición más
ti
J r1j)¡y dx =O
C1

(condición de ortogonalidad), el problema


b
~ yL(y)dx
_0-,- = mln
~ ry2dx
el

tendrá de nuevo una solución "1>, (x).


§ 16. LOS VALORES Y LAS FUNCIONES PROPIOS 177

'Si A,I es el valor mínimo correspondiente, entonces A, será el


siguiente, en cuanto a la magnitud (A.l ;;;a: ~). valor propio y 'ila (x)
será la función propia ortogonal a '$1 (x) que le corresponde. En gene-
ral. si se conocen ya los k primeros valores propios positivos
A.¡ ~ )'z ~ •. . ~ Ah
Y el sistema ortogonal correspondiente de funciones propias
~ (x), "PI (x), . . ., 'l'k (x).
el valor propio siguiente. será igual a
b
S yL(y) áx
A.Mi = mín _;;;a...
b-_-
l'ED 'i f!J2 dx
(1

con la particularidad de que Se consideran aquellas funciones admi-


sibles y (x) que, a parte de (13), satisfacen las siguientes condiciones
complementarlas
b
J r(x) 1j>'II (x) y(x)dx=O (v=l, 2•...• k).
a
Si en la ecuación (11) se tiene que Ia función r (x) O en la, b) >
con frecuencia se emplea para estimar por arriba el menor valor
propio positivo Al la s ígutente desigualdad (principio de Rayleigh).
b
S !JL (y) dx
Al -< .:.;.1I"'b---

S ry2 dx
a
EJEMPLO 5. Valiéndose del principio de Rayleígh, estimar ~
en el siguiente problema de contorno
_yW = Ay, 1/' (O) = O, y (1) = O.
SOLUCION. En nuestro caso tenemos L (y) = -!t. o sea, P (x) e
;:; l > 0, q (x) := O y r (x) El> O en [O. 1]. Es obvio que ~ = O,
~1 = 1, a, = I Y fls = O de modo que al f¡f = 1 > O y al
+ ~i = 1 > O. Tomemos como función admistble ti (x) = 1 - Xli;
+ +
según el principio de Raylelgh, tendremos
t t
~ yL (y) dx ~ 2 (1 - x2) dx 4
~ c~
O ....:0:..,- = 3 = 2.5.
= -8-
IIot """ 1 1
~ yr'l. dx S (1_x2) dx 15
O O
:n2
Recordemos que el valor exacto es Al =4~ 2,4674.

12-01387
178 CAP. 1lJ. MaTaDOS DIRECTOS

Estimar el menor valor propio en los problemas que


siguen:
229. -y" = J.. (10 - X2) y; y (-1) = y (1) = O.
230. -y" = Ay; Y (O) = Y (1) = O.
En el problema de la determinación de los valores y de las fun-
ciones propios también se puede emplear el método de Kantoróvícn
(método de reducción a ecuaciones diferenciales ordinarias). Suponga-
mos, por ejemplo. que en un recinto D se tiene la ecuacíón
6.z + "-z = O
y que
zlr=O,
donde r es la frontera del recinto D.
Busquemos la solución en la forma
m
Zm (x,. X2 •.... Xn) = 2j a.h (x) IPk (x, y) +q>o (x. y)
k""t
escogiendo las funciones coordenadas q>k (x, y) y las funciones 0'.11 (x),
por ahora incógnitas. de modo que Zm (x, y) se anule en todos los
puntos de r.
Las funciones al (x), al! (z), ...• O'.m (x) deben satisfa-
cer el sistema de ecuaciones

J [Mm+)..zmJIP)I(x,y)dy=O (k=I.2 ..... In) (14)


D:f

y deben anularse en los valores extremos del argumento. Aquí D.'!(


es la in tersección del recinto D y de la recta x = consto .
Aquellos valores de ).. para los cuales el sistema (14) tendrá solu-
ción no trivial darán una aproximación de los valores propios y las
soluclónes correspondientes darán una aproximación de las funciones
propias.
EJEMPLO 6. Hallar aproximadamente el primer valor propio
y la primera función propia en el problema
6.z + AZ = 0, z Ir = O
donde el recinto D es el rectángulo: -a ~ x ~ a, -b '" y ~ b,
SOLUCIÓN, Buscamos la solución del problema en la forma
ZI (x, y) = (y2 - ba) a, (x).
La eucación (14) toma en este caso la lor ma
11
J 120'., + IY2. - ¿'Z) al + ¡, (y2. - b2) al} ly2 - b2) dy = O.
-11
§ 15. LOS VALORES Y I.AS FUNCIONES PROPIOS 179

o sea,
..!ib5ctí+
15
(~b5¡"
15
__ !3 ba) al=O ' ( 15)

al (-a)=at (0)=0.
la solución general de (15) es
ai (x) = C I sen V '}.,---2":O"~Z-X + C2 <:05 V A- 2!;¿ x,
Teniendo en cuenta la simetría del problema y tomando una solución
particular, obtenemos

Ct.:...O y Czcos V i..- 2;2 a=O;


queda claro de aquí que tendremos una solución no trivial sólo si
• /' 5 n
V ¡,- 2b2 a=t2k-I)T;
(2k-I)2 n2 5
hA = (2a)2 -1- 2b2 •
En particular, para k.....,._ 1 encontramos
n2 10
A, = (2a)2 + 2b2
siendo el valor exacto
n2 n.2
Al = (2a)2 + 2b2'
El error es menor que 1.3%.
Para la primera función propia oh tenemos la aproximación
nx
zs(x, y) = (g2_ b2) COS 2Q'

Hallar una aproximación del primer valor propio en los


problemas que siguen.
231. y" +
')..2y = O, Y (O) = y (1) = O.
232. lJ" +
j. (2 +
cos x) y = O, y (O)= y (n) = O.
233. Hallar aproximadamente el primer valor propio en ~I
problema
~z + AZ = O z Ir = O,
donde D es el círculo de radio uno con centro en el origen
de coordenadas.
RESPUESTAS E INDICACIONES
lo al 'mlD =0 en el punto (O. O); b) tmáx = 1 en el punto (O. O);
e) no hay extremo. 2. No hay extremo. 3. f mili = -8 en los puntos
eV2", - V2) y (- V:2,
V2); en el punto (O, O) no hay extremo.
4. !mln =O en el punto (O, O); en los puntos de la circunferencia
x2+y2= 1 hay máximo no estricto. 5, 'máx= en el punto (1,-1). Vs
6. f mtn =4 en el punto (-}, 1, I ) . 7. f mln = - t en el punto (l, O).

R. fmlo-- a- 3113 en el punto


(2n
3' 2rc) ;
3"" fmáx=-g-
3113 enelpunto
n2+n+2

( ~, ;). 9. fmax:= ( n2;-~+2) 2 para x,=X2-='''=X",=


+
:-:;n2 2n+2' 11. No. 13. Los números a.1t Y ~h deben ser los coefi-

cientes de Fourier de la función' (x). 14. tmln = -i- en los puntos

1 ' - y2"1) y (1- V'2' y21) ; fmb.=T I en los puntos


( V2
1 lf2
( V2' 1) y (1- Vii · - Vi1) . 15. fmln=l336 en el punto
18 12 )
( 13 ''""i3 . 16. fmá-x=4 en los puntos (2,2,1), (1,2,2) y (2,1,2);

4
f máx "'"4 27 en los pun 10$ ("3'" 3"4 37) ' (73" "3'
4 3'4) t y
(12
4 3'
(3' 7 3"
4) . 17, fmáx =(!
T • 18. fm1n=1 en el punto

( :' !); fmllx = I1 en ,1 punto ( - ~ , - ~ ). 19. i-: = -9


en el punto (-1 2. - 2); fmñx =9 en el punto (1, -- 2, 2). 20.
RESPUESTAS E INDICACIONES 181

fmáx=i;- en el punto (~, ~, ~). 21. Indicación. Hallar el

mínimo de la función z=+(xn+yn) con la condición x+y=S.

22. c.
, 4V5
23. -5-' 24.
1911'2
8 . 25. El cuadrado de dimensión

a=R'V2. 26. El radio de la base del cilindro r = ~ -,/2+ V'S


y la altura del cilindro h = R V V5' 2- 27. Primero. 28. Proximi-

dad de cualquier orden. 29. Proximidad de cualquier orden. 30. p=e-1•


2n+3
31. p=l. 32. p=e-l. 33. p¡=e-1. 34. P2= 6 .36. PIOOl'=e.

36. Continua. 37. Continua. 38. Discontinua (conSiderar la sucesión

Yn (x) = senn nx) . 39. a) Discontinua; b) continua. 40. a) Discontinua;


l-e2
b) continua. 41. Continua. ",S. llJ=-2-

48. a; O,J 6J
1,2 1 a2
-0,1 -0,098 -0,1 11J =a+ 5; oJ =a.
0,01 0,01002 0,01

49. M 3(e2-1) +6 (3-) 2+~. "J _ 3 (e2_1)


4 a e a 5' u - 4 a·
M f¡,J
4,7919 6,6821
0,1 0,4792 0,4963
0,01 0,0479 0,0481
50. Diferencíable; 2) diferenciable; 3) diferenclable; 4) no di-
1)
[erenciables. 51.{jJ2 [g (x)) = 2J (g (x)] ss.
e
63. M=3k+--1 k2; 6J=3k.
e-
A M sr
I 4,582 3
0,1 0,3158 0,3
0,01 0,03016 0,03
182 R,ESPUESTAS E INDJCACIONES

5 8 5
54. tlJ--k+-
- 3 7' k2. tJJ=:rk
11 AJ 6J
2,810 1,667
0,1 0,181 0,167
0,01 0,0168 0,0167
4
55. M=Tk?; ()J == O.
k 6J M
-1 O 1,3333
0,3 O 0,1200
0,03 O 0,0012
b b
57. 6J= J ~ydx. 58. 6J=2) (y6y-y' Oy'}dx.
a a
1
59. 6J = 2y «» 6y {O) + i (x ~y.+2y' ()y') dx.
O
n
80. 6J = J (!J' cos y 6y +sen y tJy') cU.
o
b
61. (JJ =
rl ( -a-
of 6Yl +-(}
al- 6112+'" +r
af,) 6Yn d.t.
• {Jt 112 !In
a
62. 6ZJ IY, y) = 2) ¡«Sy, 6y) o

63. 62eF(I/)=eF(lI) {(6F)2 + 62FJ.

J~
b m
65. (J2J = LJ ¡J'l·F 6y(lt) 6y(l1 dx.
Oy(lI) ayO)
a /l. 1-0

66. 67-J = Jr Jr (F~ {&)2+F" ,6~ 6z~+... +F" •• (6z;")2J dx dy.


Ux z¡.trv
G
b m. n

67.0'1.)= 1[ ~
a i, 11-1
F;,lIkOy¡8YII+
t,
~
11-1
F;¡lIÍt6Yt{JYÁ+

11
R.ESPUESTAS E INDICACIONES 183

68. Considerar la funcional


J [cp+a.TJJ=11> (a.)
y emplear la segunda definición de la variación. Exigiendo que
6J =0, llegamos a la ecuación integral
b
JK (s, t) q¡ (s] ds+cp (1)- f (t) =0.
a

69. Procediendo de la misma forma que en el problema anterior,


encontramos que la -cuacíón funcional de Euler, que expresa la anula-
ción de la primera var iación, viene dada por
(pq¡')' - CJ' (x+2)- q> (x- 2) +cp (x)+ f (x)=0.
Esta última es una ecuación mixta con derivadas y diferencias.

70. -(pq:')'+qq¡=f(x), 71. y=_x3, 72. y= Sh~~~X) .73.Dos


extremales
1 + (3 + 2 V2) (2x- 1)2
y= 4 (V2 ± 1) •

74. Dos extremales 1I=1"(X+1)2 e y=)'I'"(3x_1)2. 75. y=(C+


I
+x)senx, donde e es Una constan! ... arbitraria. 76. 1I=-rX
7 1 13
X[e-x+(l+e)xe-x-IJ. tt. Y="6"'-6X3, 78. Y=6"'-
1
- '"6 ",3+ 2. 79. 11 = In x, 8]. La integra! no depende del camino
de integración; el problema var iacional carece de sentido. 82. y ....O
si a.=0; siendo a. =fr. 0, no existe extrerna l suave. 83. y=cosx.
84. y=cosx+Csenx. donde C es una constante arbitraria. 86. 1!=
sh x
=x + \. 86 • .11= ShT' 2
87. Y =e (I-x). 88. No hay extrernaíes; la

ecuación de Eu ler no tiene soluciones. 89. y=Cj -t-C2"'- ~z. .


1
90. No hay extreruales. 93. y=C1e:>:+C2e-x-¡ Txex:. 94. U=2chx .

.lit sen x . J
96. y . 97. y=2x. 98. La circunferencia -=K. 99. 11=
~O~ r
x3
=(I-x)shx. 100. U=T{x3+6x+f). 10]. No hay extremo. 102.
184 ~ESPUESTAS E I NDICACION.ES

El problema variacíonal carece de sentido porque bajo el signo de la


integral figura una diferencial exacta. 103. y=sh x,
y=sen 2x,
104. g=+x2• 105. { x2. 32+n2
2=-2+ 8n x.

J06. Y==x-.{- (x3+5x-6l. 107. { !I = sen x,


{
Z z=sen x,
x2.
lOS.
11='""'2+1,
{
z= 1.
j}z ) 2 iJ2z ( iJz ) 2 iJ2z
110. ( Tx 'aX2+ ay ay2 =f(x, y).

n
112.
• 1
a [ aj(x,.
~""'liX""
j
X2 ••••• Xn) ~
ox¡ J +c(x,. X2 •• , '. xu) z=
1-
= f (XI, X2 •••• , Xn)·

113. Solución. El planteam lento del problema es el siguiente.


Entre las superficies z =q> (x. g) que se proyectan en el recinto D del
ptano xOy y que pasan por cierta curva cerrada alabeada cuya pro-
yeccíén es la curva frontera r del recinto D.,!ha llar la superficie 'cuyo
álea

s =j lVI +
!)
IP! + cp; dx dy
sea mfnlmo (problema de Platean). La ecuacién dHerencial de Euler
para este prob lema es

a CP,t: +..!.. IPy o


"1X VI +CP~+CP~ ~g 1f1 +q>!+II'~
o en forma desarrollada
11'_ (1 + II'~)- 2CPzl{q>z<p,,+ CPvv(1 +q>!) =0.
Esta es la ecuación diferencial buscada de las superficies de área
mínimo. La realización ñstca de la superficie de área mlníma se puede
obtener, por ejemplo, con una pelicu la de jabón tendida sobre un lazo
de alambre.
114. Z (x, 11) = y. El problema tiene solución única aunque las
condlcíones de frontera no se dan CIl toda la frontera.
RESPUEST AS E I NDICi\CIONF.$ 185

115. r cos IP+ea = el In I r sen IP+ V,Z sen? IP- en


117. x2 ces C2- y2 cos Ca-2xy sen ez =C,.
1t8. Campo central, 119. a) Campo propio; b) campo central;
e) no forma campo. 120. Campo propio. 121. a) Campo central: b) no
forma campo; e) campo propio. 122. a) Campo central; b) campo pro-
pio; e} no forma campo. 123. No lorma campo porque esta familia
de curvas no cubre lodo el recinto D. 124. y=C¡ eh x forman un campo
propio de extrema les; y = C2 sh x forman un campo centra I de ex tre-
males. 125. y=Ccos.v: forman un campo propio de extremales: y=
= C sen x forman un campo central de extremales. 126. La extrema I
y= ~ (l-x2) puede ser incluida en el campo central de exlremales

y=C¡x~ ~ con centro en el punto O (O, O). 127. La extremal Ij=e>.:


se puede incluir en el campo propio de extremares y=ex+C. 128. Si
a<n, la extremal !I::.~O se puede incluir en el campo central de ex-
trernales y=Csenx con centro en el punto 0(0. O). Si a>n, las cur-
vas de la lami! ia y =C sen x no forman campo. 129. La extrema] y =
=x+ 1 se puede incluir en el campo propio y=x+e. 130. y=
:c - x; . 131. Y ( ~ -x) = O. 132. y?-I =0. 133. 0+ (l. O).
134. No hay punto conjugado. 135. Se cumple. 136. Se cumple cual-
quiera que sea a. 137. La condición de Jacobi se cumple. La extremal
11=0 se puede incluir tanto en un campo central como propio de ex-
trema les. 138. La condición de Jacobl se cumple. La extremal y =
!..=.l.. x+ 1
(l
se puede incluir en un campo central de extremales con
centro en el punto A (O, l). 139. La condición de Jacobi no se cumple.
142. Se puede. 143. Se puede. 144. Se puede. 145. Se puede aunque la
condición de Legendre se cumple sólo para !
< l. 146. Se alcanza
mlnlmo fuerte en la función y=t'J!.. 147. Se alcanza mínimo fuerte
en la función y =21n (x +'1). 148. Se alcanza míntmo débil en la Iun-
b
ción 11 = J;2 • .149. Se alcanza míntmo débil en la recta y=- x ,
a
. In (I+x)
160. Se alcanza mínimo íuerte en la curva y 1n 2 . 151. Se al-
canza máximo fuerte en la curva y=cosx+senx. 152. No se alcanza
extremo en curvas continuas. 153. Se alcanza mínimo débil en la recta
11= 2x +
l. No hay extremo fuerte. 154. Se alcanza min írno fuerte en
la extremal 1I=2x.-1. 156. Se alcanza mlnimo fuerte en la extremal
$t=xa. 166. Se alcanza minimo débil en la ex trema I !I =x-I, 157.
Én la extrernai y =!_ x se alcanza mínimo déb i1 si lb' <~
a V2
y máxímc a~l>j) ;¡i ,b, > -V2 . No hay extremo si b
I I = ';-2 .
186 RESPUESTAS E INDICACIONES

158. Si P =F q, se alcanza mínimo débil en la extremal y=


= {[(lfI2 p3/2'] x+p3/2)2.; si p=q. la extrema! es la recta y=p
que ofrece mínimo déb il.
x
sh -v=;-
159. a) Para 1;:> O la extremal y realiza el rníntmo
2
shVs
1
fuerte de la funcional. b) Si E, <O Y le I > nz' la extrernal U=
x.
sen--=-
V~I t; real iza ('1 máximo ruerte de la funcional. e) Sí e =O.
sen V le I
no existe solucrén di!! problema extremal en la clase de Iunclones
x-I

cont inuas. Consideremos la función Ur, (x) =f! Ve (e> O) que es


solución de la ecuacién de Euler ell"-II=O de la funcional constde-
rada. La función lIe (xl satisface la condición de frontera y (1) = I
y no satisface la segunda condición de frontera y (O)= O. No obstante,
lím !Ir. (O) = O. Para e -+- O obtenemos de !le (x) la esolución límite»
e-()
O, O~x<l.
Y (x) ={ l. x=L
2 In (1 .i, x) ,
160. La extremal 11= - In 2' realiza el mínimo fuerte.
161. Hay mínimo fuerte en la extrernal g (x) = 1. 162. En la extremal
b I •. d •b . f •
g ( x ) =ax se a canza mrmmo e I SI
b V3
a<-2- y rnaxrmo
,- d e•b ilI

. b
51
V3
a> -2-; . a=-2-'
SI
b V3 no se a Icanza rn'..
siquiera e I ex t remo

débil. 163. En 111 recia se alcanza mínimo débil si b «;a


!J"~x
a
y máximo déb il si b> a¡ si b.;? a 'Va.
hay máximo fuerte mientras
que para b < a y3
no hay ni mínimo ni máximo fuertes. 164. Se
alcanza mínimo débil en la extremar U=2x, z=4x. 165. La exlremal
es la parábola { ~:.:~ .-x • que se puede incluir en el campo cen-
tral de extremales

( 1)
RESPUESTAS E 1NDICACTONES 187

a y ~ son unos parámetros) con centro en el punto (O, O, O). Es


obvio que se cumplen las condiciones reforzadas de Legendre. Demos-
tremos que en el segmento O ~x ~ 1 no hay punto x· conjugado del
punto x = O. Para ello bastará persuad irse de que las ex trema les de la
familia (I) no cortan la extrernal dada si xE [O, IJ' Supongamos que
en un punto x· E (O. 1) se cortan dos extremales e la familia (1).
Tendremos entonces
asx· = (%2X·, }
x·2 + ~tx·= x·"+ ~"x·.
De aquí resulta que <x, =(%2 Y que ~. = ~z. Luego, no hay dos extre-
males distintas que se corten. Por consiguiente, la condición reforzada
de Jacob i se cumple en el segmento rO. 1) e incluso en cualquier seg-
mento de lonttud finita. 166. La familia de extremales es y (x) =
=e, ch X~l 2 A. Las constantes arbitrarias el y Cz y el paráme-
tro ). se determinan de las condiciones

e xo-Ca
!lo= ,eh el A.,
lCt

) 1f1"'+7i dx= el (Sh Xl"Z. C 2


Sh
xo-c" )-1
el - .
XI)

167. Y (x) =3X2+2x+J.


168. y(x}=±2sennn,x. donde n es un
I
número entero 169. y(x)=T(2x-xZ). ,r-
170. V 6. 17t. r=R, z=c1 +
+ezCJ)· 172. V4S . ,/"-
173. V 20.114.2
,r
V 2-1. rrs,
VIO
----ro' 178.

V'Ti
--2-.179. Ir
r 17+4 v- (5"2-'1/"-)
6 V 6 . 180. 1. 181. Si COS Xi =F O,
y=O,
el extremo se puede alcanzar solamente en la recta { . En
Z=O
cambio, si cos x¡ =0, o sea Xl = ; + me. donde t1 es un número entero.
y=e,senx,
se tiene { z= -
e,senx, siendo e" una constante arh itraria. 182.

J (A, B) = 4 cth 1. 183. J (A. B) = ~ . 184.!J = 2x2/3• 185. Las líneas


quebradas formadas por los segmentos de las rectas y = x e y=;:; I o por
los segmentos de las rectas y=O e g=x-I realizan el mínimo abso-
luto. La recta Y=TX realiza el máximo débil. 186. y= -)C para
188 Rr;SPUESTAS e INDICACIONES

0<:«1; y=x-2para 1<x-<4cy=xparaO~x~3; y=-x+6


para 3 <x <
4. En ambas quebradas la funcional alcanza su mínimo
O, :<-<0,
a b soIuta. 187. No existen. 188. y = { 189. Las extremales
x, x>O.
sao líneas rectas. Si I YZ-Y1
;(2-Xl
1< 1, existen dos soluciones dlsconti-
nuas que son líneas quebradas paralelas a las bisectrices de los ángu-
los coordenados. 190. La recta y =.dg (j) Que pasa por los puntos fijos
realiza máximo débil si O<tgcp<n, realiza mínimo débil si n<
< tg!jl < 2.n, etc, El mínimo Iuerte se alcanza en la línea quebrada
formada por segmentos de rectas tales que la tangente de los ángulos
de su inclinación es igual a 4n;- I n (n es un número entero).

3 16
±Tx, O "
r '" x""5'
191-
y (xl" t ±~V9.-
_ 3(x-10)
(x- 5)2.

34
16 ,_
5 ",-x.o:::,5'
,/ 34

+ 4 5<x<10.

192. Las exlremales son las elipses

()I

con centros en el eje Ox, La frontera del recinto admisible se deter-


mina por I11S ecuaciones y=O e y2=± 2 (x-ea) (la última es [a
solución de la ecuación 1_.y2y'2=0). Los parámetros e, y e'l, se esco-
gen de modo que la elipse (1) pase por los puntos fijos A y B. La funcional
alcanza máximo en el arco de la elipse. Si el camino del punto A al
punto B se escoge según los arcos de dos parábolas (Y. posiblemente,
según un segmento de la recta y =O), se obtiene una solución con
puntos angulares en la que la funcional alcanza su mínimo (mín J =O).
dy )',2y2 dp x'l.y dy P dp p2
193. dX = 4p2 • dx ='2;)' 194. dx = 2xy' dx = 4xy'l. •

195• .!!!L P , dp = !J • J96. dY1 =


dx 11x2"¡' y2_ p? dx 11",2+ y2_ p2 dx

= P21• dPl =0
dx 'dx
dY2 =..E!.
2'
dP2 =2y . 197.
dx 2
dYI =...f!!_ ddYx2=
dx 2Yt'
P2 e/PI ...EL dp2 p~ d!1I PI dyz
~ 2!h ' '""dT = 2yf ' -¡¡;- --- 4y~ 198. "'(IX = - T' Tx =

¡ = -11,0;, :1 ~ 2x, ':!;2 -O: 'J'xf .- ~t. ~~2 =11P'l..


~ESPUESTAS E INDICACIONES 189

:~I =2x, ~~ =0. 199. yJ=C1x3+C2. 200. y3=ln2x. 201. x""


dy x2_x_1
=C,
J 11G2(y)_Q +C2• 202. En la extremal 11= 2 s~


a 1canza e 1 mmimc ruer t e: • J = --¡-
mm 5 . 203'. p (x, y) = 112-
2xyXl •

Las extremales son las semicircunferencias Y=Vq-(X-C2)2 con


-v
centros en el eje Ox; JI = 2Ctx ·_x2 son las exlremales que pasan
por el origen 0(0. O); el campo es el semiplano superior. 204. El arco
de la circunferencia que pasa por el punto MI (Xi. JI,) Y que tiene el
centro en el punto O (O. O) realiza el mínimo íuerte. 205. xF (~) =C.
206. Las elipses 3x2-8xy+6y2=C. 207. x3+2g3-3xy2_2x2y=C.
208. I =

==v( *+
V¡-::¡:yt2. 209. I =xy

;2) (X211'2.+1I2).
La integral de la acción es J =
V
y'. 210. l =xyy'2. 21 t.

212. Una catenaria.

J JI' p~ +2h
213. Indicación.

V p2_f-p'2dcp.
t=

214. Las

2 ces fi sen2 ~
trayectorias son las el ipses -C
XZ
+ -2h2y2
-
C - V
. C (2h-C)
xy =- ..
- .
1<

215. La solución exacta es 11= :~ ~ - x, 216. Las soluciones exactas

son a) y c= O, b) y=x. 217. La solución exacta es JI =. ~ (x2-x). 218.

La solución exacta es y= 2s~h; -x. 219. Indicación: la solución apto-


rt
xímada debe buscarse en la forma 11n (x) = (l_x2) ~ altx211. La 50-
11... 0

lución exacta es y = cos n: . 220. Indicación: tomar xy como función


b2-a2 • ,
coordenada; entonces, Zt b2+a2 xl/. 221. Indicación: tomar. IPo(x,y)=
=X2+y2, cpdx, y)=xy(l-x-y), lP2,(x,y}=x2y(l-x-y) •...
..•• cp" (x, g) =xny(l-x-y} como (unciones coordenadas; entonces,
l3 (x, y) = xz+ 1I2+xy (l-x- 11)[3,0401-0.0562 (x+x2)]. 222. Indica-
ción: hallar la primera aproximación en la forma zl (x, y) = ( y2 _

_ ~2) a(x); entonces 4dx. y) = _ ~ (1- ~) (112- ;2 ).


190 RESPUESTAS E INDICACIONES

223. 2'1
3 ( yZ-'31)
(x, y)='4 x2 (I._ 35 2.35
1_36X-I_3&X-5-1 )
. 224. "-n=
=1 + n .nZ 2• Un (x) = ± V'2 sen nnx (n = 1, 2, " .). 225. h.n =

_lnz2+4nZr¡Z l= sen (To;-Inx) 226' 25+2n2.nZ


- 4I z2 ' !In (x - _ _. • "'n = 4 '
n Ylny2Vx
fin (X )-_ +
_
V'2 senVi(nn In x) (n -
000 I , 2• •••.
) 2"7'
lO. "'/1 = 1-I\,!/n
Z
(X
)
=

=± V! sen sx (1\= 1, 2, ... ).


228.
nn In (I+x) ]
131n22+4n2n2 sen [ In 2
41n22 ' !In (x) = ± ,r----'¡=,r
V In V 2 V 1 +x
(n=I,2, ... ).

229. Tomando !I = 1- x~, encontramos Al -< 13 :S. El valor exacto


I
es Al = T' 230. Tomando y = x (1- x), encontramos Al 10. El valor -<
exacto es Al =:n:2• 23'. Al = 10¡ el valor exacto es i..f = n2• 232. Af=
= 0,493. 233. Al= 6; Zt (x, JI) = IX (x2 -1- y2 - 1).
A NUESTROS LECTORES:

MIR-RUBIÑOS 1860 edita libros soviéticos tradu-


cidos al español. Entre ellos figuran las mejores obras
de las distintas ramas de la ciencia y la técnica: ma-
nuales para los centros de enseñanza superíor y escue-
las tecnológicas; literatura sobre ciencias naturales y
médicas. También se incluyen monografías, libros de
divulgación científica y ciencia ficción.
Dirijan sus opiniones a MIR-RUBIÑOS 1860.
el. Alcalá, 98 - 28009 Madrid.
Al principio de cada capitulo
se resumen los resultados prin-
cipales. se exponen los conoci-
mientos teóricos necesarios, las
rórmulas requeridas y se estu-
dian con gran detalle ejemplos
típicos ilustrativos.
Este manual contiene más de
100 ejemplos analizados y 230
problemas destinados para resol-
verse independientemente. Unos
problemas se acompañan con
las respuestas, otros, con las
reíereneiss de cómo deben re-
solverse.
La obra está destinada para los
estudiantes de centros de ense-
ñanza técnica superior que se
especializan en los cálculos ma-
tcmétíees.
ISBN 84-604-1605-4

You might also like